0% found this document useful (0 votes)
118 views

02B Polynomials

This document defines polynomials and discusses their basic properties. A polynomial is an algebraic expression with terms that are natural number powers of a variable. The degree of a polynomial is the highest power of the variable. Polynomials can be added, subtracted, multiplied, and divided. Special cases of polynomials include monomials (single term), binomials (two terms), and trinomials (three terms). Polynomials are usually written in standard form with terms in descending order of variable powers. The constant term of a polynomial is independent of the variable.

Uploaded by

Azizmanva
Copyright
© © All Rights Reserved
Available Formats
Download as PDF, TXT or read online on Scribd
0% found this document useful (0 votes)
118 views

02B Polynomials

This document defines polynomials and discusses their basic properties. A polynomial is an algebraic expression with terms that are natural number powers of a variable. The degree of a polynomial is the highest power of the variable. Polynomials can be added, subtracted, multiplied, and divided. Special cases of polynomials include monomials (single term), binomials (two terms), and trinomials (three terms). Polynomials are usually written in standard form with terms in descending order of variable powers. The constant term of a polynomial is independent of the variable.

Uploaded by

Azizmanva
Copyright
© © All Rights Reserved
Available Formats
Download as PDF, TXT or read online on Scribd
You are on page 1/ 88

POLYNOMIALS

09 OCTOBER 2021
REVISION: 2130

AZIZ MANVA
[email protected]

ALL RIGHTS RESERVED


Aziz Manva ([email protected])

Table of contents
Part I: Operations 3 6.1 Division 42
6.2 Remainders 45
1. BASICS ...................................................... 3 6.3 𝒙𝟐 + 𝒙 + 𝟏 47

1.1 Definition 3 Part III: Equations 51


1.2 Coefficients 6
1.3 Degree 8 7. EXPRESSIONS AND EQUATIONS ........ 51
1.4 Roots 9
7.1 Expressions 51
1.5 Graphing Polynomials 13
7.2 Cubic and Quartic Equations 51
2. ADDITION AND SUBTRACTION ......... 15 7.3 Rational Roots Theorem 54
7.4 Misc: Equations in Two Variables 59
2.1 Basics 15 7.5 Misc: Solving in terms of unknown Constants 59
2.2 Degree 16 7.6 Graphs 60

3. MULTIPLICATION ................................ 19 Part IV: Roots 62


3.1 Multiplication Algorithms 19
3.2 Coefficients 19 8. POLYNOMIAL ROOTS .......................... 62
3.3 Degree 20 8.1 Vieta’s Formulas 62
8.2 Roots meeting conditions 67
4. DIVISION ............................................... 22 8.3 Vieta’s Formulas: Higher Degree 70
4.1 Polynomial Long Division 22 8.4 Transformation of Roots 72
4.2 Synthetic Division 23
4.3 Division Formula-I 25 9. MORE THEOREMS ............................... 77
4.4 Division Formula-II 28 9.1 Descartes’ Rule of Signs 77
4.5 Applications 31 9.2 Rational Inequalities: Wavy Curve Method 77
4.6 Remainders 32
4.7 Summary 32 Part V: Identities 78
Part II: Remainder and Factor 10. IDENTITIES ......................................... 78
Theorem 33 10.1 Cube of a Sum/Difference 78
10.2 Sum/Difference of Cubes 80
5. LINEAR DIVISORS................................ 33 10.3 Trinomial Identities 81
5.1 Basics 33 10.4 Applications 84
5.2 Synthetic Substitution 37 10.5 Sophie Germain Identity 86
5.3 Finding Coefficients 37 10.6 AMC Questions 86

6. HIGHER DEGREE DIVISORS ............... 42

Questions Added to Tracker

P a g e 2 | 88
Aziz Manva ([email protected])

PART I: OPERATIONS
1. BASICS
A. Learning Note
There are many definitions in this Note. These definitions are not difficult, but need to be understood and
remember to solve numerical questions

1.1 Definition
A. Polynomial
An algebraic expression made of more than one term is called a polynomial. The powers in a polynomial must
be:
𝑁𝑎𝑡𝑢𝑟𝑎𝑙 𝑁𝑢𝑚𝑏𝑒𝑟𝑠: 𝑛 ∈ ℕ

Some examples of polynomials are:


4𝑥 2 + 3𝑥 − 6
𝑥 3 − 12
𝑥+5
3𝑥
12

1.1: Polynomial
An expression of the form:
𝑃(𝑥) = 𝑎𝑛 𝑥 𝑛 + 𝑎𝑛−1 𝑥 𝑛−1 + ⋯ + 𝑎1 𝑥 + 𝑎0 , 𝑛∈ℕ
Is a polynomial in the variable 𝑥 with degree 𝑛
With
𝑇𝑒𝑟𝑚𝑠 𝑎𝑟𝑒 𝑤𝑖𝑡ℎ 𝑡ℎ𝑒 𝑣𝑎𝑟𝑖𝑎𝑏𝑙𝑒: 𝑎𝑛 𝑥 𝑛 , 𝑎𝑛−1 𝑥 𝑛−1 , … , 𝑎1 𝑥, 𝑎0
𝐶𝑜𝑒𝑓𝑓𝑖𝑐𝑖𝑒𝑛𝑡𝑠 𝑎𝑟𝑒 𝑤𝑖𝑡ℎ𝑜𝑢𝑡 𝑡ℎ𝑒 𝑣𝑎𝑟𝑖𝑎𝑏𝑙𝑒:
𝑎𝑛 ≠ 0

1.2: Terms
The terms in a polynomial are separated by plus or minus signs. The terms in the polynomial
𝑃(𝑥) = 𝑎𝑛 𝑥 𝑛 + 𝑎𝑛−1 𝑥 𝑛−1 + ⋯ + 𝑎1 𝑥 + 𝑎0 , 𝑛∈ℕ
Are
: 𝑎𝑛 𝑥 𝑛 , 𝑎𝑛−1 𝑥 𝑛−1 , … , 𝑎1 𝑥, 𝑎0

Terms are stated with the variable, if they have one.

1.3: Coefficients
The numbers which multiply with the variables in a polynomial
𝑃(𝑥) = 𝑎𝑛 𝑥 𝑛 + 𝑎𝑛−1 𝑥 𝑛−1 + ⋯ + 𝑎1 𝑥 + 𝑎0 , 𝑛∈ℕ
are the coefficients.
𝑎𝑛 , 𝑎𝑛−1 , … , 𝑎0

Coefficients are without the variable.

As we will see below, polynomials with real coefficients can have complex roots. But, we will not consider
polynomials with complex coefficients (for this chapter).

P a g e 3 | 88
Aziz Manva ([email protected])

Example 1.4
For each polynomial, identify the terms, the coefficients, and the number of terms
A. 4𝑥 2 + 3𝑥 − 6
2
B. 5𝑥 2 − 77 𝑥 + 31
C. 𝑎𝑥 3 + 𝑏𝑥 2 + 𝑐𝑥 + 𝑑

Part A

𝐹𝑖𝑟𝑠𝑡 𝑇𝑒𝑟𝑚 = 4𝑥 2
𝑆𝑒𝑐𝑜𝑛𝑑 𝑇𝑒𝑟𝑚 = 3𝑥
𝑇ℎ𝑖𝑟𝑑 𝑇𝑒𝑟𝑚 = −6

𝐶𝑜𝑒𝑓𝑓𝑖𝑐𝑖𝑒𝑛𝑡𝑠: 4,3, −6
Part B
2
5𝑥 2 , −𝑥, 31
77
2
𝐶𝑜𝑒𝑓𝑓𝑖𝑐𝑖𝑒𝑛𝑡𝑠: 5, − , 31
77

Part C

𝑎𝑥 3 , 𝑏𝑥 2 , 𝑐𝑥, 𝑑

Example 1.5
Is the expression
3𝑥 2 + 2𝑎𝑥 + 4𝑎2
A. A polynomial in 𝑥
B. A polynomial in 𝑎
C. A polynomial in either 𝑎, or 𝑥, depending upon the interpretation
D. A polynomial in neither 𝑎, nor 𝑥

Option C

Rewrite 3𝑥 2 + 2𝑎𝑥 + 4𝑎2 as a polynomial in 𝑎 in standard form

4𝑎2 + 2𝑎𝑥 + 3𝑥 2

1.6: Constant Term in a Polynomial


The 𝑡𝑒𝑟𝑚 𝑖𝑛𝑑𝑒𝑝𝑒𝑛𝑑𝑒𝑛𝑡 𝑜𝑓 𝑥 in the polynomial
𝑃(𝑥) = 𝑎𝑛 𝑥 𝑛 + 𝑎𝑛−1 𝑥 𝑛−1 + ⋯ + 𝑎1 𝑥 + 𝑎0 , 𝑛∈ℕ
Is called the constant term

When we say independent of 𝑥, we mean that as the value of 𝑥 changes, the value of the term does not change.

Example 1.7
𝑃(𝑥) = 4𝑥 2 + 3𝑥 − 6
A. If 𝑥 = 0, what is the value of the last term?

P a g e 4 | 88
Aziz Manva ([email protected])

B. If 𝑥 = 23, what is the value of the last term?

𝑥 = 0 ⇒ 𝑃(𝑥) = 4(0)2 + 3(0) − 6 ⇒ 𝐿𝑎𝑠𝑡 𝑇𝑒𝑟𝑚 = −6


𝑥 = 23 ⇒ 𝑃(𝑥) = 4(23)2 + 3(23) − 6 ⇒ 𝐿𝑎𝑠𝑡 𝑇𝑒𝑟𝑚 = −6

Example 1.8
For each polynomial, identify the constant term:
A. 4𝑥 2 + 3𝑥 − 6
2
B. 5𝑥 2 − 𝑥 + 31
77
C. 𝑎𝑥 3 + 𝑏𝑥 2 + 𝑐𝑥 + 𝑑

B. Special Cases

1.9: Monomial
A monomial is a polynomial with a single term:
➢ A Variable (𝑒𝑔: 𝑦, 𝑥 2 )
1
➢ A number (𝑒𝑔: 7, )
2
1
➢ Or a product of a number and a variable (2 𝑥 2 , 7𝑦)

1.10: Binomial
A binomial is a polynomial with two terms:
➢ (𝑎 + 𝑏)
➢ (7𝑥 3 + 3𝑦 2 )

1.11: Trinomial
A trinomial is a polynomial with three terms:
➢ 𝑎2 + 𝑎𝑏 + 𝑏 2

C. Standard Form

1.12: Standard Form


➢ In standard form, a polynomial is written in descending powers of the variable.
➢ Like terms have been added together

Example 1.13
Write the following in standard form:
A. 5 + 𝑥 + 𝑥 2
B. 𝑥 4 + 𝑥 7 − 9
C. 5 + 𝑥 + 𝑥 2 + 3

𝑥2 + 𝑥 + 5
𝑥7 + 𝑥4 − 9
𝑥2 + 𝑥 + 8

Example 1.14

P a g e 5 | 88
Aziz Manva ([email protected])

Decide whether the following are in standard form:


A. 3𝑥 2 + 2𝑥 2 + 5

Example 1.15
The sum of the roots of the equation 4𝑥 2 + 5 − 8𝑥 = 0 is equal to: (AHSME 1950/3)
The sum of the roots of the quadratic 𝑎𝑥 2 + 𝑏𝑥 + 𝑐 = 0 is given by:
𝑏

𝑎
5
Explain the “trick” in this question, and why − is not the answer.
4

𝑎𝑥 2 + 𝑏𝑥 + 𝑐 = 0
Is written in standard form.

But
4𝑥 2 + 5 − 8𝑥 = 0
Is not written in standard.

1.2 Coefficients
A. Leading Coefficient

1.16: Leading Coefficient


The first coefficient when the polynomial is written in standard form is called the leading coefficient.

Example 1.17
What is the leading coefficient in the following polynomials:
A. 3𝑥 4 + 7𝑥 5
B. 0𝑥 5 + 3𝑥 4 + 7
C. 0𝑥 7 − 3𝑥 2 + 4𝑥 6

𝐴: 7𝑥 5 + 3𝑥 4 → 7
𝐵: 3𝑥 4 + 7 → 3

1.18: Monic Polynomial


A monic polynomial is a polynomial where the leading coefficient is 1.

Example 1.19
Consider the equation below
3𝑥 3 + 27𝑥 + 81𝑥 2 + 243 = 0
where the left hand is not a monic polynomial. Rewrite the equation so that the left side is a monic polynomial,
and write the polynomial in standard form.

Divide both sides by 3:


𝑥 3 + 27𝑥 2 + 9𝑥 + 81 = 0

Example 1.20

P a g e 6 | 88
Aziz Manva ([email protected])

Consider the equation below


1 3 1 1 2 3
𝑥 + 𝑥+ 𝑥 + =0
64 16 128 256
where the left hand is not a monic polynomial. Rewrite the equation so that the left side is a monic polynomial,
and write the polynomial in standard form.

1 3
𝑥 3 + 𝑥 2 + 4𝑥 + = 0
2 4
B. Coefficients
The numbers that multiply with the variables are called coefficients.

Example 1.21
Consider
𝑃(𝑥) = 3𝑥 2 + 5𝑥 + 4
A. What is the sum of the coefficients?
B. What is the product of the coefficients?

Part A
𝑃(𝑥) = 3𝑥 2 + 5𝑥 + 4𝑥 0
Hence, the last term (4) is also a coefficient.

To find the sum of the coefficients, you substitute 𝑥 = 1:


𝑆𝑢𝑚 = 3 + 5 + 4 = 12

Part B
𝑃𝑟𝑜𝑑𝑢𝑐𝑡 = 3 × 5 × 4 = 60

Multiple Choice Multiple Correct 1.22


The quadratic polynomial which has a non-zero constant term is:
𝑃(𝑥) = 𝑎𝑥 2 + 𝑏𝑥 + 𝑐
Has product of its coefficients zero. Then, which of the following can be true:
A. 𝑎 = 0
B. 𝑏 = 0
C. 𝑎𝑏 = 0
D. 𝑎𝑐 = 0
E. 𝑏𝑐 = 0

Product of coefficients is zero


𝑎𝑏𝑐 = 0 ⇒ 𝑎 = 0 𝑂𝑅 𝑏 = 0 𝑂𝑅 𝑐 = 0

Polynomial is quadratic
𝑎≠0
And the question tells us
𝑐≠0
This means that
𝑏=0

Hence, the right answer is:


𝑂𝑝𝑡𝑖𝑜𝑛 𝐵, 𝑂𝑝𝑡𝑖𝑜𝑛 𝐶, 𝑂𝑝𝑡𝑖𝑜𝑛 𝐸

P a g e 7 | 88
Aziz Manva ([email protected])

Multiple Choice Multiple Correct 1.23


The quadratic polynomial which has a non-zero constant term
𝑃(𝑥) = 𝑎𝑥 2 + 𝑏𝑥 + 𝑐
Has sum of its coefficients zero. Then, classify each of the following as:
1. Can be true
2. Must be true
3. Must be false

A. 𝑎=0
B. 𝑏=0
C. 𝑎𝑏 = 0
D. 𝑎𝑐 = 0
E. 𝑏𝑐 = 0

𝑎 = 0 ⇒ 𝐹𝑎𝑙𝑠𝑒
𝑏 = 0 ⇒ 𝐶𝑎𝑛 𝑏𝑒 𝑡𝑟𝑢𝑒
𝑎𝑏 = 0 ⇒ 𝐶𝑎𝑛 𝑏𝑒 𝑡𝑟𝑢𝑒
𝑎𝑐 = 0 ⇒ 𝐹𝑎𝑙𝑠𝑒
𝑏𝑐 = 0 ⇒ 𝐶𝑎𝑛 𝑏𝑒 𝑡𝑟𝑢𝑒

1.3 Degree
A. Degree

1.24: Degree
The degree of a polynomial is the value of the highest power in the polynomial.

Example 1.25
Is the condition that 𝑎𝑛 ≠ 0 required for a polynomial? If yes, explain why

Otherwise, you can change the degree of the polynomial to any degree that you wish.

Example 1.26
Determine the degree of the following expressions:
A. 4𝑥 2 + 5𝑥 + 6
B. 7𝑥 5
C. 𝑎𝑥 4 + 𝑏𝑥 5 + 𝑐
D. 𝑎𝑥 5! + 𝑏𝑥 4! + 𝑐

𝐴: 2
𝐵: 5
𝐶: 5
𝐷: 120

Example 1.27
Determine the degree and the maximum number of terms of:
A. A Quadratic Polynomial
B. A Cubic Polynomial

P a g e 8 | 88
Aziz Manva ([email protected])

𝑄𝑢𝑎𝑑𝑟𝑎𝑡𝑖𝑐: 𝑎𝑥 2 + 𝑏𝑥 + 𝑐 → 𝐷𝑒𝑔𝑟𝑒𝑒 2, 𝑇𝑒𝑟𝑚𝑠 3


𝐶𝑢𝑏𝑖𝑐: 𝑎𝑥 3 + 𝑏𝑥 2 + 𝑐𝑥 + 𝑑 → 𝐷𝑒𝑔𝑟𝑒𝑒 3, 𝑇𝑒𝑟𝑚𝑠 4

Example 1.28
In a polynomial of degree 𝑛, what is the:
A. Maximum number of non-zero terms?
B. Minimum number of non-zero terms?

𝑃(𝑥) = 𝑎𝑛 𝑥 𝑛 + 𝑎𝑛 𝑥 𝑛−1 + ⋯ + 𝑎1 𝑥 + 𝑎0
𝑀𝑎𝑥 𝑇𝑒𝑟𝑚𝑠: 𝑛 + 1
𝑀𝑖𝑛 𝑇𝑒𝑟𝑚𝑠: 0
1.4 Roots
A. Definition

1.29: Roots of a Polynomial


The roots of a polynomial 𝑃(𝑥) are the values that satisfy
𝑃(𝑥) = 0

Example 1.30
Determine the roots of the polynomial:
3𝑥 + 5

5 5
3𝑥 + 5 = 0 ⇒ 𝑥 = − ⇒ 𝑅𝑜𝑜𝑡 𝑖𝑠 −
3 3

Example 1.31
Determine the roots of the polynomial:
2 4 3 7
( 𝑥 + ) (− 𝑥 + )
3 5 4 2

2 4 3 7
( 𝑥 + ) (− 𝑥 + ) = 0
3 5 4 2
Apply the zero-product property:
2 4 2 4 4 3 6
𝑥+ =0⇒ 𝑥=− ⇒𝑥=− × =−
3 5 3 5 5 2 5
3 7 3 7 7 4 14
− 𝑥+ =0⇒− 𝑥=− ⇒𝑥= × =
4 2 4 2 2 3 3
6 14
𝑅𝑜𝑜𝑡𝑠 𝑎𝑟𝑒 {− , }
5 3

Example 1.32
Determine the roots of the polynomial:
𝑃(𝑥) = 𝑐𝑥 2 + 𝑏𝑥 + 𝑎

−𝑏±√𝑏2 −4𝑎𝑐
Substitute 𝑎 = 𝑐, 𝑏 = 𝑏, 𝑐 = 𝑎 in the quadratic formula 𝑥 = 2𝑎
:
−𝑏 ± 2
√𝑏 − 4𝑎𝑐
𝑥=
2𝑐

P a g e 9 | 88
Aziz Manva ([email protected])

Example 1.33
Determine the number of roots and the degree of the polynomial
A. (𝑥 − 𝛼1 )(𝑥 − 𝛼2 ) … (𝑥 − 𝛼𝑛 )
B. (𝑥 − 𝛼1 )(𝑥 − 𝛼2 )(𝑥 − 𝛼𝑛 )

𝐴: 𝑛 𝑟𝑜𝑜𝑡𝑠, 𝑑𝑒𝑔𝑟𝑒𝑒 𝑛
𝐴: 3 𝑟𝑜𝑜𝑡𝑠, 𝑑𝑒𝑔𝑟𝑒𝑒 3
B. Number and Type of Roots

1.34: Number of Roots


A polynomial of degree 𝑛 has exactly 𝑛 roots.
➢ Roots may be real or complex
➢ Roots may be repeated

We will not prove this here (though it can be proven).

Example 1.35
A polynomial of degree 12 has exactly 4 real roots. How many complex roots does it have?

12 − 4 = 8

Example 1.36
What is the number of roots of the polynomial:
27 2 26 12
𝑥 + 𝑥+
26 27 19

𝑄𝑢𝑎𝑑𝑟𝑎𝑡𝑖𝑐: 2 𝑟𝑜𝑜𝑡𝑠

Example 1.37
A polynomial is of degree 35. What is the number of roots it has?

35

1.38: Multiplicity of Roots


If a root of a polynomial occurs more than once, it is said to have the corresponding multiplicity.

Example 1.39
What is the multiplicity of the root −5 in the following polynomials:
A. 𝑥 + 5
B. (𝑥 + 5)2 (𝑥 − 5)

Part A
𝑥 + 5 = 0 ⇒ 𝑥 = −5 ⇒ 𝑀𝑢𝑙𝑡𝑖𝑝𝑙𝑖𝑐𝑖𝑡𝑦 𝑖𝑠 𝑂𝑛𝑒

Part B
𝑥 + 5 = 0 ⇒ 𝑥 = −5 ⇒ 𝑀𝑢𝑙𝑡𝑖𝑝𝑙𝑖𝑐𝑖𝑡𝑦 𝑖𝑠 𝑂𝑛𝑒
(𝑥 + 5)2 ⇒ 𝑥 = −5 ⇒ 𝑀𝑢𝑙𝑡𝑖𝑝𝑙𝑖𝑐𝑖𝑡𝑦 𝑖𝑠 𝑇𝑤𝑜

P a g e 10 | 88
Aziz Manva ([email protected])

1.40: Odd and Even Multiplicity of Roots


➢ A root that occurs an odd number of times has odd multiplicity.
➢ A root that occurs an even number of times has even multiplicity.

Example 1.41
3
6 2 4 2 7 2020 2020 2019 2021
(3𝑥 − 4) ( 𝑥 − ) ( 𝑥 − ) ( 𝑥− )
7 5 3 9 2019 2020
A. What is the degree of the polynomial?
B. What is the leading coefficient of the polynomial? Give your answer in unsimplified form.
C. State each root of the polynomial.
D. Determine for each root whether it has odd or even multiplicity.

Parts A and B
𝐷𝑒𝑔𝑟𝑒𝑒 = 2021 + 2020 + 4 + 3 = 4048
6 2 2020 6 × 2 × 2020
𝐿𝑒𝑎𝑑𝑖𝑛𝑔 𝐶𝑜𝑒𝑓𝑓𝑖𝑐𝑖𝑒𝑛𝑡 = 3 × × × =
7 3 2019 7 × 2019

Parts C and D
To find the roots, we equate polynomial to zero:
6 2 4 2 7 2020 2020 2019 2021
(3𝑥 − 4)3 ( 𝑥 − ) ( 𝑥 − ) ( 𝑥− ) =0
7 5 3 9 2019 2020

Apply the zero-product property:


𝑎𝑏𝑐𝑑 = 0 ⇒ 𝑎 = 0 𝑂𝑅 𝑏 = 0 𝑂𝑅 𝑐 = 0 𝑂𝑅 𝑑 = 0

Take the first Term


(3𝑥 − 4)3 = 0
Take Cube Roots both sides:
4
3𝑥 − 4 = 0 ⇒ 𝑥 = , 3 𝑖𝑠 𝑜𝑑𝑑 ⇒ 𝑂𝑑𝑑 𝑀𝑢𝑙𝑡𝑖𝑝𝑙𝑖𝑐𝑖𝑡𝑦
3
6 2 4 6 2 2 7 7
( 𝑥− ) =0⇒ 𝑥− =0⇒𝑥= × = , 4 𝑖𝑠 𝑒𝑣𝑒𝑛 ⇒ 𝐸𝑣𝑒𝑛 𝑀𝑢𝑙𝑡𝑖𝑝𝑙𝑖𝑐𝑖𝑡𝑦
7 5 7 5 5 6 15
2 7 2020 2 7 7 3 7
( 𝑥− ) =0⇒ 𝑥− =0⇒𝑥= × = , 2020 𝑖𝑠 𝑒𝑣𝑒𝑛 ⇒ 𝐸𝑣𝑒𝑛 𝑀𝑢𝑙𝑡𝑖𝑝𝑙𝑖𝑐𝑖𝑡𝑦
3 9 3 9 9 2 6

2020 2019 2021


𝑥− ( ) =0
2019 2020
2020 2019 2019 2019 2019 2
𝑥− =0⇒𝑥= × =( ) , 2021 𝑖𝑠 𝑜𝑑𝑑 ⇒ 𝑂𝑑𝑑 𝑀𝑢𝑙𝑡𝑖𝑝𝑙𝑖𝑐𝑖𝑡𝑦
2019 2020 2020 2020 2020

1.42: Forming Polynomial with Given Roots


If the roots of a polynomial are
𝛼1 , 𝛼2 , … , 𝛼𝑛
Then the polynomial is
(𝑥 − 𝛼1 )(𝑥 − 𝛼2 ) … (𝑥 − 𝛼𝑛 )

Example 1.43

P a g e 11 | 88
Aziz Manva ([email protected])

Form the polynomial with


𝑅𝑜𝑜𝑡 3 𝑜𝑓 𝑀𝑢𝑙𝑡𝑖𝑝𝑙𝑖𝑐𝑖𝑡𝑦 2
𝑅𝑜𝑜𝑡 − 2 𝑜𝑓 𝑀𝑢𝑙𝑡𝑖𝑝𝑙𝑖𝑐𝑖𝑡𝑦 3

𝑃(𝑥) = (𝑥 − 3)2 (𝑥 + 2)3

1.44: Complex Roots occur in pairs


➢ Complex roots of a polynomial with real coefficients occur only in pairs.
➢ These pairs are called complex conjugates of each other.

Every complex root has associated with it, one more root, which is its complex conjugate. For instance, in a
quadratic, the roots are:
−𝑏 + √𝑏 2 − 4𝑎𝑐 −𝑏 − √𝑏 2 − 4𝑎𝑐
,
2𝑎 2𝑎

Complex roots are roots which have not only real parts but also imaginary parts. Complex numbers are written
in the form
𝑎 + 𝑏𝑖, 𝑖 = √−1

In general, complex conjugates are of the form:


𝑎 + 𝑏𝑖, 𝑎 − 𝑏𝑖

Example 1.45
Consider the polynomial with roots 4 and 5𝑖. Does it violate the condition that complex roots occur only in pairs?

(𝑥 − 4)(𝑥 − 5𝑖) = 𝑥 2 − 4𝑥 − 5𝑖𝑥 + 2𝑖


And as we can see above, the coefficients are complex.
Hence, the condition is not violated.

Example 1.46
Is it possible that a polynomial of degree 12 with real coefficients has exactly 5 real roots?

𝑁𝑜. 𝑜𝑓 𝐶𝑜𝑚𝑝𝑙𝑒𝑥 𝑅𝑜𝑜𝑡𝑠 = 12 − 5 = 7 ⇒ 𝑂𝑑𝑑 ⇒ 𝑁𝑜𝑡 𝑝𝑜𝑠𝑠𝑖𝑏𝑙𝑒

C. Revision: Nature of Roots of A


Quadratic Nature of D

1.47: Discriminant of a Quadratic


The discriminant of the quadratic 𝑦 =
Positive Zero Negative
𝑎𝑥 2 + 𝑏𝑥 + 𝑐 is
𝑏 2 − 4𝑎𝑐
Perfect Not a Perfect Real, Equal & Complex
Example 1.48 Square Square Repeated Conjugates
List the possible cases for the roots of a
quadratic in terms of whether they are
Rational, Irrational,
real or complex.
Distinct Distinct

Example 1.49
P a g e 12 | 88
Aziz Manva ([email protected])

List the possible cases for the roots of a quadratic with real distinct roots in terms of whether they are irrational
or real

D. Nature of Roots of a Cubic

Example 1.50
Explain why a cubic cannot have either no real roots, or two real roots.
Nature of
The number of roots that a cubic has is Roots
3
Recall that complex roots occur in pairs. Hence, the only possible One Real Three Real
values for the number of complex roots: Root Roots
0 𝑎𝑛𝑑 2
Two Complex No Complex
The corresponding value for the number of real roots is: Conjugates Roots
3 𝑎𝑛𝑑 1

Example 1.51
List the possible cases for the roots of a cubic in terms of whether they are real or complex.

1.52: Number of Real Roots of a Polynomial


➢ A polynomial with even degree must have an even number of real roots.
➢ A polynomial with odd degree must have an odd number of real roots.

1.5 Graphing Polynomials


A. Zeroes of a Function

1.53: Zeroes of a Polynomial Function


The zeroes of a polynomial function are the roots of the corresponding polynomial equation.

Example 1.54

1.55: Cross versus Bounce


The graph of a polynomial will
➢ Bounce at the 𝑥-axis at a zero, if the zero has even multiplicity
➢ Cross at the 𝑥-axis at a zero, if the zero has odd multiplicity

Example 1.56
Use a graphing calculator to sketch the polynomial with roots
𝑅𝑜𝑜𝑡 3 𝑜𝑓 𝑀𝑢𝑙𝑡𝑖𝑝𝑙𝑖𝑐𝑖𝑡𝑦 2
𝑅𝑜𝑜𝑡 − 2 𝑜𝑓 𝑀𝑢𝑙𝑡𝑝𝑙𝑖𝑐𝑖𝑡𝑦 3

P a g e 13 | 88
Aziz Manva ([email protected])

𝑃(𝑥) = (𝑥 − 3)2 (𝑥 + 2)3


B. End Behavior of a Polynomial
The leading coefficient determines the behavior of the polynomial for very large
values (∞) and very negative values (−∞).

1.57: Polynomials of even degree


If the leading coefficient is
➢ positive, then the polynomial is positive at ±∞
➢ negative, then the polynomial is negative at ±∞

1.58: Polynomials of odd degree


If the leading coefficient is
➢ positive, then the polynomial is positive at ∞, and negative at −∞
➢ negative, then the polynomial is negative at ∞, and positive at −∞

P a g e 14 | 88
Aziz Manva ([email protected])

2. ADDITION AND SUBTRACTION


2.1 Basics
A. Basics

2.1: Adding and Subtracting Polynomials


We add polynomials term by term. Terms with like coefficients can be added.

Example 2.2
Given that
𝑃(𝑥) = 3𝑥 2 − 5𝑥 + 3
𝑄(𝑥) = 2𝑥 2 − 4𝑥 + 7
Determine the value of:
A. 𝑃(𝑥) + 𝑄(𝑥)
B. 𝑃(𝑥) − 𝑄(𝑥)

𝑃(𝑥) + 𝑄(𝑥) = 5𝑥 2 − 9𝑥 + 10
𝑃(𝑥) − 𝑄(𝑥) = 𝑥 2 − 𝑥 − 4

Example 2.3
The expressions below are polynomials written in standard form:
𝑃(𝑥) = 𝑥 999 + 2𝑥 996 + 3𝑥 993 + ⋯ + 𝑎
𝑄(𝑥) = 𝑥 998 + 2𝑥 996 + 3𝑥 994 + ⋯ + 𝑏
A. Determine the value of 𝑎.
B. Determine the value of 𝑏.
C. Determine the first six terms of 𝑃(𝑥) + 𝑄(𝑥).
D. Determine the number of terms in 𝑃(𝑥) + 𝑄(𝑥).
Part A
𝑃(𝑥) = 𝟏𝑥⏟ 999 + 𝟐𝑥⏟ 996 + 𝟑𝑥 ⏟ 993 + ⋯ + ⏟
𝑎
1𝑠𝑡 𝑇𝑒𝑟𝑚 𝟐𝒏𝒅 𝑻𝒆𝒓𝒎 𝟑𝒓𝒅 𝑻𝒆𝒓𝒎 𝑎 𝑡ℎ 𝑇𝑒𝑟𝑚
Note that the coefficients have a pattern
𝑉𝑎𝑙𝑢𝑒 𝑜𝑓 𝑐𝑜𝑒𝑓𝑓𝑖𝑐𝑖𝑒𝑛𝑡 = 𝑇𝑒𝑟𝑚 𝑁𝑢𝑚𝑏𝑒𝑟 𝑓𝑟𝑜𝑚 𝑡ℎ𝑒 𝑅𝑖𝑔ℎ𝑡
Look at the exponents:
999, 996,993 = {3 × 333, 3 × 332, … ,3 × 0} ⇒ 𝑎 = 334

Part B
𝑄(𝑥) = 𝑥 998 + 2𝑥 996 + 3𝑥 994 + ⋯ + 𝑏

2.4: Addition is Negative Subtraction


𝑥 − 𝑦 = 𝑥 + (−𝑦)

Example 2.5
Find 𝑅(𝑥) given that:
𝑃(𝑥) = 3𝑥 2 + 4𝑥 + 7
𝑄(𝑥) = 2𝑥 2 − 4𝑥 + 3
𝑆(𝑥) = 𝑃(𝑥) − 𝑄(𝑥) = 𝑃(𝑥) + 𝑅(𝑥)

𝑆(𝑥) = 𝑃(𝑥) − 𝑄(𝑥) = 𝑃(𝑥) + [−𝑄(𝑥)]

P a g e 15 | 88
Aziz Manva ([email protected])

Hence, by comparing, we know that:


𝑅(𝑥) = −𝑄(𝑥) = −(2𝑥 2 − 4𝑥 + 3) = −2𝑥 2 + 4𝑥 − 3
2.2 Degree
2.6: Degree Function
For the sake of simplifying notation, introduce a function 𝑑 that takes a polynomial as an input and gives the
degree of the polynomial as an output.
𝑑[𝑃(𝑥)] = 𝐷𝑒𝑔𝑟𝑒𝑒 𝑜𝑓 𝑃𝑜𝑙𝑦𝑛𝑜𝑚𝑖𝑎𝑙 𝑃(𝑥)

Example 2.7
Evaluate
A. 𝑑(2𝑥 2 + 5𝑥 − 7)
B. 𝑑(7𝑥 99 + 5𝑥 − 7)
C. 𝑑(𝑥 − 7)
D. 𝑑(−7)

𝑑(2𝑥 2 + 5𝑥 − 7) = 2
𝑑(7𝑥 99 + 5𝑥 − 7) = 99
𝑑(𝑥 − 7) = 1
𝑑(−7) = 0

Example 2.8
Write the degrees of the following functions:
A. Quadratic
B. Cubic
C. Linear
D. Constant

𝑄𝑢𝑎𝑑𝑟𝑎𝑡𝑖𝑐 ⇒ 𝐷𝑒𝑔𝑟𝑒𝑒 2
𝐶𝑢𝑏𝑖𝑐 ⇒ 𝐷𝑒𝑔𝑟𝑒𝑒 3
𝐿𝑖𝑛𝑒𝑎𝑟 ⇒ 𝐷𝑒𝑔𝑟𝑒𝑒 1
𝐶𝑜𝑛𝑠𝑡𝑎𝑛𝑡 ⇒ 𝐷𝑒𝑔𝑟𝑒𝑒 0

2.9: Maximum Function


The maximum function returns the greatest of its inputs.
𝑀𝑎𝑥(𝑎, 𝑏, 𝑐) = 𝐺𝑟𝑒𝑎𝑡𝑒𝑠𝑡 𝑉𝑎𝑙𝑢𝑒 𝑎𝑚𝑜𝑛𝑔 𝑎, 𝑏, 𝑐

2.10: Minimum Function


The minimum function returns the smallest of its inputs.
𝑀𝑖𝑛(𝑎, 𝑏, 𝑐) = 𝑆𝑚𝑎𝑙𝑙𝑒𝑠𝑡 𝑉𝑎𝑙𝑢𝑒 𝑎𝑚𝑜𝑛𝑔 𝑎, 𝑏, 𝑐

Example 2.11
Find
A. 𝑀𝑎𝑥(𝜋 2 , 10)
𝜋
B. 𝑀𝑖𝑛 (1.5, )
2
C. 𝑀𝑖𝑛(−3, −2)

P a g e 16 | 88
Aziz Manva ([email protected])

𝑀𝑎𝑥(𝜋 2 , 10) = 𝑀𝑎𝑥(≈ 9.85,10) = 10


𝜋 3.14 1.57
𝑀𝑖𝑛 (1.5, ) = 𝑀𝑖𝑛 (1.5, ≈ ) 𝑀𝑖𝑛 (1.5, ≈ ) = 1.5
2 2 2
𝑀𝑖𝑛(−3, −2) = −3

2.12: Degree of Result


Consider two polynomials 𝑃(𝑥) and 𝑄(𝑥) with:
𝑑(𝑃(𝑥)) = 𝑝, 𝑑(𝑄(𝑥)) = 𝑞
Then
𝑀𝑎𝑥(𝑑[𝑃(𝑥) + 𝑄(𝑥)]) = 𝑀𝑎𝑥(𝑝, 𝑞)
𝑀𝑖𝑛(𝑑[𝑃(𝑥) + 𝑄(𝑥)]) = 0

Example 2.13
If I add a fifth degree polynomial to a third degre polynomial:
A. What is the highest value of the degree of the answer?
B. What is the lowest value of the degree of the answer?

2.14: Degree is Zero


If
𝑑[𝑃(𝑥) + 𝑄(𝑥)] = 0
Then if you 𝑟𝑒𝑚𝑜𝑣𝑒 𝑡ℎ𝑒 𝑐𝑜𝑛𝑠𝑡𝑎𝑛𝑡 𝑡𝑒𝑟𝑚
𝑃(𝑥) = −𝑄(𝑥)

Example 2.15
𝑃(𝑥) = 3𝑥 + 5
𝑄(𝑥) = −3𝑥 + 7

𝑃(𝑥) + 𝑄(𝑥) = 3𝑥 + 5(−3𝑥 + 7) = 12

Example 2.16
𝑑[𝑃(𝑥) + 𝑄(𝑥)] = 0
What is the relation between 𝑃(𝑥) and 𝑄(𝑥)

𝑑[𝑃(𝑥) + 𝑄(𝑥)] = 0
This means that the degree of their sum is zero. Hence, for some constant c, we must have:
𝑃(𝑥) + 𝑄(𝑥) = 𝑐 ⇒ 𝑃(𝑥) = 𝑐 − 𝑄(𝑥)

Hence, the two polynomials only differ by a constant.

Example 2.17
𝑃(𝑥) = 2𝑥 2 + 8𝑥 − 4
𝑑[𝑃(𝑥) + 𝑄(𝑥)] = 0

If 𝑄(𝑥) is a polynomial with constant term 6, find 𝑄(𝑥).

𝑃(𝑥) + 𝑄(𝑥) = 𝐶 ⇒ 𝑄(𝑥) = 𝐶 − 𝑃(𝑥) = 𝐶 − (2𝑥 2 + 8𝑥 − 4) = −2𝑥 2 − 8𝑥 + 4 + 𝐶

𝑄(𝑥) = −2𝑥 2 − 8𝑥 + 6

P a g e 17 | 88
Aziz Manva ([email protected])

Example 2.18
𝑃(𝑥) is a polynomial of degree 𝑝, and 𝑄(𝑥) is a polynomial of degree 𝑞.
A. What is the range of degrees that 𝑃(𝑥) + 𝑄(𝑥) can take?
B. What is the product of the minimum value that the degree of 𝑃(𝑥) + 𝑄(𝑥) can take, and the maximum
value that the degree of 𝑃(𝑥) + 𝑄(𝑥) can take?

𝑀𝑎𝑥(𝑝, 𝑞) = 𝑛 ⇒ 𝑅𝑎𝑛𝑔𝑒{0,1,2, … , 𝑛}
𝑃𝑟𝑜𝑑𝑢𝑐𝑡 = 𝑀𝑖𝑛 × 𝑀𝑎𝑥 = 0 × 𝑛 = 0

Example 2.19
Consider polynomials 𝑃(𝑥) and 𝑄(𝑥) such that the sum of the polynomials is equal to the degree of the sum of
the polynomials. That is:
𝑃(𝑥) + 𝑄(𝑥) = 𝑑[𝑃(𝑥) + 𝑄(𝑥)], 𝑑 𝑖𝑠 𝑡ℎ𝑒 𝑑𝑒𝑔𝑟𝑒𝑒 𝑓𝑢𝑛𝑐𝑡𝑖𝑜𝑛
Then find:
𝑃(𝑥) + 𝑄(𝑥)

Method I
For some constant c:
𝑑[𝑃(𝑥) + 𝑄(𝑥)] = 𝑐
𝑃(𝑥) + 𝑄(𝑥) = 𝑐

But degree of c is 0. Hence


𝑐 = 0 = 𝑃(𝑥) + 𝑄(𝑥)

Method II
We do not know anything about 𝑃(𝑥) or 𝑄(𝑥). However, we can give the range of 𝑑[𝑃(𝑥) + 𝑄(𝑥)], which is:
𝑅𝑎𝑛𝑔𝑒{0,1,2, … }

Suppose
𝑑[𝑃(𝑥) + 𝑄(𝑥)] = 0 ⇒ 𝑃(𝑥) + 𝑄(𝑥) = 0 ⇒ 𝑊𝑜𝑟𝑘𝑠
𝑑[𝑃(𝑥) + 𝑄(𝑥)] = 1 ⇒ 𝑃(𝑥) + 𝑄(𝑥) = 1 ⇒ 𝐷𝑜𝑒𝑠 𝑛𝑜𝑡 𝑊𝑜𝑟𝑘
𝑑[𝑃(𝑥) + 𝑄(𝑥)] = 2 ⇒ 𝑃(𝑥) + 𝑄(𝑥) = 2 ⇒ 𝐷𝑜𝑒𝑠 𝑛𝑜𝑡 𝑊𝑜𝑟𝑘

P a g e 18 | 88
Aziz Manva ([email protected])

3. MULTIPLICATION
3.1 Multiplication Algorithms

3.2 Coefficients
A. Coefficient of a Single Term

Example 3.1
Consider the product
(2𝑥 4 − 3𝑥 3 + 7𝑥 2 + 3𝑥 + 6)(−5𝑥 4 + 2𝑥 3 + 5𝑥 2 + 8𝑥 − 3)
Find the coefficient of the:
A. constant term
B. 𝑥 term
C. 𝑥 2 term
D. 𝑥 5 term
E. 𝑥 7 term

Constant Term
6(−3) = 18

𝑥 term
3(−3) + 8(6) = −9 + 48 = 39

𝑥 2 term
7(−3) + 3(8) + 6(5) = −21 + 24 + 30 = 33
𝑥 5 term
2(8) − 3(5) + 7(2) + 3(−5) = 16 − 15 + 14 − 15 = 0
𝑥 7 term
2(2) − 3(−5) = 4 + 15 = 19

Example 3.2
What is the coefficient of 𝑥 3 when 𝑥 4 − 3𝑥 3 + 5𝑥 2 − 6𝑥 + 1 is multiplied by 2𝑥 3 − 3𝑥 2 + 4𝑥 + 7 and the like
terms are combined? (MathCounts 2005 Warm-Up 10)

(−3)(7) + (5)(4) + (−6)(−3) + (1)(2) = −21 + 20 + 18 + 2 = 19


B. Back Calculations

Example 3.3
If (𝑥 2 − 𝑘)(𝑥 + 𝑘) = 𝑥 3 + 𝑘(𝑥 2 − 𝑥 − 5) and 𝑘 ≠ 0, what is the value of 𝑘? (MathCounts 2009 Warm-up 10)

The given equality is true for all values of x, and hence the coefficients must be equal on both sides. Since, we
have only one variable, we only need to consider the constant terms:
⏟2 = −5𝑘
−𝑘 ⏟⇒𝑘=5
𝑳𝑯𝑺 𝑹𝑯𝑺

C. Sum of Coefficients

3.4: Sum of Coefficients

P a g e 19 | 88
Aziz Manva ([email protected])

To find the sum of the coefficients of an expression, substitute:


𝑥=1

Example 3.5
If (𝑥 + 2)(3𝑥 2 − 𝑥 + 5) = 𝐴𝑥 3 + 𝐵𝑥 2 + 𝐶𝑥 + 𝐷, what is the value of 𝐴 + 𝐵 + 𝐶 + 𝐷?(MathCounts 1993 Chapter
Sprint)

Substitute 𝑥 = 1:
(1 + 2)(3 − 1 + 5) = (3)(7) = 21

Example 3.6
Find the sum of the coefficients of:
(𝑥 + 1)50

Substitute 𝑥 = 1:
(1 + 1)50 = 250
D. Sum of Coefficients: Application

Example 3.7
If the product (3𝑥 2 − 5𝑥 + 4)(7 − 2𝑥) can be written in the form 𝑎𝑥 3 + 𝑏𝑥 2 + 𝑐𝑥 + 𝑑, where 𝑎, 𝑏, 𝑐, 𝑑 are real
numbers, then find 8𝑎 + 4𝑏 + 2𝑐 + 𝑑. (AOPS Alcumus, Algebra, Polynomial Multiplication)

Substitute 𝑥 = 2:
18

3.3 Degree
A. Degree

3.8: Degree of Product of Two Polynomials


𝑑[𝑃(𝑥)𝑄(𝑥)] = 𝑑[𝑃(𝑥)] + 𝑑[𝑄(𝑥)]

The highest power is what matters when deciding the degree.


(𝑎𝑛 𝑥 𝑛 + 𝑎𝑛−1 𝑥 𝑛−1 + ⋯ )(𝑎𝑚 𝑥 𝑚 + 𝑎𝑚−1 𝑥 𝑚−1 + ⋯ ) ⇒ 𝑎𝑛 𝑎𝑚 𝑥 𝑛+𝑚 + ⋯

3.9: Degree of Power of a Polynomial


[𝑃(𝑥)]𝑛 ℎ𝑎𝑠 𝑑𝑒𝑔𝑟𝑒𝑒 = 𝑛 × 𝑑[𝑃(𝑥)]

(𝑎𝑛 𝑥 𝑛 + 𝑎𝑛−1 𝑥 𝑛−1 + ⋯ )2 = (𝑎𝑛 𝑥 𝑛 )2 + ⋯

Example 3.10
Consider the polynomials
𝑃(𝑥) = 3𝑥 4 + 2𝑥 3 − 4𝑥 2 + 7
𝑄(𝑥) = 7𝑥 3 − 2𝑥 2 + 𝑥 + 4

What is the degree of:


A. 𝑃(𝑥)𝑄(𝑥)

P a g e 20 | 88
Aziz Manva ([email protected])

Example 3.11
Consider polynomial 𝑃(𝑥), which has degree 4. What is the degree of:
A. [𝑃(𝑥)]2
B. [𝑃(𝑥)]5

𝐷𝑒𝑔𝑟𝑒𝑒 𝑜𝑓 [𝑃(𝑥)]2 = 𝐷𝑒𝑔𝑟𝑒𝑒 𝑜𝑓 [𝑃(𝑥)𝑃(𝑥)] = 4 + 4 = 8


𝐷𝑒𝑔𝑟𝑒𝑒 𝑜𝑓 [𝑃(𝑥)]5 = 4 × 5 = 20

Example 3.12
Consider polynomial 𝑃(𝑥), which has degree 5, and polynomial 𝑄(𝑥), which has degree 7. What is the degree of:
A. 𝑃(𝑥)𝑄(𝑥)
B. [𝑃(𝑥)]2 [𝑄(𝑥)]3

5 + 7 = 12
[𝑃(𝑥)]2 [𝑄(𝑥)]3 = 10 + 21 = 31
B. Application: Counting

Example 3.13
Suppose that 𝑃(𝑥) and 𝑄(𝑥) are polynomials and 𝑃(𝑥)𝑄(𝑥) has degree 10. If 𝑎 = 𝑑[𝑃(𝑥)] and 𝑏 = 𝑑[𝑄(𝑥)], then
how many values can the ordered pair (𝑎, 𝑏) take?

𝑎 + 𝑏 = 10
(0,10), (1,9), … (10,0) ⇒ 11 𝑆𝑜𝑙𝑢𝑡𝑖𝑜𝑛𝑠

In the above question, if the polynomials are of distinct degree, then what is the number of ordered pairs (𝑎, 𝑏)?

We are not ok with


(5,5)
Hence, the number of solutions is
11 = 1 − 10

In the above question, what is the number of unordered pairs (𝑎, 𝑏)?

(0,10), (1,9), … (5,5) ⇒ 6 𝑆𝑜𝑙𝑢𝑡𝑖𝑜𝑛𝑠

P a g e 21 | 88
Aziz Manva ([email protected])

4. DIVISION
4.1 Polynomial Long Division
A. Division

Example 4.1
Divide using factorization:
𝑥 2 +5𝑥+6
A. 𝑥+3
𝑥 2 +5𝑥+8
B.
𝑥+3

𝑥 2 + 5𝑥 + 6 (𝑥 + 2)(𝑥 + 3)
= =𝑥+2
𝑥+3 𝑥+3
𝑥 2 + 5𝑥 + 8 (𝑥 + 2)(𝑥 + 3) 2 2
= + =𝑥+2+
𝑥+3 𝑥+3 𝑥+3 𝑥+3

4.2: Polynomial Division


𝑃(𝑥) 𝑟
= 𝑄(𝑥) + for Quotient 𝑄(𝑥), and Remainder 𝑟
𝑥−𝑎 𝑥−𝑎

When we divide a polynomial


𝑃𝑜𝑙𝑦𝑛𝑜𝑚𝑖𝑎𝑙 → 𝑃(𝑥)
by a linear expression
𝑙𝑖𝑛𝑒𝑎𝑟 𝑒𝑥𝑝𝑟𝑒𝑠𝑠𝑖𝑜𝑛 → 𝑥 − 𝑎
we get another polynomial
𝑃𝑜𝑙𝑦𝑛𝑜𝑚𝑖𝑎𝑙 → 𝑄(𝑥)
And a Remainder
𝑅𝑒𝑚𝑎𝑖𝑛𝑑𝑒𝑟 − 𝑟

B. Polynomial Long Division


In the previous example, you knew the factorization, and hence were able to cancel. Suppose you do not know
the factorization. You can still carry out the division.

Example 4.3
Simplify by using Polynomial Long Division
𝑥 2 +5𝑥+8
A. 𝑥+3

P a g e 22 | 88
Aziz Manva ([email protected])

4.4: Degree
In polynomial division, consider
𝑃(𝑥) 𝑅(𝑥)
= 𝑄(𝑥) + , 𝑃(𝑥) → 𝐷𝑖𝑣𝑖𝑑𝑒𝑛𝑑, 𝐷(𝑥) → 𝐷𝑖𝑣𝑖𝑠𝑜𝑟, 𝑅(𝑥) → 𝑅𝑒𝑚𝑎𝑖𝑛𝑑𝑒𝑟
𝐷(𝑥) 𝐷(𝑥)
Given that
𝐷(𝑥) ℎ𝑎𝑠 𝑑𝑒𝑔𝑟𝑒𝑒 𝑛
Then:
𝐷𝑒𝑔𝑟𝑒𝑒 𝑜𝑓 𝑅(𝑥) 𝑐𝑎𝑛 𝑏𝑒 𝑚𝑎𝑥𝑖𝑚𝑢𝑚 (𝑛 − 1)
𝐷𝑒𝑔𝑟𝑒𝑒 𝑜𝑓 𝐷(𝑥) + 𝐷𝑒𝑔𝑟𝑒𝑒 𝑜𝑓 𝑄(𝑥) = 𝐷𝑒𝑔𝑟𝑒𝑒 𝑜𝑓 𝑃(𝑥)

Example 4.5
Carry out the division
𝑥 3 + 2𝑥 2 + 5𝑥 + 3
𝑥+2

𝑥 3 + 2𝑥 2 + 5𝑥 + 3 7
= 𝑥2 + 5 −
𝑥+2 𝑥+2

Example 4.6
Carry out the division
2𝑥 3 + 3𝑥 + 5
𝑥−1

4.2 Synthetic Division


A. Algorithm
Synthetic division focuses on the numbers that are used in polynomial long division. The result (and the
calculations) are exactly the same, but the writing work is a little shorter.

Example 4.7
Divide using Synthetic Division

P a g e 23 | 88
Aziz Manva ([email protected])

𝑥 2 + 5𝑥 + 8
𝑥+3

Clever Method

𝑥 2 + 5𝑥 + 6 2 (𝑥 + 3)(𝑥 + 2) 2 2
+ = + =𝑥+2+
𝑥+3 𝑥+3 𝑥+3 𝑥+3 𝑥+3

𝑥 + 3 = 0 ⇒ 𝑥 = −3

Original 𝑥2 𝑥 𝐶𝑜𝑛𝑠𝑡𝑎𝑛𝑡
Polynomial
1 5 8
−3 −3 −6
1 2 2
Answer 𝑥 𝐶𝑜𝑛𝑠𝑡𝑎𝑛𝑡 Remainder

Example 4.8
Divide using Synthetic Division
2𝑥 3 + 3𝑥 + 5
𝑥−1

𝑥−1=0⇒𝑥 =1
𝐷𝑖𝑣𝑖𝑑𝑒𝑛𝑑 = 2𝑥 3 + 0𝑥 2 + 3𝑥 + 5

2 0 3 5
1 2 2 5
2 2 5 10

B. Variables

Example 4.9
If 𝑥 2 + 𝑎𝑥 + 6 is divisible by 𝑥 + 2, then find the value of 𝑎.

Because this is a quadratic, we can multiply easily to find the value of a.


𝑥 2 + 𝑎𝑥 + 6 = (𝑥 + 2)(𝑥 + 𝑐) = 𝑥 2 + (2 + 𝑐)𝑥 + 2𝑐
𝑥 2 + 𝑎𝑥 + 6 = 𝑥 2 + (2 + 𝑐)𝑥 + 2𝑐

Match the constant terms:


2𝑐 = 6 ⇒ 𝑐 = 3 ⇒ 𝑎 = 2 + 𝑐 = 2 + 3 = 5

Do the above question using synthetic division

𝑥2 𝑥 𝐶𝑜𝑛𝑠𝑡𝑎𝑛𝑡

P a g e 24 | 88
Aziz Manva ([email protected])

Equate the factor to zero, and find the root: Term


𝑥 + 2 = 0 ⇒ 𝑥 = −2 −2 1 𝑎 6
−2 −2𝑎 + 4
However, since 𝑥 + 2 is a factor of 𝑥 2 + 𝑎𝑥 + 6, the remainder has to 1 𝑎−2 10 − 2𝑎
be zero.
Hence,
10 − 2𝑎 = 0 ⇒ 2𝑎 = 10 ⇒ 𝑎 = 5

Example 4.10
If 𝑥 2 + 𝑎𝑥 + 6 is divisible by 𝑥 − 2, then find the value of 𝑎 using synthetic division.
3
Equate the factor to zero, and find the root: 𝑥2 𝑥 𝐶𝑜𝑛𝑠𝑡𝑎𝑛𝑡
𝑥−2=0⇒𝑥 =2 Term
2 1 𝑎 6
2 2 4 + 2𝑎
However, since 𝑥 − 2 is a factor of 𝑥 + 𝑎𝑥 + 6, the remainder has to
be zero. 1 𝑎+2 10 + 2𝑎
Hence,
10 + 2𝑎 = 0 ⇒ 2𝑎 = −10 ⇒ 𝑎 = −5

Example 4.11
If 𝑥 3 + 𝑎𝑥 2 − 5𝑥 − 6 is divisible by 𝑥 + 3, then find the value of 𝑎 using synthetic division.

𝑥 + 3 = 0 ⇒ 𝑥 = −3
−3 1 a -5 -6
9𝑎 − 18 = 0 ⇒ 9𝑎 = 18 ⇒ 𝑎 = 2 −3 9 − 3𝑎 9𝑎 − 12
1 𝑎−3 4 − 3𝑎 9𝑎 − 18

Example 4.12 (Calculator Allowed)


If 𝑥 4 + 𝑎𝑥 3 + 𝑏𝑥 2 − 5𝑥 + 14 is divisible by 𝑥 2 + 8𝑥 + 7, then find the value of 𝑎 and 𝑏.

-7 1 a b -5 14
-7 49 − 7𝑎 −343 + 49𝑎 − 7𝑏 2436 − 343𝑎 + 49𝑏
1 -7+a 49 − 7𝑎 + 𝑏 −348 + 49𝑎 − 7𝑏 2450 − 343𝑎 + 49𝑏

4.3 Division Formula-I


A. A Division Pattern

Example 4.13
Divide
𝑥6 − 1
𝑥−1

P a g e 25 | 88
Aziz Manva ([email protected])

𝑥6 − 1
= 𝑥5 + 𝑥4 + 𝑥3 + 𝑥2 + 𝑥 + 1
𝑥−1

B. Formula-I

4.14: Division Formula-I


𝑥𝑛 − 1
= 𝑥 𝑛−1 + 𝑥 𝑛−2 + 𝑥 𝑛−3 + ⋯ + 1
𝑥−1

𝑥 − 1 ≡ 0 (𝑚𝑜𝑑 𝑥 − 1)
Add 1 to both sides:
𝑥 ≡ 1 (𝑚𝑜𝑑 𝑥 − 1)
Multiply LHS by 𝑥, and RHS by 1 (𝑛 − 1) times:
𝑥 𝑛 ≡ 1 (𝑚𝑜𝑑 𝑥 − 1)
Subtract 1 from both sides:
𝑥 𝑛 − 1 ≡ 0 (𝑚𝑜𝑑 𝑥 − 1)

We can also prove the above property using synthetic division

𝑥 𝑛 − 1 = 𝑥 𝑛 + 0𝑥 𝑛−1 + 0𝑥 𝑛−2 + ⋯ + 0𝑥 − 1

𝑥−1=0⇒𝑥 =1

𝑥𝑛 𝑥 𝑛−1 . . . 𝑥0
1 1 0 0 0 0 −1
1 1 1 1 1
1 1 1 1 1 0
Answer 𝑥 𝑛−1 𝑥 𝑛−2 . . . Remainder

Example 4.15

P a g e 26 | 88
Aziz Manva ([email protected])

𝑥3 − 1
𝑥−1

𝑥3 − 1
= 𝑥2 + 𝑥 + 1
𝑥−1

Example 4.16
Divide:
𝑥6 − 1
𝑥−1

𝑥6 − 1
= 𝑥5 + 𝑥4 + 𝑥3 + 𝑥2 + 𝑥 + 1
𝑥−1

What is the number of the terms in the quotient above?

Example 4.17
What is the number of terms in the quotient when we carry out the division in the expression below:
𝑥 2022 − 1
𝑥−1

𝑥 2021 + 𝑥 2020 + ⋯ + 1 ⇒ 2022 𝑇𝑒𝑟𝑚𝑠

4.18: Number of Terms


𝑥 𝑛 −1
𝑥−1
= 𝑥 𝑛−1 + 𝑥 𝑛−2 + 𝑥 𝑛−3 + ⋯ + 1 has
𝑛 𝑡𝑒𝑟𝑚𝑠

Example 4.19
Divide
𝑥 7 +2
A. 𝑥−1
𝑥 7 −5
B. 𝑥−1

𝑥7 − 1 3 3
+ = (𝑥 6 + 𝑥 5 + ⋯ + 1) + ⇒ 𝑅𝑒𝑚𝑎𝑖𝑛𝑑𝑒𝑟 = 3
𝑥−1 𝑥−1 𝑥−1
𝑥7 − 1 −4 −4
+ = (𝑥 6 + 𝑥 5 + ⋯ + 1) + ⇒ 𝑅𝑒𝑚𝑎𝑖𝑛𝑑𝑒𝑟 = −4
𝑥−1 𝑥−1 𝑥−1

Example 4.20
When 𝑥 13 + 1 is divided by 𝑥 − 1, the remainder is: (AHSME 1950/20)

𝑥 13 + 1 𝑥 13 − 1 2 2
= + = (𝑥 12 + 𝑥 11 + ⋯ + 1) +
𝑥−1 𝑥−1 𝑥−1 𝑥−1

P a g e 27 | 88
Aziz Manva ([email protected])

𝑄𝑢𝑜𝑡𝑖𝑒𝑛𝑡 = 𝑥 12 + 𝑥 11 + ⋯ + 1
𝑅𝑒𝑚𝑎𝑖𝑛𝑑𝑒𝑟 = 2

What is the sum of the coefficients of the quotient?

The quotient is this:


𝑥 12 + 𝑥 11 + ⋯ + 1
Substitute 𝑥 = 1 to find the sum of the coefficients:
1 + 1 + ⋯ 1 = 1 × 13 = 13

13 𝑇𝑖𝑚𝑒𝑠

What is the sum of the exponents of the terms in the quotient that have non-zero coefficient?

All terms in the quotient have non-zero coefficient.


12(13)
12 + 11 + ⋯ + 1 + 0 = = 6 × 13 = 781
2

4.21: Multiplication Formula-I


𝑥 𝑛 − 1 = (𝑥 − 1)(𝑥 𝑛−1 + 𝑥 𝑛−2 + 𝑥 𝑛−3 + ⋯ + 1)

We can rearrange the division formula above to get a formula that will let us multiply.

Example 4.22
(𝑥 − 1)(𝑥 2021 + 𝑥 2020 + ⋯ + 1)

𝑥 2022 − 1
= 𝑥 2021 + 𝑥 2020 + ⋯ + 1
𝑥−1
Multiply both sides by 𝑥 − 1:
𝑥 2022 − 1 = (𝑥 − 1)(𝑥 2021 + 𝑥 2020 + ⋯ + 1)

4.4 Division Formula-II


A. Odd Values of 𝒏

4.23: Division Formula-II


𝑥𝑛 + 1
= 𝑥 𝑛−1 − 𝑥 𝑛−2 + 𝑥 𝑛−3 − ⋯ + 1, 𝑛 𝑖𝑠 𝑜𝑑𝑑 𝑛𝑎𝑡𝑢𝑟𝑎𝑙 𝑛𝑢𝑚𝑏𝑒𝑟
𝑥+1
Note the signs alternate.

𝑥 + 1 = 0 ⇒ 𝑥 = −1

Odd Even Odd


𝑥𝑛 𝑥 𝑛−1 𝑥 𝑛−2 . . . 𝑥2 𝑥1 𝑥0
−1 1 0 0 0 0 1
−1 1 −1 1 −1
1 −1 1 −1 1 0
Answer 𝑥 𝑛−1

1 𝑛(𝑛+1)
Using the formula 𝑛 + (𝑛 − 1) + (𝑛 − 2) + ⋯ + 1 =
2

P a g e 28 | 88
Aziz Manva ([email protected])

Even Odd Even

If n is not even, then this does not work

Even Odd Even


𝑥𝑛 𝑥 𝑛−1 𝑥 𝑛−2 . . . 𝑥2 𝑥1 𝑥0
−1 1 0 0 0 0 1
−1 1 1 −1 1
1 −1 1 1 −1 2
Answer 𝑥 𝑛−1
Odd Even Odd

Example 4.24
𝑥2 + 1
𝑥+1

𝑥2 − 1 2 2
+ =𝑥−1+
𝑥+1 𝑥+1 𝑥+1

Example 4.25
Divide
𝑥7 + 2
𝑥+1

𝑥7 + 1 1 1
+ = (𝑥 6 − 𝑥 5 + 𝑥 4 − 𝑥 3 + ⋯ + 1) +
𝑥+1 𝑥+1 𝑥+1

𝑥7 𝑥6 𝑥5 𝑥4 𝑥3 𝑥2 𝑥1 𝑥0
−1 1 0 0 0 0 0 0 1
−1 1 −1 1 −1 1 −1
1 −1 1 −1 1 −1 1 0
Quotient 𝑥6 𝑥5 𝑥4 𝑥3 𝑥2 𝑥1 𝑥0 𝑅𝑒𝑚𝑎𝑖𝑛𝑑𝑒𝑟

Example 4.26
Divide
𝑥7 − 1
𝑥+1

𝑥7 + 1 −2 −2
+ = (𝑥 6 − 𝑥 5 + 𝑥 4 − 𝑥 3 + ⋯ + 1) +
𝑥+1 𝑥+1 𝑥+1

What is the sum of the coefficients of the quotient?

𝑥6 − 𝑥5 + 𝑥4 − 𝑥3 + 𝑥2 − 𝑥 + 1
Substitute 𝑥 = 1:
1−1+⏟
⏟ 1−1+⏟
1−1+1=0+0+0+1=1
𝑃𝑎𝑖𝑟 𝑃𝑎𝑖𝑟 𝑃𝑎𝑖𝑟

P a g e 29 | 88
Aziz Manva ([email protected])

B. Even Values of 𝒏

4.27: Division Formula-III


𝑥𝑛 − 1
= 𝑥 𝑛−1 − 𝑥 𝑛−2 + 𝑥 𝑛−3 − ⋯ − 1, 𝑛 𝑖𝑠 𝑎𝑛 𝑒𝑣𝑒𝑛 𝑛𝑎𝑡𝑢𝑟𝑎𝑙 𝑛𝑢𝑚𝑏𝑒𝑟
𝑥+1

𝑥 𝑛 + 1 = 𝑥 𝑛 + 0𝑥 𝑛−1 + 0𝑥 𝑛−2 + ⋯ + 0𝑥 + 1
IF 𝑛 is even:
➢ All the even powers have coefficient 1
➢ All the odd powers have coefficient −1

Even Odd Even


Dividend 𝑥𝑛 𝑥 𝑛−1 𝑥 𝑛−2 . . . 𝑥2 𝑥1 𝑥0
−1 1 0 0 0 0 −1
−1 1 1 −1 0
𝑄𝑢𝑜𝑡𝑖𝑒𝑛𝑡 1 −1 1 1 −1 0
Quotient Odd Even Odd Remainder

Example 4.28
Divide
𝑥6 + 1
𝑥+1

𝑥6 + 1 𝑥6 − 1 2 2
= + = (𝑥 5 − 𝑥 4 + 𝑥 3 − 𝑥 2 + 𝑥 − 1) +
𝑥+1 𝑥+1 𝑥+1 𝑥+1
C. Sum of Coefficients

Example 4.29
Find the sum of the coefficients of the quotient when we carry out the division below:
𝑥 2022 + 2022
𝑥+1

𝑥 2022 + 2022 𝑥 2022 − 1 2023 2023


= + = (𝑥 2021 − 𝑥 2020 + 𝑥 2019 − 𝑥 2018 + ⋯ + 𝑥 − 1) +
𝑥+1 𝑥+1 𝑥+1 𝑥+1

We only need to focus on the quotient here:


𝑥 2021 − 𝑥 2020 + 𝑥 2019 − 𝑥 2018 + ⋯ + 𝑥 − 1
To find sum of coefficients, substitute 𝑥 = 1, and form 1010 pairs, each of which adds up to zero:
1−1+⏟
⏟ 1 − 1 + ⋯+ ⏟ 1 − 1 = 0 + 0 + ⋯+ 0 = 0
𝑃𝑎𝑖𝑟 𝑃𝑎𝑖𝑟 𝑃𝑎𝑖𝑟

Example 4.30
𝑥𝑛 + 𝑛 𝑅1 (𝑥)
𝐹𝑜𝑟 𝑠𝑜𝑚𝑒 𝑝𝑜𝑠𝑖𝑡𝑖𝑣𝑒 𝐼𝑛𝑡𝑒𝑔𝑒𝑟 𝑛: = 𝑄1 (𝑥) +
𝑥+1 𝑥+1
Where
𝑄1 (𝑥)𝑖𝑠 𝑡ℎ𝑒 𝑞𝑢𝑜𝑡𝑖𝑒𝑛𝑡 𝑜𝑓 𝑡ℎ𝑒 𝑓𝑖𝑟𝑠𝑡 𝑑𝑖𝑣𝑖𝑠𝑖𝑜𝑛
Answer each part independently:
A. If the sum of the coefficients of 𝑄1 (𝑥) is 0. Determine whether 𝑛 is even or odd?

P a g e 30 | 88
Aziz Manva ([email protected])

B. If the sum of the coefficients of 𝑄1 (𝑥) is 1. Determine whether 𝑛 is even or odd?

𝑃𝑎𝑟𝑡 𝐴: 𝐸𝑣𝑒𝑛
𝑃𝑎𝑟𝑡 𝐵: 𝑂𝑑𝑑
D. Multiplication Formula

4.31:
𝑥 𝑛 + 1 = (𝑥 + 1)(𝑥 𝑛−1 − 𝑥 𝑛−2 + 𝑥 𝑛−3 − ⋯ + 1), 𝑛 𝑖𝑠 𝑜𝑑𝑑 𝑛𝑎𝑡𝑢𝑟𝑎𝑙 𝑛𝑢𝑚𝑏𝑒𝑟
𝑥 𝑛 − 1 = (𝑥 + 1)(𝑥 𝑛−1 − 𝑥 𝑛−2 + 𝑥 𝑛−3 − ⋯ − 1), 𝑛 𝑖𝑠 𝑎𝑛 𝑒𝑣𝑒𝑛 𝑛𝑎𝑡𝑢𝑟𝑎𝑙 𝑛𝑢𝑚𝑏𝑒𝑟

Example 4.32
(𝑥 2 + 1)(𝑥 20 − 𝑥 18 + 𝑥 16 + ⋯ + 1)

Use a change of variable. Let 𝑦 = 𝑥 2:


𝑦11 + 1 = (𝑦 + 1)(𝑦10 − 𝑦 9 + 𝑥 8 + ⋯ + 1)

Change the variable back:


𝑥 22 + 1

4.33:
𝑥 3𝑛 − 1
= 𝑥 3(𝑛−1) + 𝑥 3(𝑛−2) + 𝑥 3(𝑛−3) + ⋯ + 1
𝑥3 − 1

4.5 Applications
A. Applications

Example 4.34
𝑥 2 +2𝑥+5
What is the greatest integer value of 𝑥 such that 𝑥−3
is an integer? (MathCounts 2002 National Sprint)

𝑥 2 + 2𝑥 + 5 20
= ⏟𝑥+5 +
𝑥−3 𝐼𝑛𝑡𝑒𝑔𝑒𝑟
𝑥−3

20
Hence, we need to always be an integer, which will happen only when 𝑥 − 3 is a factor of 20.
𝑥−3
The greatest factor of 20, is 20 itself.

Hence, the greatest of 𝑥 is 23.

What are the number of values that 𝑥 can take in the above question?

For this, we need to find the number of factors of 20:


20 = 4 × 5 = 22 × 51 ⇒ 𝑁𝑜. 𝑜𝑓 𝑃𝑜𝑠𝑖𝑡𝑖𝑣𝑒 𝐹𝑎𝑐𝑡𝑜𝑟𝑠 = (2 + 1)(1 + 1) = 3 × 2 = 6

𝑇𝑜𝑡𝑎𝑙 𝐹𝑎𝑐𝑡𝑜𝑟𝑠 = 2 × 6 = 12 ⇒ 12 𝑣𝑎𝑙𝑢𝑒𝑠 𝑜𝑓 𝑥

What are the possible integer values that 𝑥 can take in the above question?

P a g e 31 | 88
Aziz Manva ([email protected])

𝑥 can take values such that 𝑥 − 3 is a factor of 20.

Factor 𝑥−3 20 10 5 4 2 1 -1 -2 -4 -5 -10 -20


Value 𝑥 23 13 8 7 5 4 2 1 -1 -2 -7 -17

Example 4.35
𝑥 2 +2𝑥+𝑐
What is the value of 𝑐 such that 𝑥−3
is an integer for integer values of 𝑥?

𝑥 2 + 2𝑥 + 𝑐 15 + 𝑐
= ⏟𝑥+5 +
𝑥−3 𝐼𝑛𝑡𝑒𝑔𝑒𝑟
𝑥−3
The first term is an integer.
Hence, the second term has to be an integer for all integer values of 𝑥.
15+𝑐
The only way this will happen is if the numerator of the expression 𝑥−3
is zero.
15 + 𝑐 = 0 ⇒ 𝑐 = −15

4.6 Remainders
A. Basics

Example 4.36

4.7 Summary
4.37: Summary of Division Formulas
𝑥𝑛 − 1
= 𝑥 𝑛−1 + 𝑥 𝑛−2 + 𝑥 𝑛−3 + ⋯ + 1
𝑥−1
𝑥𝑛 + 1
= 𝑥 𝑛−1 − 𝑥 𝑛−2 + 𝑥 𝑛−3 − ⋯ + 1, 𝑛 𝑖𝑠 𝑜𝑑𝑑 𝑛𝑎𝑡𝑢𝑟𝑎𝑙 𝑛𝑢𝑚𝑏𝑒𝑟
𝑥+1
𝑛
𝑥 +1 2
= 𝑥 𝑛−1 − 𝑥 𝑛−2 + 𝑥 𝑛−3 + ⋯ − 1 + , 𝑛 𝑖𝑠 𝑒𝑣𝑒𝑛 𝑛𝑎𝑡𝑢𝑟𝑎𝑙 𝑛𝑢𝑚𝑏𝑒𝑟
𝑥+1 𝑥+1

P a g e 32 | 88
Aziz Manva ([email protected])

PART II: REMAINDER AND FACTOR THEOREM


5. LINEAR DIVISORS
5.1 Basics
A. Statement

5.1: Division Property


𝐷𝑖𝑣𝑖𝑑𝑒𝑛𝑑 𝑅𝑒𝑚𝑎𝑖𝑛𝑑𝑒𝑟
= 𝑄𝑢𝑜𝑡𝑖𝑒𝑛𝑡 +
𝐷𝑖𝑣𝑖𝑠𝑜𝑟 𝐷𝑖𝑣𝑖𝑠𝑜𝑟

𝐴𝑙𝑡𝑒𝑟𝑛𝑎𝑡𝑒 𝐹𝑜𝑟𝑚: 𝐷𝑖𝑣𝑖𝑑𝑒𝑛𝑑 = 𝐷𝑖𝑣𝑖𝑠𝑜𝑟 × 𝑄𝑢𝑜𝑡𝑖𝑒𝑛𝑡 + 𝑅𝑒𝑚𝑎𝑖𝑛𝑑𝑒𝑟

Example 5.2
43
⏟ ÷ ⏟
7
𝐷𝑖𝑣𝑖𝑑𝑒𝑛𝑑 𝐷𝑖𝑣𝑖𝑠𝑜𝑟

43 1
=6+
7 7
Multiply both sides by 7:
43
⏟ = ⏟
7 × ⏟
6 + ⏟
1
𝐷𝑖𝑣𝑖𝑑𝑒𝑛𝑑 𝐷𝑖𝑣𝑖𝑠𝑜𝑟 𝑄𝑢𝑜𝑡𝑖𝑒𝑛𝑡 𝑅𝑒𝑚𝑎𝑖𝑛𝑑𝑒𝑟

Example 5.3

𝑥 2 + 7𝑥 + 12 (𝑥 + 3)(𝑥 + 4)
= =𝑥+3
𝑥+4 𝑥+4
𝑥 2 + 7𝑥 + 14 (𝑥 + 3)(𝑥 + 4) + 2 2
= =𝑥+3+
𝑥+4 𝑥+4 𝑥+4
Multiply both sides by 𝑥 + 4:

⏟2 + 7𝑥 + 14 = ⏟
𝑥 (𝑥 + 3) ⏟
(𝑥 + 4) + ⏟
2
𝐷𝑖𝑣𝑖𝑑𝑒𝑛𝑑 𝐷𝑖𝑣𝑖𝑠𝑜𝑟 𝑄𝑢𝑜𝑡𝑖𝑒𝑛𝑡 𝑅𝑒𝑚𝑎𝑖𝑛𝑑𝑒𝑟

5.4: Remainder Theorem


The remainder when a polynomial function 𝑓(𝑥) is divided by a linear expression 𝑥 − 𝑎 is the same as the value
of the function at the root of the expression.
𝑓(𝑥) 𝑟
= 𝑃(𝑥)
⏟ + ⇒ 𝑟 = 𝑅𝑒𝑚𝑎𝑖𝑛𝑑𝑒𝑟 = 𝑓(𝑎)
𝑥 − 𝑎 𝑄𝑢𝑜𝑡𝑖𝑒𝑛𝑡 𝑥 − 𝑎

Multiply both sides of the equality by 𝑥 − 𝑎 to get:


𝑓(𝑥) = (𝑥
⏟ − 𝑎) 𝑃(𝑥)
⏟ + ⏟
𝑟
𝐷𝑖𝑣𝑖𝑠𝑜𝑟 𝑄𝑢𝑜𝑡𝑖𝑒𝑛𝑡 𝑅𝑒𝑚𝑎𝑖𝑛𝑑𝑒𝑟
Substitute 𝑥 = 𝑎:
𝑓(𝑎) = (𝑎 − 𝑎)𝑃(𝑥) + 𝑟 = (0)𝑃(𝑥) + 𝑟 = 0 + 𝑟 = 𝑟
B. Calculating Remainder

Example 5.5

P a g e 33 | 88
Aziz Manva ([email protected])

Find the remainder when


A. 𝑃(𝑥) = 3𝑥 3 + 2𝑥 2 + 4𝑥 − 2 is divided by 𝑥 − 1.
B. 𝑃(𝑥) = 2𝑥 3 + 6𝑥 2 − 11𝑥 + 4 is divided by 𝑥 + 1.
C.

𝑥 − 1 = 1 ⇒ 𝑥 = −1 ⇒ 𝑃(1) = 3 + 2 + 4 − 2 = 7
𝑥 + 1 = 0 ⇒ 𝑥 = −1 ⇒ 𝑃(−1) = −2 + 6 + 11 + 4 = 19

Example 5.6
When 𝑥 13 + 1 is divided by 𝑥 − 1, the remainder is: (AHSME 1950/20)

Find the root of 𝑥 − 1:


𝑥−1=0⇒𝑥 =1

Evaluate the polynomial at the root:


𝑥 = 1 ⇒ 𝑥 13 + 1 = 113 + 1 = 1 + 1 = 2

Example 5.7
Find the remainder when:
A. 𝑥 2020 is divided by 𝑥 + 1
B. 𝑥 35 is divided by 𝑥 − 1
C. 𝑥 1949 is divided by 𝑥 + 𝑐

𝑥 + 1 = 0 ⇒ 𝑥 = −1 ⇒ 𝑥 2020 = (−1)2020 = 1
𝑥 − 1 = 0 ⇒ 𝑥 = 1 ⇒ 𝑥 35 = 135 = 1
𝑥 + 𝑐 = 0 ⇒ 𝑥 = −𝑐 ⇒ 𝑥 1949 = (−𝑐)1949 = −𝑐1949

Example 5.8
Find the remainder when:
A. 𝑃1 (𝑥) = 𝑥 2020 + 𝑥 2019 + ⋯ + 𝑥 + 1 is divided by 𝑥 − 1
B. 𝑃1 (𝑥) = 𝑥 2020 + 𝑥 2019 + ⋯ + 𝑥 + 1 is divided by 𝑥 + 1

𝑥 − 1 = 0 ⇒ 𝑥 = 1 ⇒ 𝑃1 (1) = ⏟
1 + 1 + ⋯ + 1 = 2021
2021 𝑡𝑖𝑚𝑒𝑠
𝑥 − 1 = 0 ⇒ 𝑥 = −1 ⇒ 𝑃1 (1) = ⏟
−1 + 1 − 1 + 1 + 1 = 1
2020
𝑃𝑎𝑖𝑟𝑠
2

C. Factor Theorem
The factor theorem is a special case of the remainder theorem.

Example 5.9
A. Find the remainder when 𝑦 = 𝑓(𝑥) = 4𝑥 2 − 8𝑥 + 3 is divided by 2𝑥 − 1.
B. Hence, explain why 2𝑥 − 1 is a factor of 4𝑥 2 − 8𝑥 + 3.

Find the root of the divisor:


1
2𝑥 − 1 = 0 ⇒ 𝑥 =
2
Evaluate the function at the root:

P a g e 34 | 88
Aziz Manva ([email protected])

1 1 2 1
𝑓( ) = 4( ) − 8( ) + 3 = 1 − 4 + 3 = 0
2 2 2
The Remainder is zero.

5.10: Factor Theorem


If P(𝒂) = 𝟎 for a polynomial 𝑷(𝒙), then 𝒙 − 𝒂 is a factor of 𝑷(𝒙).

𝑓(𝑥) = 𝑄(𝑥)(𝑥 − 𝑎) + 𝑅
Let 𝑓(𝑎) = 0:
𝑓(𝑎) = 0 = 𝑄(0)(𝑎 − 𝑎) + 𝑅
0=𝑅
Which tells us the Remainder when dividing 𝑓(𝑥) by (𝑥 − 𝑎) is zero.

D. Checking for Factors


We can use the factor theorem to check if a number is a factor of an expression.

Example 5.11
Check if 𝑥 + 2 is a factor of 𝑓(𝑥) = 𝑥 3 + 2𝑥 2 − 7𝑥 + 4

𝑥 + 2 = 0 ⇒ 𝑥 = −2
𝑓(−2) = (−2)3 + 2(−2)2 − 7(−2) + 4 = ⏟
−8 + 8 ⏟
+14 + 4 ≠ 0 ⇒ 𝑥 + 2 𝑖𝑠 𝑛𝑜𝑡 𝑎 𝑓𝑎𝑐𝑡𝑜𝑟
𝑍𝑒𝑟𝑜 +𝑣𝑒

E. Units Digit

Example 5.12
Find the units digits of the remainder when 𝑥 1949 is divided by 𝑥 + 2

𝑥 + 2 = 0 ⇒ 𝑥 = −2 ⇒ 𝑥 1949 = (−2)1949
1949 ≡ 49 ≡ 1 (𝑚𝑜𝑑 4)

𝑛(𝑚𝑜𝑑 4) 2𝑛 𝑥 2𝑛 𝑥 Last Digit


1 21 2 25 32 2
2 22 4 26 64 4
3 23 8 27 128 8
0 24 16 28 256 6

𝐿𝑎𝑠𝑡 𝐷𝑖𝑔𝑖𝑡 = 2

5.13: Cyclicity of Units Digit

Last Last Digit of Cyclicity


Digit of Square Cube 4th Power Pattern
No.
0 0 0 0 0 0 0 0 0 1
1 1 1 1 1 1 1 1 1 1
2 4 8 16 2 4 8 6 2, 4, 8, 6 4
3 9 27 81 3 9 7 1 3, 9, 7, 1 4

P a g e 35 | 88
Aziz Manva ([email protected])

4 16 64 256 4 6 4 6 4, 6 2
5 25 125 625 5 5 5 5 5 1
6 36 216 1296 6 6 6 6 6 1
7 49 343 2401 7 9 3 1 7, 9, 3, 1 4
8 64 512 4096 8 4 2 6 8, 4, 2, 6 4
9 81 729 6561 9 1 9 1 9, 1 2

Example 5.14
Find the units digits of the remainder when 𝑥 𝑛 , 𝑛 ∈ ℕ is divided by 𝑥 − 6, what is the units digit of the
remainder?

𝑥 − 6 = 0 ⇒ 𝑥 𝑛 = 6𝑛

6, 36, 216, 1296 ⇒ 𝐿𝑎𝑠𝑡 𝐷𝑖𝑔𝑖𝑡 𝑖𝑠 𝑎𝑙𝑤𝑎𝑦𝑠 6


𝐿𝑎𝑠𝑡 𝐷𝑖𝑔𝑖𝑡 𝑜𝑓 6𝑛 = 6

Example 5.15
When 𝑥 𝑛 , 𝑛 ∈ ℕ is divided by 𝑥 − 𝑎, where 𝑎 is a single digit integer, the units digits of the remainder is 6. What
can be the possible values of 𝑎?

𝑥 − 𝑎 = 0 ⇒ 𝑥 = 𝑎 ⇒ 𝑥 𝑛 = 𝑎𝑛
𝐿𝑎𝑠𝑡 𝐷𝑖𝑔𝑖𝑡 𝑜𝑓 𝑎𝑛 = 6 ⇒ 𝑎 ∈ {2,4,6,8}
F. Division Properties

5.16: Property-I
𝑥 𝑛 − 1 is divisible by 𝑥 − 1

𝑥−1=0⇒𝑥 =1
𝑛
Evaluate 𝑥 − 1 at 𝑥 = 1:
𝑥 = 1 ⇒ 𝑥 𝑛 − 1 = 1𝑛 − 1 = 0

5.17: Property-II
𝑥 𝑛 + 1 is divisible by 𝑥 + 1 for odd values of 𝑛.
𝑥 𝑛 + 1 has remainder two when divided by 𝑥 + 1 for even values of n.

𝑥 + 1 = 0 ⇒ 𝑥 = −1
𝑥 𝑛 + 1 = (−1)𝑛 + 1

𝑛 𝑖𝑠 𝑒𝑣𝑒𝑛 ⇒ (−1)𝑛 + 1 = 1 + 1 = 2
𝑛 𝑖𝑠 𝑜𝑑𝑑 ⇒ (−1)𝑛 + 1 = −1 + 1 = 0

5.18: Property-III
𝑥 𝑛 − 1 is divisible by 𝑥 + 1 for even values of 𝑛.
𝑥 𝑛 − 1 has remainder −2 when divided by 𝑥 + 1 for odd values of n.

𝑥 + 1 = 0 ⇒ 𝑥 = −1
𝑥 𝑛 − 1 = (−1)𝑛 − 1

P a g e 36 | 88
Aziz Manva ([email protected])

𝑛 𝑖𝑠 𝑒𝑣𝑒𝑛 ⇒ (−1)𝑛 + 1 = 1 − 1 = 0
𝑛 𝑖𝑠 𝑜𝑑𝑑 ⇒ (−1)𝑛 + 1 = −1 − 1 = −2

5.2 Synthetic Substitution


A. Introduction
We look at an interesting application of the remainder theorem that let us use synthetic division in a different
way.
B. Evaluating a Function (Synthetic Substitution)
By the remainder theorem, we know that
➢ if 𝑃(𝑎) = 𝑅, then 𝑅 is the remainder when 𝑃(𝑥) is divided by 𝑥 − 𝑎.
Also, we know that
➢ synthetic division gives us the remainder when 𝑃(𝑥) is divided by 𝑥 − 𝑎.

We can combine the above two to evaluate a function at a point.

Example 5.19
Given that 𝑓(𝑥) = 𝑥 2 + 5𝑥 + 8, evaluate 𝑓(−3) by using synthetic division.

𝑓(−3) = (−3)2 + 5(−3) + 8 = 9 − 15 + 8 = 2 1 5 8


−3 −3 −6
𝑥 2 + 5𝑥 + 8 2 1 2 2
=𝑥+2+
𝑥+3 𝑥+3

The remainder when 𝑓(𝑥) is divided by 𝑥 + 3 is the same as 𝑓(−3).


We can also get the remainder using polynomial long division.
And synthetic division is just a shortcut to perform polynomial long division.

Example 5.20
Evaluate 𝑓(𝑥) = −3𝑥 2 + 2𝑥 − 4 at 𝑥 = −3.

𝑥2 𝑥 𝐶𝑜𝑛𝑠𝑡𝑎𝑛𝑡
−3 −3 2 −4
𝑓(−3) = −3(9) + 2(−3) − 4 = −27 − 6 − 4 = −37 0 9 −33
−3 11 −37

Example 5.21
Evaluate 𝑓(𝑥) = 4𝑥 3 − 5𝑥 2 + 7 at 𝑥 = 2.
𝑥3 𝑥2 𝑥 𝐶𝑜𝑛𝑠𝑡𝑎𝑛𝑡
𝑓(2) = 4(8) − 5(4) + 7 = 32 − 20 + 7 = 19
2 4 −5 0 7
8 6 12
4 3 6 19
5.3 Finding Coefficients
A. Linear Equations

Example 5.22
Find the value of 𝑎 if:
A. 𝑥 2 + 𝑎𝑥 + 6 is divisible by 𝑥 + 2

P a g e 37 | 88
Aziz Manva ([email protected])

B. 𝑥 2 + 𝑎𝑥 + 6 is divisible by 𝑥 − 2
C. 𝑥 3 + 𝑎𝑥 2 − 5𝑥 − 6 is divisible by 𝑥 + 3

Part A
𝑥 + 2 = 0 ⇒ 𝑥 = −2
(−2)2 + 𝑎(−2) + 6 = 0 ⇒ 10 = 2𝑎 ⇒ 𝑎 = 5

Alternate Method: Undetermined Coefficients


𝑥 2 + 𝑎𝑥 + 6 = (𝑥 + 2)(𝑥 + 𝑐) = 𝑥 2 + 2𝑥 + 𝑐𝑥 + 2𝑐
2𝑐 = 6 ⇒ 𝑐 = 3
Part B
𝑥−2=0⇒𝑥 =2
(2)2 + 𝑎(2) + 6 = 0 ⇒ 2𝑎 = −10 ⇒ 𝑎 = −5
Part C
𝑥 + 3 = 0 ⇒ 𝑥 = −3

(−3)3 + 𝑎(−3)2 − 5(−3) − 6 = 0


−27 + 9𝑎 + 15 − 6 = 0 ⇒ 9𝑎 − 18 = 0 ⇒ 9𝑎 = 18 ⇒ 𝑎 = 2

Example 5.23
When 𝑓(𝑥) = 𝑥 3 + 𝑘𝑥 2 − 7𝑥 + 3 is divided by 𝑥 + 1, the remainder is seven times the remainder that is found
when the same expression is divided by 𝑥 + 2. Find the value of 𝑘.

𝑥 + 1 = 0 ⇒ 𝑥 = −1
𝑓(−1) = (−1)3 + 𝑘(−1)2 − 7(−1) + 3 = 𝑘 + 9
𝑓(−2) = (−2)3 + 𝑘(−2)2 − 7(−2) + 3 = −8 + 4𝑘 + 14 + 3 = 4𝑘 + 9

𝑓(−1) = 7𝑓(−2) ⇒ 𝑘 + 9 = 7(4𝑘 + 9) ⇒ 𝑘 + 9 = 28𝑘 + 63 ⇒ −54 = 27𝑘 ⇒ 𝑘 = −2

Example 5.24
𝑓(𝑥) = 2𝑥 3 − 7𝑥 2 + 7𝑎𝑥 + 16 is divisible by 𝑥 − 𝑎. Find the remainder when 𝑓(𝑥) is divided by 2𝑥 + 1.

2𝑎3 − 7𝑎2 + 7𝑎2 + 16 = 2𝑎3 = −16 ⇒ 𝑎3 = −8 ⇒ 𝑎 = −2

𝑓(𝑥) = 2𝑥 3 − 7𝑥 2 − 14𝑥 + 16
1
𝑓 (− ) = 21
2

Example 5.25
𝑥 − 1 is a factor of 𝑓(𝑥) = 𝑥 3 − 6𝑥 2 + 𝑎𝑥 + 𝑏. Show that the remainder when 𝑓(𝑥) is divided by 𝑥 − 3 is twice
the remainder when 𝑓(𝑥) is divided by 𝑥 − 2.

𝑥 − 1 = 0 ⇒ 𝑥 = 1 ⇒ 𝑓(1) = 0
𝑓(1) = 13 − 6 ∙ 12 + 𝑎 ∙ 1 + 𝑏 = 0 ⇒ −5 + 𝑎 + 𝑏 = 0 ⇒ 𝑏 = 5 − 𝑎

𝑓(𝑥) = 𝑥 3 − 6𝑥 2 + 𝑎𝑥 + 5 − 𝑎

𝑥−2=0⇒𝑥 =2
3 2
𝑓(2) = 2 − 6 ∙ 2 + 𝑎 ∙ 2 + 5 − 𝑎 = 8 − 24 + 2𝑎 + 5 − 𝑎 = 𝑎 − 11

P a g e 38 | 88
Aziz Manva ([email protected])

𝑥−3=0⇒𝑥 =3
𝑓(3) = 33 − 6 ∙ 32 + 𝑎 ∙ 3 + 5 − 𝑎 = 27 − 54 + 3𝑎 + 𝑏 = 2𝑎 − 22 = 2(𝑎 − 11)

B. No Solutions

Example 5.26

C. Infinite Solutions

Example 5.27
When 𝑓(𝑥) = 3𝑥 5 − 𝑎𝑥 + 𝑏 is divided by 𝑥 − 1 and 𝑥 + 1, the remainders are equal. Find the values that 𝑎 and 𝑏
can take.

𝑥−1=0⇒𝑥 =1
𝑥 + 1 = 0 ⇒ 𝑥 = −1

𝑓(1) = 𝑓(−1) ⇒ 3 − 𝑎 + 𝑏 = −3 + 𝑎 + 𝑏 ⇒ 2𝑎 = 6 ⇒ 𝑎 = 3, 𝑏 ∈ ℝ
D. Simultaneous Equations

Example 5.28
When the polynomial 3𝑥 3 + 𝑎𝑥 + 𝑏 is divided by 𝑥 − 2, the remainder is 2, and when divided by 𝑥 + 1, it is 5.
Find the value of 𝑎, and the value of 𝑏.

𝑥 − 2 = 0 ⇒ 𝑥 = 2 ⇒ 𝑓(2) = 24 + 2𝑎 + 𝑏 = 2 ⇒ ⏟
2𝑎 + 𝑏 = −22
𝑬𝒒𝒖𝒂𝒕𝒊𝒐𝒏 𝑰
𝑥 + 1 = 0 ⇒ 𝑥 = −1 ⇒ 𝑓(−1) = −3 − 𝑎 + 𝑏 = 5 ⇒ ⏟
−𝑎 + 𝑏 = 8
𝑬𝒒𝒖𝒂𝒕𝒊𝒐𝒏 𝑰𝑰
Subtract Equation II from Equation I:
3𝑎 = −30 ⇒ 𝑎 = −10

Substitute 𝑎 = −10 in Equation II:


10 + 𝑏 = 8 ⇒ 𝑏 = −2

Example 5.29
The cubic polynomial 3𝑥 3 + 𝑝𝑥 2 + 𝑞𝑥 − 2 has a factor (𝑥 + 2), and leaves a remainder 4 when divided by 𝑥 + 1.
Find the value of 𝑝 and 𝑞.

𝑥 + 2 = 0 ⇒ 𝑥 = −2 ⇒ 𝑓(−2) = 0 ⇒ −24 + 4𝑝 − 2𝑞 − 2 = 0 ⇒ ⏟
2𝑝 − 𝑞 = 13
𝑬𝒒𝒖𝒂𝒕𝒊𝒐𝒏 𝑰
𝑥 + 1 = 0 ⇒ 𝑥 = −1 ⇒ 𝑓(−1) = 4 ⇒ −3 + 𝑝 − 𝑞 − 2 = 4 ⇒ 𝑝
⏟− 𝑞 = 9
𝑬𝒒𝒖𝒂𝒕𝒊𝒐𝒏 𝑰𝑰
Subtract Equation II from Equation I:
𝑝 = 4 ⇒ 𝑞 = −5

Example 5.30

P a g e 39 | 88
Aziz Manva ([email protected])

𝑓(𝑥) 𝑓(𝑥)
2𝑥 − 1 is a factor of 𝑓(𝑥) = 𝑎𝑥 3 + 4𝑥 2 + 𝑏𝑥 − 2. If 𝑥−2 has remainder twice of the remainder of 𝑥+1 , find the
value of 𝑎 and 𝑏.

1 𝑎 𝑏
𝑓( ) = +1+ − 2 = 0 ⇒ ⏟
𝑎 + 4𝑏 = 8
2 8 2 𝑬𝒒𝒖𝒂𝒕𝒊𝒐𝒏 𝑰
𝑓(2) = 2𝑓(−1) ⇒ 8𝑎 + 16 + 2𝑏 − 2 = 2(−𝑎 + 4 − 𝑏 − 2) ⇒ ⏟
10𝑎 + 4𝑏 = −10
𝑬𝒒𝒖𝒂𝒕𝒊𝒐𝒏 𝑰𝑰

𝑎 = −2, 𝑏 = 5/2

E. Quadratic Equations

Example 5.31
The same remainder is found when 𝑓(𝑥) = 2𝑥 3 + 𝑘𝑥 2 + 6𝑥 + 32 and 𝑔(𝑥) = 𝑥 4 − 6𝑥 2 − 𝑘 2 𝑥 + 9 are divided by
𝑥 + 1. Find the possible values of 𝑘.

𝑥 + 1 = 0 ⇒ 𝑥 = −1
𝑓(−1) = 𝑔(−1)
−2 + 𝑘 − 6 + 32 = 1 − 6 + 𝑘 2 + 9

Collate all terms on side to get a quadratic and solve:


𝑘 2 − 𝑘 − 20 = 0 ⇒ (𝑘 + 5)(𝑘 − 4) = 0 ⇒ 𝑘 ∈ {−4,5}

F. Minimum Value of A Quadratic

Example 5.32
When 𝑥 2 + 4𝑥 − 𝑏 is divided by 𝑥 − 𝑎, the remainder is 𝑎. Find the smallest possible value of 𝑏.

𝑥−𝑎 =0⇒𝑥 =𝑎

𝑓(𝑎) = 𝑎2 + 4𝑎 − 𝑏 = 𝑎
2 2
9 9 3 2 9
𝑏 = 𝑎 + 3𝑎 = 𝑎 + 3𝑎 + − = (𝑎 + ) −
4 4 2 4

Example 5.33
When 𝑥 2 + 4𝑥 − 𝑏 is divided by 𝑥 − 𝑎, the remainder is 2. Find the smallest possible value of 𝑏.

𝑥−𝑎 =0⇒𝑥 =𝑎

𝑓(𝑎) = 𝑎2 + 4𝑎 − 𝑏 = 2
𝑏 = 𝑎2 + 4𝑎 − 2 = 𝑎2 + 4𝑎 + 4 − 4 − 2 = (𝑎 + 2)2 − 6

G. Quadratic Inequality

Example 5.34
The remainder when 𝑓(𝑥) = 𝑥 3 + 𝑝𝑥 2 + 𝑝2 𝑥 + 21 is divided by 𝑥 + 3 is positive. What can be the value of 𝑝?

P a g e 40 | 88
Aziz Manva ([email protected])

𝑓(−3) = −27 + 9𝑝 − 3𝑝2 + 21 = 9𝑝 − 3𝑝2 − 6

9𝑝 − 3𝑝2 − 6 > 0 ⇒ 𝑝2 − 3𝑝 + 2 < 0 ⇒ (𝑝 − 1)(𝑝 − 2) < 0 ⇒ 𝑝 ∈ (1,2)


H. More Challenging Example

Example 5.35
The polynomial 𝑓(𝑥) = 𝑥 4 + 𝑝𝑥 3 + 𝑞𝑥 2 + 𝑟𝑥 + 6 is exactly divisible by each of (𝑥 − 1), (𝑥 − 2) and (𝑥 − 3). Find
the values of 𝑝, 𝑞 and 𝑟.

𝑥 − 1 = 0 ⇒ 𝑥 = 1 ⇒ 𝑓(1) = 1 + 𝑝 + 𝑞 + 𝑟 + 6 = 0 ⇒ ⏟
𝑝 + 𝑞 + 𝑟 = −7
𝑬𝒒𝒖𝒂𝒕𝒊𝒐𝒏 𝑰
𝑥 − 2 = 0 ⇒ 𝑥 = 2 ⇒ 𝑓(2) = 16 + 8𝑝 + 4𝑞 + 2𝑟 + 6 = 0 ⇒ ⏟
4𝑝 + 2𝑞 + 𝑟 = −11
𝑬𝒒𝒖𝒂𝒕𝒊𝒐𝒏 𝑰𝑰
𝑥 − 3 = 0 ⇒ 𝑥 = 3 ⇒ 𝑓(3) = 81 + 27𝑝 + 9𝑞 + 3𝑟 + 6 = 0 ⇒ 9𝑝
⏟ + 3𝑞 + 𝑟 = −29
𝑬𝒒𝒖𝒂𝒕𝒊𝒐𝒏 𝑰𝑰𝑰

Subtract Equation I from Equation II:


3𝑝 + 𝑞 = −4

𝑬𝒒𝒖𝒂𝒕𝒊𝒐𝒏 𝑰𝑽
Subtract Equation I from Equation III:
8𝑝 + 2𝑞 = −22 ⇒ ⏟
4𝑝 + 𝑞 = −11
𝑬𝒒𝒖𝒂𝒕𝒊𝒐𝒏 𝑽

Subtract Equation IV from Equation V:


𝑝 = −7
Substitute 𝑝 = −7 in Equation IV:
3𝑝 + 𝑞 = −4 ⇒ −21 + 𝑞 = −4 ⇒ 𝑞 = 17
Substitute 𝑝 = −7, 𝑞 = 17 in Equation I:
𝑝 + 𝑞 + 𝑟 = −7 ⇒ −7 + 17 + 𝑟 = −7 ⇒ 𝑟 = −17

(𝑝, 𝑞, 𝑟) = (−7,17, −17)

Alternate Method
𝑥 4 + 𝑝𝑥 3 + 𝑞𝑥 2 + 𝑟𝑥 + 6 = (𝑥 − 𝑎)(𝑥 − 1)(𝑥 − 2)(𝑥 − 3)

6 = (−𝑎)(−1)(−2)(−3) ⇒ 6 = 6𝑎 ⇒ 𝑎 = 1

𝑓(𝑥) = (𝑥 − 1)(𝑥 − 1)(𝑥 − 2)(𝑥 − 3) =

P a g e 41 | 88
Aziz Manva ([email protected])

6. HIGHER DEGREE DIVISORS


6.1 Division
A. Exactly Divisible

6.1: Applying Factor Theorem to Quadratic Expressions


If 𝑄(𝑥) = (𝑥 − 𝛼)(𝑥 − 𝛽) divides a polynomial 𝑃(𝑥) then
𝑃(𝛼) = 0
𝑃(𝛽) = 0

This property tells us that if a quadratic expression divides a polynomial, then each of its factors also divides the
polynomial.

Example 6.2
Find 𝑎𝑏 if 3𝑥 6 + 𝑎𝑥 5 + 𝑏𝑥 4 − 𝑥 3 + 12𝑥 2 − 𝑥 − 7 is divisible by 𝑥 2 − 1.

𝑥 − 1 = 0 ⇒ 𝑥 = 1 ⇒ 3 + 𝑎 + 𝑏 − 1 + 12 − 1 − 7 = 0 ⇒ ⏟
𝑎+𝑏+6 = 0
𝑬𝒒𝒖𝒂𝒕𝒊𝒐𝒏 𝑰
𝑥 + 1 = 0 ⇒ 𝑥 = −1 ⇒ 3 − 𝑎 + 𝑏 + 1 + 12 + 1 − 7 = 0 ⇒ ⏟
−𝑎 + 𝑏 + 10 = 0
𝑬𝒒𝒖𝒂𝒕𝒊𝒐𝒏 𝑰𝑰
Add Equation I and II:
2𝑏 + 16 = 0 ⇒ 𝑏 = −8
Substitute 𝑏 = −8 in Equation I:
𝑎 − 8 + 6 = 0 ⇒ 𝑎 = 2 ⇒ 𝑎𝑏 = (2)(−8) = −16

Example 6.3
𝑓(𝑥) = 𝐴𝑥 3 + 𝐵𝑥 2 + 𝑥 + 6 is divisible by (𝑥 2 − 𝑥 − 2). Find the value of 𝐴 and 𝐵.

𝑥 2 − 𝑥 − 2 = 0 ⇒ (𝑥 + 1)(𝑥 − 2) = 0 ⇒ 𝑥 ∈ {−1,2}

𝑓(−1) = −A + B − 1 + 6 = 0 ⇒ ⏟
A−B =5
𝑬𝒒𝒖𝒂𝒕𝒊𝒐𝒏 𝑰
𝑓(2) = 8A + 4B + 2 + 6 = 0 ⇒ ⏟
2A + B = −2
𝑬𝒒𝒖𝒂𝒕𝒊𝒐𝒏 𝑰𝑰

Add Equations I and II:


3𝐴 = 3 ⇒ 𝐴 = 1 ⇒ 𝐵 = −4

Example 6.4 (Calculator Allowed)


If 𝑥 4 + 𝑎𝑥 3 + 𝑏𝑥 2 − 5𝑥 + 14 is divisible by 𝑥 2 + 8𝑥 + 7, then find the value of 𝑎 and 𝑏.

𝑥 2 + 8𝑥 + 7 = (𝑥 + 7)(𝑥 + 1) ⇒ 𝑥 ∈ {−1, −7}

(−7)4 + 𝑎(−7)3 + 𝑏(−7)2 − 5(−7) + 14 = 0


2401 − 343𝑎 + 49𝑏 + 35 + 14 = 0
−343𝑎 + 49𝑏 = −2450

𝑬𝒒𝒖𝒂𝒕𝒊𝒐𝒏 𝑰

1 − 𝑎 + 𝑏 + 5 + 14 = 0 ⇒ −𝑎 + 𝑏 = −20

P a g e 42 | 88
Aziz Manva ([email protected])

Multiply by −49:
49𝑎 − 49𝑏 = 980

𝑬𝒒𝒖𝒂𝒕𝒊𝒐𝒏 𝑰𝑰

Add Equations I and II:


1470
−294𝑎 = −1470 ⇒ 𝑎 = − =5
−294

−𝑎 + 𝑏 = −20 ⇒ −5 + 𝑏 = −20 ⇒ 𝑏 = −15


B. Multi-Step Division

Example 6.5
1
If 𝑞1 (𝑥) and 𝑟1 are the quotient and remainder, respectively, when the polynomial 𝑥 8 is divided by 𝑥 + , and if
2
1
𝑞2 (𝑥) and 𝑟2 are the quotient and remainder, respectively, when 𝑞1 (𝑥) is divided by 𝑥 + 2, then 𝑟2 equals:
(AHSME 1979/25)

1
Divide 𝑥 8 by 𝑥 + using synthetic division:
2
1 1
𝑥+ =0⇒𝑥=−
2 2

𝑥8 𝑥7 𝑥6 𝑥5 𝑥4 𝑥3 𝑥2 𝑥1 𝑥0
1 1 0 0 0 0 0 0 0 0

2
1 1 1 1 1 1 1 1
− − − −
2 4 8 16 32 64 128 256
1 1 1 1 1 1 1 1 1
− − − −
2 4 8 16 32 64 128 256
Quotient 𝑥7 𝑥6 𝑥5 𝑥4 𝑥3 𝑥2 𝑥1 𝑥0 Remainder

Note that because of the zeroes the values in the middle row exactly match the values in the bottom row.
1 1 1 1 1 1 1
𝑞1 (𝑥) = 𝑥 7 − 𝑥 6 + 𝑥 5 − 𝑥 4 + 𝑥 3 − 𝑥 2 + 𝑥 −
2 4 8 16 32 64 128

1 1
To find the remainder when the 𝑞1 is divided by 𝑥 + 2, we use the remainder theorem and evaluate 𝑞1 (− 2):
1 1 7 1 1 6 1 1 5 1 1 4 1 1 3 1 1 2 1 1 1
𝑞1 (− ) = (− ) − (− ) + (− ) − (− ) + (− ) − (− ) + (− ) −
2 2 2 2 4 2 8 2 16 2 32 2 64 2 128
1
Each of the terms in the above expression evaluates to − 27:
1 1 1 1 1 1 1 1 1 1 1 1 1 1
= − 7 − ( 6 ) − 2 ( 5) − 3 ( 4) − 4 ( 3 ) − 5 ( 2) − 6 ( 1 ) − 7
2 2 2 2 2 2 2 2 2 2 2 2 2 2
1 1 1
= − 7 − 7 − ⋯− 7
⏟2 2 2
8 𝑇𝑖𝑚𝑒𝑠
8 23 1 1
=− 7=− 7=− 4=−
2 2 2 16

𝑥7 𝑥6 𝑥5 𝑥4 𝑥3 𝑥2 𝑥1 𝑥0
1 1 1 1 1 1 1 1 1
− − − − −
2 2 4 8 16 32 64 128
P a g e 43 | 88
Aziz Manva ([email protected])

1 1 3 4 5 6 7
− − − −
2 2 8 16 32 64 128
1 −1 3 1 5 6 7 8 1
− − − =−
4 2 16 32 64 128 16
Quotient Remainder

Example 6.6
Find the remainder when 𝑓(𝑥) = 𝑥 𝑛 − 𝑛𝑥 + 𝑛 − 1, 𝑛 > 3 is divided by 𝑥 3 − 3𝑥 2 + 3𝑥 − 1

Step I
Factor the divisor
𝑥 3 − 3𝑥 2 + 3𝑥 − 1 = (𝑥 − 1)3
Step II
We can check the remainder when 𝑥 𝑛 − 𝑛𝑥 + 𝑛 − 1 is divided by 𝑥 − 1:
𝑥 − 1 = 0 ⇒ 𝑥 = 1 ⇒ 𝑓(1) = 1 − 𝑛 + 𝑛 − 1 = 0 ⇒ 𝐷𝑖𝑣𝑖𝑠𝑖𝑏𝑙𝑒

Now that we know it is divisible, carry out the division by splitting the terms:
𝑥 𝑛 − 𝑛𝑥 + 𝑛 − 1 𝑥𝑛 − 1 𝑛(𝑥 − 1) 𝑥 𝑛−1 + 𝑥 𝑛−2 + ⋯ + 1 𝑛
= − = −
(𝑥 − 1)3 (𝑥 − 1)3 (𝑥 − 1)3 (𝑥 − 1)2 (𝑥 − 1)2

Step III
Divide by 𝑥 − 1 one more time by using synthetic division:

𝑥 𝑛−1 𝑥 𝑛−2 𝑥 𝑛−3 . . . 𝑥1 𝑥0


1 1 1 . . . 1 1−𝑛
1 1 2 𝑛−2 𝑛−1
1 2 3 . . . 𝑛−1 0
𝑥 𝑛−2 𝑥 𝑛−3 𝑥 𝑛−4 . . . 𝑥0 𝑅𝑒𝑚𝑎𝑖𝑛𝑑𝑒𝑟

𝑄𝑢𝑜𝑡𝑖𝑒𝑛𝑡 = 𝑥 𝑛−2 + 2𝑥 𝑛−3 + 3𝑥 𝑛−4 + ⋯ + (𝑛 − 1)


Step III
Divide by 𝑥 − 1 one last time by using synthetic division:

𝑥 𝑛−2 𝑥 𝑛−3 𝑥 𝑛−4 𝑥 𝑛−5 . . . 𝑥0


1 2 3 4 . . . 𝑛−1
1 1 3 6 . . .
1 3 6 10 . . . (𝑛 − 1)𝑛
2
𝑥 𝑛−3 𝑥 𝑛−4 𝑥 𝑛−5 𝑥 𝑛−6 . . . 𝑅𝑒𝑚𝑎𝑖𝑛𝑑𝑒𝑟

Example 6.7
Verify that the formula from the previous example works for 𝑛 = 4

𝑥 2 + 4𝑥 − 3 𝑥 3 + 𝑥 2 + 𝑥 − 3 𝑥 2 + 2𝑥 + 3 6
3
= 2
= =𝑥+3+
(𝑥 − 1) (𝑥 − 1) 𝑥−1 𝑥−1

P a g e 44 | 88
Aziz Manva ([email protected])

6.2 Remainders
A. Degree of Remainder

6.8: Degree of Remainder


When a polynomial function is divided by another polynomial function the degree of Remainder is at max, 1 less
than degree of Divisor

So,
𝐷𝑒𝑔𝑟𝑒𝑒[𝑅(𝑥)] ≤ 𝐷𝑒𝑔𝑟𝑒𝑒[𝑃(𝑥)] − 1
where
𝑃1 (𝑥)
= 𝑄(𝑥) + 𝑅(𝑥)
𝑃2 (𝑥)
𝑃1 (𝑥) = 𝐷𝑖𝑣𝑖𝑑𝑒𝑛𝑑
𝑃2 (𝑥) = 𝐷𝑖𝑣𝑖𝑠𝑜𝑟
𝑄(𝑥) = 𝑄𝑢𝑜𝑡𝑖𝑒𝑛𝑡
𝑅(𝑥) = 𝑅𝑒𝑚𝑎𝑖𝑛𝑑𝑒𝑟

Example 6.9
𝑃(𝑥) is divided by a quadratic expression. What are the possible values that the degree of the remainder can be?

𝐷𝑒𝑔𝑟𝑒𝑒 ∈ {0,1}
B. Finding Linear Remainders

6.10: Dividing by Linear Expression


𝑓(𝑥) = (𝑥
⏟ − 𝑎) 𝑃(𝑥)
⏟ + ⏟
𝑟
𝐷𝑖𝑣𝑖𝑠𝑜𝑟 𝑄𝑢𝑜𝑡𝑖𝑒𝑛𝑡 𝑅𝑒𝑚𝑎𝑖𝑛𝑑𝑒𝑟

6.11: Degree of Remainder


Suppose
➢ 𝑃(𝑥) is a polynomial of degree greater than two.
➢ 𝐷(𝑥) = (𝑥 − 𝛼)(𝑥 − 𝛽) is a quadratic expression.
➢ 𝑟 is the remainder when 𝑃(𝑥) is divided by 𝐷(𝑥)
➢ 𝑄(𝑥) is the quotient when 𝑃(𝑥) is divided by 𝑄(𝑥)

Then, we can write:


𝑃(𝑥)
⏟ =⏟ (𝑥 − 𝛼)(𝑥 − 𝛽) 𝑃(𝑥)
⏟ + ⏟
𝑟
𝐷𝑖𝑣𝑖𝑑𝑒𝑛𝑑 𝐷𝑖𝑣𝑖𝑠𝑜𝑟 𝑄𝑢𝑜𝑡𝑖𝑒𝑛𝑡 𝑅𝑒𝑚𝑎𝑖𝑛𝑑𝑒𝑟

Example 6.12
Determine the remainder of
𝑥 2021
𝑥 2 + 3𝑥 + 2

Step I: Factor the Divisor


Note that
𝑥 2 + 3𝑥 + 2 = (𝑥 + 2)(𝑥 + 1)

Step II: Find Equations for the Remainder

P a g e 45 | 88
Aziz Manva ([email protected])

⏟2021 = ⏟
𝑥 (𝑥 + 2)(𝑥 + 1) 𝑄(𝑥)
⏟ + 𝑎𝑥
⏟ + 𝑏 𝑓𝑜𝑟 𝑠𝑜𝑚𝑒 𝑎 𝑎𝑛𝑑 𝑏
𝐷𝑖𝑣𝑖𝑑𝑒𝑛𝑑 𝐷𝑖𝑣𝑖𝑠𝑜𝑟 𝑄𝑢𝑜𝑡𝑖𝑒𝑛𝑡 𝑅𝑒𝑚𝑎𝑖𝑛𝑑𝑒𝑟

Note that if any term of the Divisor becomes zero, it multiplies with the quotient, and we are only left with the
remainder term.
Substitute 𝑥 + 1 = 0 ⇒ 𝑥 = −1:
(−1)2021 = (𝑥 + 2)(−1 + 1)𝑄(𝑥) + 𝑎(−1) + 𝑏

The term with the quotient is multiplied with zero, and hence it “vanishes”:
−1 = −𝑎 + 𝑏 ⇒ ⏟ 1=𝑎−𝑏
𝑬𝒒𝒖𝒂𝒕𝒊𝒐𝒏 𝑰

Substitute 𝑥 + 2 = 0 ⇒ 𝑥 = −2:
(−2)2021 = (−2 + 2)(−2 + 1)𝑄(𝑥) + 𝑎(−2) + 𝑏
The term with the quotient is multiplied with zero, and hence it “vanishes”:
⏟ 2021 = −2𝑎 + 𝑏
(−2)
𝑬𝒒𝒖𝒂𝒕𝒊𝒐𝒏 𝑰𝑰

Step III: Solve the Equations to find the values of 𝒂 and 𝒃


Add Equations I and II:
(−2)2021 + 1 = −𝑎
𝑎 = 22021 − 1

Substitute 𝑎 = 22021 − 1 in Equation I:


1 = 22021 − 1 − 𝑏
𝑏 = 22021 − 2

When divided 𝑥 2021 by 𝑥 2 + 3𝑥 + 2:


𝑅𝑒𝑚𝑎𝑖𝑛𝑑𝑒𝑟 = 𝑎𝑥 + 𝑏 = (22021 − 1)𝑥 + 22021 − 2

Example 6.13
The remainder 𝑅 obtained by dividing 𝑥 100 by 𝑥 2 − 3𝑥 + 2 is a polynomial of degree less than 2. Then 𝑅 may be
written as: (AHSME 1969/34)

Step I: Factor the Divisor


Note that
𝑥 2 − 3𝑥 + 2 = (𝑥 − 2)(𝑥 − 1)

Step II: Find Equations for the Remainder


100 (𝑥 − 2)(𝑥 − 1) 𝑄(𝑥)
𝑥⏟ =⏟ ⏟ + 𝑎𝑥
⏟ + 𝑏 𝑓𝑜𝑟 𝑠𝑜𝑚𝑒 𝑎 𝑎𝑛𝑑 𝑏
𝐷𝑖𝑣𝑖𝑑𝑒𝑛𝑑 𝐷𝑖𝑣𝑖𝑠𝑜𝑟 𝑄𝑢𝑜𝑡𝑖𝑒𝑛𝑡 𝑅𝑒𝑚𝑎𝑖𝑛𝑑𝑒𝑟

Note that if any term of the Divisor becomes zero, it multiplies with the quotient, and we are only left with the
remainder term.
Substitute 𝑥 − 1 = 0 ⇒ 𝑥 = 1:
1100 = (𝑥 − 2)(1 − 1)𝑄(𝑥) + 𝑎(1) + 𝑏

The term with the quotient is multiplied with zero, and hence it “vanishes”:
1=𝑎+𝑏

𝑬𝒒𝒖𝒂𝒕𝒊𝒐𝒏 𝑰

P a g e 46 | 88
Aziz Manva ([email protected])

Substitute 𝑥 − 2 = 0 ⇒ 𝑥 = 2:
2100 = (2 − 2)(2 − 1)𝑄(𝑥) + 𝑎(2) + 𝑏
The term with the quotient is multiplied with zero, and hence it “vanishes”:
2100 = 2𝑎 + 𝑏

𝑬𝒒𝒖𝒂𝒕𝒊𝒐𝒏 𝑰𝑰

Step III: Solve the Equations to find the values of 𝒂 and 𝒃


Subtract Equation I from Equation II:
2100 − 1 = 𝑎

Substitute 𝑎 = 2100 − 1 in Equation I:


1 = 2100 − 1 + 𝑏
𝑏 = 2 − 2100

When divided 𝑥 100 by 𝑥 2 − 3𝑥 + 2, the remainder is:


𝑅𝑒𝑚𝑎𝑖𝑛𝑑𝑒𝑟 = 𝑎𝑥 + 𝑏 = (2100 − 1)𝑥 + 2 − 2100

6.3 𝒙𝟐 + 𝒙 + 𝟏
A. Division

Example 6.14

B. Properties of 𝒙𝟐 + 𝒙 + 𝟏 = 𝟎
𝑥 2 + 𝑥 + 1 has three terms. We can, perhaps, counter-intuitively, simplify it by multiplying.

6.15: Simplifying
𝑥2 + 𝑥 + 1 = 0 ⇒ 𝑥3 = 1

Multiply both sides by 𝑥 − 1:


𝑥3 = 1

Example 6.16
Show by reduction that if 𝑥 2 + 𝑥 + 1 = 0, then 𝑥 3 = 1.

𝑥 2 = −𝑥 − 1
𝑥 3 = 𝑥 ∙ 𝑥 2 = 𝑥(−𝑥 − 1) = −𝑥 2 − 𝑥 = −(−𝑥 − 1) − 𝑥 = 𝑥 + 1 − 𝑥 = 1

6.17: Reduction Technique


The reduction technique lowers the degree of a polynomial by substituting an equivalent value of lower degree.

C. Remainders
We will do the next question in multiple ways. The underlying concept remains the same, but it gets used in
different ways. Make sure you understand both the concept and the technique.
We do this using division first.

P a g e 47 | 88
Aziz Manva ([email protected])

(Algebraic Manipulation Method) Challenge 6.18


Determine the remainder when (𝑥 4 − 1)(𝑥 2 − 1) is divided by 1 + 𝑥 + 𝑥 2 . (HMMT 2000, Guts Round/13)

Note that:
(1 + 𝑥 + 𝑥 2 )(𝑥 − 1) = 𝑥 3 − 1

𝑬𝒒𝒖𝒂𝒕𝒊𝒐𝒏 𝑰
⏟ + 𝑥 + 𝑥 2 )(𝑥 − 1)𝑥 = (𝑥 3 − 1)𝑥 = 𝑥 4 − 𝑥
(1
𝑬𝒒𝒖𝒂𝒕𝒊𝒐𝒏 𝑰𝑰

Start with the division:


(𝑥 4 − 1)(𝑥 2 − 1) (𝑥 4 − 𝑥 + 𝑥 − 1)(𝑥 2 − 1) (𝑥 4 − 𝑥)(𝑥 2 − 1) (𝑥 − 1)(𝑥 2 − 1)
= = +
1 + 𝑥 + 𝑥2 1 + 𝑥 + 𝑥2 ⏟ 1 + 𝑥 + 𝑥2 ⏟ 1 + 𝑥 + 𝑥2
𝑻𝒆𝒓𝒎 𝑨 𝑻𝒆𝒓𝒎 𝑩

Term A is now completely divisible by using Equation II:


(𝑥 4 − 𝑥)(𝑥 2 − 1) [(1 + 𝑥 + 𝑥 2 )(𝑥 − 1)𝑥][𝑥 2 − 1]
= = [(𝑥 − 1)𝑥][𝑥 2 − 1]
1 + 𝑥 + 𝑥2 1 + 𝑥 + 𝑥2

Hence, we only need to consider Term B:


(𝑥 − 1)(𝑥 2 − 1) 𝑥 3 − 𝑥 2 − 𝑥 + 1 𝑥3 − 1 𝑥2 + 𝑥 − 2
= = −
1 + 𝑥 + 𝑥2 1 + 𝑥 + 𝑥2 1 + 𝑥 + 𝑥2 ⏟ 1 + 𝑥 + 𝑥2

𝑻𝒆𝒓𝒎 𝑪 𝑻𝒆𝒓𝒎 𝑫
By Equation I, Term C is also completely divisible. Hence, we only need to consider Term D:
𝑥2 + 𝑥 + 1 3 3
− + = −1 +
1 + 𝑥 + 𝑥2 1 + 𝑥 + 𝑥2 1 + 𝑥 + 𝑥2

Hence, the remainder is 3.

(Alternate: Remainder Theorem Method) Challenge 6.19


Determine the remainder when (𝑥 4 − 1)(𝑥 2 − 1) is divided by 1 + 𝑥 + 𝑥 2 . (HMMT 2000, Guts Round/13)
A. State the division in the form 𝐷𝑖𝑣𝑖𝑠𝑜𝑟 = 𝐷𝑖𝑣𝑖𝑑𝑒𝑛𝑑 × 𝑄𝑢𝑜𝑡𝑖𝑒𝑛𝑡 + 𝑅𝑒𝑚𝑎𝑖𝑛𝑑𝑒𝑟
B. Multiply both sides by 𝑥 − 1. How does this help?
C. We now want to make the term with the Quotient vanish. Substituting 𝑥 = 1 will do that. But why is it
not a good idea?
D. Show that 𝑥 3 = 1𝑥 ≠ 1 ⇒ 𝑥 2 + 𝑥 + 1 = 0
E. Instead, make the substitution 𝑥 3 = 1, 𝑥 ≠ 1.

Part A
We can get the same result that we obtained above by stating the division in terms of what the remainder
theorem tells us.

(𝑥 4 − 1)(𝑥 2 − 1) = ⏟
⏟ (1 + 𝑥 + 𝑥 2 ) 𝑄(𝑥)
⏟ + 𝑅(𝑥)

𝐷𝑖𝑣𝑖𝑠𝑜𝑟 𝐷𝑖𝑣𝑖𝑑𝑒𝑛𝑑 𝑄𝑢𝑜𝑡𝑖𝑒𝑛𝑡 𝑅𝑒𝑚𝑎𝑖𝑛𝑑𝑒𝑟

Part B
Note that 𝑥 − 1 multiplied by the dividend gives us 𝑥 3 − 1:
(𝑥 4 − 1)(𝑥 2 − 1) = (𝑥 3 − 1) 𝑄(𝑥)
(𝑥 − 1) ⏟ ⏟ + 𝑅(𝑥)
⏟ (𝑥 − 1)
𝐷𝑖𝑣𝑖𝑠𝑜𝑟 𝑄𝑢𝑜𝑡𝑖𝑒𝑛𝑡 𝑅𝑒𝑚𝑎𝑖𝑛𝑑𝑒𝑟

Part C

P a g e 48 | 88
Aziz Manva ([email protected])

Because then the Remainder term becomes


𝑅(𝑥)
⏟ (𝑥 − 1) = 𝑅(𝑥) × 0 = 0
𝑅𝑒𝑚𝑎𝑖𝑛𝑑𝑒𝑟
And if you are not careful, this can lead you to conclude that the remainder is zero.

Part D
𝑥 3 − 1 = 0 ⇒ (𝑥 − 1)(𝑥 2 + 𝑥 + 1) = 0 ⇒ 𝑥 2 + 𝑥 + 1 = 0
Part E
Substitute 𝑥 3 = 1, 𝑥 ≠ 1:
The term with the quotient vanishes.
Also, since 𝑥 4 = 𝑥 ∙ 𝑥 3 = 𝑥 ∙ 1 = 𝑥:
(𝑥 − 1)(𝑥 − 1)(𝑥 2 − 1) = 𝑅(𝑥)
⏟ (𝑥 − 1)
𝑅𝑒𝑚𝑎𝑖𝑛𝑑𝑒𝑟
Since 𝑥 ≠ 1, divide both sides by 𝑥 − 1:
(𝑥 − 1)(𝑥 2 − 1) = 𝑅(𝑥)

𝑅𝑒𝑚𝑎𝑖𝑛𝑑𝑒𝑟

We might feel like stopping here, but the expression can be further simplified:
𝑅(𝑥) = 𝑥 3 − 𝑥 2 − 𝑥 + 1
3
Substitute 𝑥 = 1:
𝑅(𝑥) = −𝑥 2 − 𝑥 + 2 = −(𝑥 2 + 𝑥 + 1) + 3
2
Substitute 𝑥 + 𝑥 + 1 = 0:
𝑅(𝑥) = 3

(Shorter Method) Challenge 6.20


Determine the remainder when (𝑥 4 − 1)(𝑥 2 − 1) is divided by 1 + 𝑥 + 𝑥 2 . (HMMT 2000, Guts Round/13)

(𝑥 4 − 1)(𝑥 2 − 1) = ⏟
⏟ (1 + 𝑥 + 𝑥 2 ) 𝑄(𝑥)
⏟ + 𝑅(𝑥)

𝐷𝑖𝑣𝑖𝑠𝑜𝑟 𝐷𝑖𝑣𝑖𝑑𝑒𝑛𝑑 𝑄𝑢𝑜𝑡𝑖𝑒𝑛𝑡 𝑅𝑒𝑚𝑎𝑖𝑛𝑑𝑒𝑟
Substitute 𝑥 3 = 1, 𝑥 ≠ 1 ⇒ 𝑥 2 + 𝑥 + 1 = 0:
(𝑥 − 1)(𝑥 2 − 1) = (0) 𝑄(𝑥)
⏟ + 𝑅(𝑥)

𝑅𝑒𝑚𝑎𝑖𝑛𝑑𝑒𝑟
⏟3 − 𝑥 2 − 𝑥 + 1 = − ⏟
𝑅(𝑥) = 𝑥 (𝑥 2 + 𝑥 + 1) + 3 = 3
=0 =0

D. Basics

Example 6.21
Find the remainder when 𝑥 2022 is divided by 𝑥 2 + 𝑥 + 1

𝑥 2022 = (𝑥 2 + 𝑥 + 1)𝑄(𝑥) + 𝑅(𝑥)


Substitute 𝑥 2 + 𝑥 + 1 = 0 ⇒ 𝑥 3 = 1:
𝑅(𝑥) = 𝑥 2022
But note that 2022 has sum of digits 2 + 0 + 2 + 2 = 6 ⇒ 2022 is divisible by 3:

Hence, for some natural number 𝑛:


𝑅(𝑥) = 𝑥 2022 = (𝑥 3 )𝑛 = 1𝑛 = 1

Example 6.22
Find the remainder when 𝑥 2020 is divided by 𝑥 2 + 𝑥 + 1

P a g e 49 | 88
Aziz Manva ([email protected])

𝑥 2020 = (𝑥 2 + 𝑥 + 1)𝑄(𝑥) + 𝑅(𝑥)


Substitute 𝑥 2 + 𝑥 + 1 = 0 ⇒ 𝑥 3 = 1:
𝑅(𝑥) = 𝑥 2020
But note that 2022 has sum of digits 2 + 0 + 2 + 0 = 4 ⇒ 2022 has remainder 1 when divided by 3:

Hence, for some natural number 𝑛:


𝑅(𝑥) = 𝑥 3𝑛+1 = 𝑥 3𝑛 × 𝑥 = 1 × 𝑥 = 𝑥

Example 6.23
Find the remainder when 𝑥 2021 is divided by 𝑥 2 + 𝑥 + 1

𝑥 2021 = (𝑥 2 + 𝑥 + 1)𝑄(𝑥) + 𝑅(𝑥)


Substitute 𝑥 2 + 𝑥 + 1 = 0 ⇒ 𝑥 3 = 1:
𝑅(𝑥) = 𝑥 2021
But note that 2021 has sum of digits 2 + 0 + 2 + 1 = 5 ⇒ 2021 has remainder 2 when divided by 3:

Hence, for some natural number 𝑛:


𝑅(𝑥) = 𝑥 3𝑛+2 = 𝑥 3𝑛 × 𝑥 2 = 1 × 𝑥 2 = 𝑥 2

However, when dividing by a quadratic the highest possible degree of the remainder is linear. Hence, we need to
reduce the expression above still further.

Substitute 𝑥 2 + 𝑥 + 1 = 0 ⇒ 𝑥 2 = −𝑥 − 1:
𝑅(𝑥) = 𝑥 2 = −𝑥 − 1
E. Applications

Example 6.24
Find the remainder when 20202021 + 2020 is divided by 20202 + 2021

Let 𝑥 = 2020:
𝑥 2021 + 𝑥 = (𝑥 2 + 𝑥 + 1)𝑄(𝑥) + 𝑅(𝑥)

Substitute 𝑥 2 + 𝑥 + 1 = 0 ⇒ 𝑥 3 = 1, 𝑥 ≠ 1:
𝑅(𝑥) = 𝑥 2021 + 𝑥 = 𝑥 2 + 𝑥
2
Substitute 𝑥 = −𝑥 − 1:
𝑅(𝑥) = −𝑥 − 1 + 𝑥 = −1

Interpret the negative remainder:


𝑅(𝑥) = 20202 + 2021 − 1 = 20202 + 2020 = 2020(2021)

P a g e 50 | 88
Aziz Manva ([email protected])

PART III: EQUATIONS


7. EXPRESSIONS AND EQUATIONS
7.1 Expressions
A. Introduction

Example 7.1
If 𝑥 = 3 + √2, find the value of 𝑥 3 − 6𝑥 2 + 9𝑥 − 2√2

𝑥(𝑥 2 − 6𝑥 + 9) − 2√2 = 𝑥(𝑥 − 3)2 − 2√2


Substitute 𝑥 = 3 + √2 ⇒ 𝑥 − 3 = √2 ⇒ 𝑥 − 3 = 2:
2𝑥 − 2√2 = 2(𝑥 − √2)
Substitute 𝑥 = 3 + √2 ⇒ 𝑥 − √2 = 3:
2×3=6

7.2 Cubic and Quartic Equations


A. Introduction
We have seen in the Note on Quadratics how to solve, graph and analyze quadratic equations, or equations
which can be reduced to Quadratics.
Here, we look at equations of higher degree.
B. Polynomials, Equations and Functions

7.2: Cubic and Quartic Polynomials


A cubic polynomial is a third-degree polynomial. That is, it has the form:
𝑎𝑥 3 + 𝑏𝑥 2 + 𝑐𝑥 + 𝑑, 𝑎≠0

A quartic polynomial is a fourth-degree polynomial. That is, it has the form:


𝑎𝑥 4 + 𝑏𝑥 3 + 𝑐𝑥 2 + 𝑑𝑥 + 𝑒, 𝑎≠0

There are two key points here


➢ 𝑎≠0
➢ The term with highest power is of degree 3 (for cubic) or degree 4 (for quartic).

7.3: Cubic and Quartic Equations


A cubic equation is an equation of the form
𝑎𝑥 3 + 𝑏𝑥 2 + 𝑐𝑥 + 𝑑 = 0, 𝑎≠0

A quartic equation is an equation of the form


𝑎𝑥 4 + 𝑏𝑥 3 + 𝑐𝑥 2 + 𝑑𝑥 + 𝑒, 𝑎≠0

Example 7.4
What is the degree of
A. A cubic polynomial
B. A quartic polynomial

P a g e 51 | 88
Aziz Manva ([email protected])

3
4
C. Coefficients

7.5: Coefficients
𝑎𝑥 3 + 𝑏𝑥 2 + 𝑐𝑥 + 𝑑 ⇒ 𝐶𝑜𝑒𝑓𝑓𝑖𝑐𝑖𝑒𝑛𝑡𝑠 𝑎𝑟𝑒 𝑎, 𝑏, 𝑐, 𝑑
𝑎𝑥 4 + 𝑏𝑥 3 + 𝑐𝑥 2 + 𝑑𝑥 + 𝑒 ⇒ 𝐶𝑜𝑒𝑓𝑓𝑖𝑐𝑖𝑒𝑛𝑡𝑠 𝑎𝑟𝑒 𝑎, 𝑏, 𝑐, 𝑑, 𝑒

7.6: Real Values for Coefficients


Unless otherwise stated, we will consider only real values for coefficients.
We can have complex values for solutions.

Example 7.7
What are the coefficients of:
4𝑥 3 + 3𝑥 2 − 2𝑥 + 7 = 0

𝑎 = 4, 𝑏 = 3, 𝑐 = −2, 𝑑=7

7.8: Sum of Coefficients


The sum of the coefficients of an equation or expression are found by substituting
𝑥=1

Example 7.9
Find the sum of the coefficients of:
1 3 3 2 5
𝑥 + 𝑥 − 𝑥+2=0
2 4 3

Substitute 𝑥 = 1:
1 3 5
+ − +2=
2 4 3

D. Finding Roots

Example 7.10
The roots of (𝑥 2 − 3𝑥 + 2)(𝑥)(𝑥 − 4) = 0 are: (AHSME 1950/13)
0,1,2,4

E. Trivial Cubics
Trivial cubics are equations that have degree three, but do not require sophisticated methods to solve them.
They look like third degree equations, but can be solved much faster.

Example 7.11
Solve:
A. 𝑥 3 = 27
B. 𝑥 3 = −27
C. 𝑥 3 = 16

P a g e 52 | 88
Aziz Manva ([email protected])

3
𝑥 3 = 27 ⇒ 𝑥 = √27 = 3
3
𝑥 3 = −27 ⇒ 𝑥 = √−27 = −3
3 3 3 3
𝑥 3 = 16 ⇒ 𝑥 = √16 = √8 × √2 = 2√2

7.12: Cube Root of a Number


1
3
𝑥 3 = 𝑎 ⇒ 𝑥 = √𝑎 = 𝑎3

Note two differences from the equation below:


𝑥 2 = 𝑎 ⇒ 𝑥 = ±√𝑎

➢ The cubic equation has only one real solution, not two.
➢ The cubic equation always has a real solution, unlike the square equation, which only has a solution
when 𝑎 ≥ 0.
F. Cubic Equations Through the Origin

7.13: Equations through the Origin


When the graph of an equation passes through the origin, zero is a root of the equation, and this can be easily
factored.

Example 7.14
Find the mean of all solutions for 𝑥 when 𝑥 3 + 3𝑥 2 − 10𝑥 = 0. (MathCounts 1995 Warm-Up 10)

𝑥(𝑥 2 + 3𝑥 − 10) = 0
𝑥(𝑥 + 5)(𝑥 − 2) = 0
Roots are
0,2, −5
Mean of Roots
0+2−5 3
= = − = −1
3 3
G. Forming Equations
Some questions may require you to form an equation that has a given set of roots.

7.15: Forming Equations with Given Roots


Given the roots 𝛼, 𝛽, 𝛾, all equations with these roots must of the form:
𝑎(𝑥 − 𝛼)(𝑥 − 𝛽)(𝑥 − 𝛾) = 0, 𝑎≠0

Where 𝑎 is a scaling factor.

Example 7.16
Find the general cubic equation that has roots 1, −1,2. State your answer as a third-degree polynomial in
standard form.

We know that the roots are


𝑥 =1⇒𝑥−1=0
𝑥 = −1 ⇒ 𝑥 + 1 = 0
𝑥 =2⇒𝑥−2=0

P a g e 53 | 88
Aziz Manva ([email protected])

Multiply the above three:


𝑎(𝑥 − 1)(𝑥 + 1)(𝑥 − 2) = 0
𝑎(𝑥 2 − 1)(𝑥 − 2) = 0
𝑎(𝑥 3 − 2𝑥 2 − 𝑥 + 2) = 0, 𝑎≠0

Example 7.17
Find all cubic equations with roots 1, −1. State your answer as a third-degree polynomial in standard form.

Case I: 1 is a repeated root


𝑎(𝑥 − 1)2 (𝑥 + 1) = 0
Case I: −𝟏 is a repeated root
𝑎(𝑥 + 1)2 (𝑥 − 1) = 0

7.3 Rational Roots Theorem


A. Idea
The rational roots theorem does not guarantee finding any roots.
What it does do is give a list of potential roots to check using the remainder theorem.

7.18: Rational Roots Theorem


Any rational root of a polynomial must belong to the set of potential roots generated by the following formula
𝐹𝑎𝑐𝑡𝑜𝑟 𝑜𝑓 𝑐𝑜𝑛𝑠𝑡𝑎𝑛𝑡 𝑡𝑒𝑟𝑚
±
𝐹𝑎𝑐𝑡𝑜𝑟 𝑜𝑓 𝑙𝑒𝑎𝑑𝑖𝑛𝑔 𝑐𝑜𝑒𝑓𝑓𝑖𝑐𝑖𝑒𝑛𝑡

The theorem only talks about rational roots. It does not:


➢ Guarantee the existence of rational roots
➢ Allow you to find complex roots
B. Solving Cubics
Cubics equations which have rational roots can be solved using the rational roots theorem. If the cubic equations
do not have rational roots, then the rational roots theorem will not be able to find the roots.

Exam questions will rarely be “nice” enough to tell you whether the roots are rational or not. However, if they
are in an exam, they are meant to be solved in some way, and the rational roots theorem is a primary method of
attack. Some experience and practice in solving these types of questions is very useful.

You can watch this video2 to see an example of the technique in action. While the rational roots theorem is not
mentioned, it is the underlying technique used to get the roots.

Example 7.19
Solve
2𝑥 3 + 3𝑥 2 − 32𝑥 + 15 = 0

Make a list of potential factors to check:


𝐹𝑎𝑐𝑡𝑜𝑟𝑠 𝑜𝑓 15 = ±{1,3,5,15}
𝐹𝑎𝑐𝑡𝑜𝑟𝑠 𝑜𝑓 2 = ±{1,2}

2
https://www.youtube.com/watch?v=HmtZvUVe9fo

P a g e 54 | 88
Aziz Manva ([email protected])

The factors to check are:


1 3 5 15 1 3 5 15
± , , , , , , ,
1 1 1 1 2 2 2 2

Let 𝑃(𝑥) = 2𝑥 3 + 3𝑥 2 − 32𝑥 + 15

𝑥 − 1 = 0 ⇒ 𝑥 = 1 ⇒ 𝑃(1) = 2 + 3 − 32 + 15 = −12
𝑥 + 1 = 0 ⇒ 𝑥 = −1 ⇒ 𝑃(−1) = −2 + 3 + 32 + 15 ≠ 0

We don’t to think of the roots first. Since we need to check both positive and negative values, we can just
substitute the values:
𝑃(3) = 54 + 27 − 96 + 15 = 0 𝑥3 𝑥2 𝑥 𝐶𝑜𝑛𝑠𝑡𝑎𝑛𝑡
𝑥 = 3 ⇒ 𝑥 − 3 𝑖𝑠 𝑎 𝑓𝑎𝑐𝑡𝑜𝑟 3 2 3 −32 15
6 27 −15
2 2 9 −5 0
2𝑥 + 9𝑥 − 5
𝑥2 𝑥 𝐶𝑜𝑛𝑠𝑡𝑎𝑛𝑡 Remainder
𝑃 = −10 = (10)(−1), 𝑆=9

2𝑥 2 + 10𝑥 − 𝑥 − 5 = 2𝑥(𝑥 − 5) − 1(𝑥 + 5) = (2𝑥 − 1)(𝑥 + 5)

2𝑥 3 + 3𝑥 2 − 32𝑥 + 15 = 0
(𝑥 − 3)(2𝑥 − 1)(𝑥 + 5) = 0

Apply the zero-product property:


𝑥−3=0⇒𝑥 =3
1
2𝑥 − 1 = 0 ⇒ 𝑥 =
2
𝑥 + 5 = 0 ⇒ 𝑥 = −5

And we can combine the three solutions to get:


1
𝑥 ∈ {−5, , 3}
2

Example 7.20
Solve
3𝑥(𝑥 2 + 6) = 8 − 17𝑥 2

3𝑥 3 + 17𝑥 2 + 18𝑥 − 8 = 0
𝐹𝑎𝑐𝑡𝑜𝑟𝑠 𝑜𝑓 8: ± 1,2,4,8

𝑃(1) = 3 + 17 + 18 − 8 ≠ 0
𝑃(−1) = −3 + 17 − 18 − 8 ≠ 0
𝑃(2) = 24 + 68 + 36 − 8 ≠ 0
𝑃(−2) = −24 + 68 − 36 − 8 = 0 ⇒ 𝑥 = −2 ⇒ 𝑥 + 2 = 0

3𝑥 2 + 11𝑥 − 4 = 3𝑥 2 + 12𝑥 − 𝑥 − 4 = 3𝑥(𝑥 + 4) − 1(𝑥 + 4)


= (3𝑥 − 1)(𝑥 + 4) 𝑥3 𝑥2 𝑥 𝐶𝑜𝑛𝑠𝑡𝑎𝑛𝑡
𝑃 = −12, 𝑆𝑢𝑚 = 11 = 12 − 1 −2 3 17 18 −8
−6 −22 8
(𝑥 + 2)(3𝑥 − 1)(𝑥 + 4) = 0 3 11 −4 0
𝑥2 𝑥 𝐶𝑜𝑛𝑠𝑡𝑎𝑛𝑡
𝑥 + 2 = 0 ⇒ 𝑥 = −2

P a g e 55 | 88
Aziz Manva ([email protected])

1
3𝑥 − 1 = 0 ⇒ 𝑥 =
3
𝑥 + 4 = 0 ⇒ 𝑥 = −4

1
𝑥 ∈ {−4, −2, }
3

Example 7.21
Solve
2𝑥 3 − 3𝑥 2 − 11𝑥 + 6 = 0

𝐹𝑎𝑐𝑡𝑜𝑟𝑠 𝑜𝑓 6: ±1,2,3,6
𝑃(1) = 2 − 3 − 11 + 6 = −6
𝑃(−1) = −2 − 3 + 11 + 6 = 12
𝑃(2) = 16 − 12 − 22 + 6 = −12
𝑃(−2) = −16 − 12 + 22 + 6 = 0

Type equation here.

2𝑥 2 − 7𝑥 + 3
x=-2,3,1/2

Example 7.22
Solve
2𝑥 3 − 11𝑥 2 − 20𝑥 − 7 = 0

7 2 -11 -20 -7
14 21 7
2 3 1 0

2𝑥 2 + 3𝑥 + 1 = 0

(x+1)(x-7)(2x+1)

Example 7.23
Solve
3𝑥 3 + 7𝑥 2 − 22𝑥 − 8 = 0

(x-2)(x+4)(3x+1)
1
𝑥 ∈ {−4, − , 2}
3

Example 7.24
Show that 2𝑥 3 − 5𝑥 2 + 10𝑥 − 4 = 0 has only one real root.

𝐹𝑎𝑐𝑡𝑜𝑟𝑠 𝑜𝑓 4: ± 1,2,4
𝐹𝑎𝑐𝑡𝑜𝑟𝑠 𝑜𝑓 2: ±1,2
1 2 4 1 2 4 1 1
± { , , , , , } = ± {1,2,4, , 𝟏, 𝟐} = ± {1,2,4, }
1 1 1 2 2 2 2 2

P a g e 56 | 88
Aziz Manva ([email protected])

𝑃(1) =
𝑃(−1) = 1 2 -5 10 -4
𝑃(2) = 2
𝑃(−2) = 1 -2 4
𝑃(4) = 2 -4 8 0
𝑃(−4) =

1 1 3 1 2 1 1 5
𝑃 ( ) = 2 ( ) − 5 ( ) + 10 ( ) − 4 = − + 5 − 4 = 0
2 2 2 2 4 4

1
(𝑥 − ) (2𝑥 2 − 4𝑥 + 8) = 0
2

Calculate the discriminant of the quadratic by substituting 𝑎 = 2, 𝑏 = −4, 𝑐 = 8:


𝑏 2 − 4𝑎𝑐 = (−4)2 − (4)(2)(8) = 16 − 64 = −48 < 0
Hence, the quadratic has two complex roots.

Hence, the cubic has one real solution, given by:


1 1
𝑥− =0⇒𝑥=
2 2
And two complex solutions.

Hence, it has only one real root.


C. Remainder Theorem

Example 7.25
4𝑥 3 + 2𝑎𝑥 − 7𝑎 leaves a remainder of −10 when divided by 𝑥 − 𝑎. Find the value of 𝑎.

𝑥 − 𝑎 = 0 ⇒ 𝑥 = 𝑎 ⇒ 𝑓(𝑎) = 4𝑎3 + 2𝑎2 − 7𝑎 = −10


4𝑎3 + 2𝑎2 − 7𝑎 + 10 = 0

Example 7.26
The polynomial 3𝑥 3 + 𝑎𝑥 + 5𝑎 leaves a remainder of −7 when divided by 𝑥 − 𝑎. Find all values of 𝑎.

𝑓(𝑎) = 3𝑎3 + 𝑎2 + 5𝑎 + 7
(𝑎 + 1)(3𝑎2 − 2𝑎 + 7) = 0

Discriminant of Quadratic:
b^2-4ac=4-(4)(3)(7)=4-72=-68
No Solutions

D. Fourth Degree Equations

E. Simultaneous Equations

Example 7.27
P a g e 57 | 88
Aziz Manva ([email protected])

Find the potential rational roots of


A. 2𝑥^2 + 3𝑥^2 − 11𝑥 − 6

F. HCF and LCM


Since we are now (theoretically) in a position to find the rational roots of any polynomial, we can factor it. A
direct application is to find the 𝐻𝐶𝐹 of two polynomials.

Example 7.28
Find the 𝐻𝐶𝐹 of 3𝑥 3 + 𝑥 2 − 5𝑥 + 21 and 6𝑥 3 + 29𝑥 2 + 26𝑥 − 21
First Polynomial: 𝑷(𝒙) = 𝟔𝒙𝟑 + 𝟐𝟗𝒙𝟐 + 𝟐𝟔𝒙 − 𝟐𝟏

1 3 7 21 1 7
±{1,3,7,21}
⏟ , ±{ , , , } = ± { , 𝟏, , 𝟕}
⏟ 6 6 6 6 ⏟ 6 6
𝑫𝒆𝒏𝒐𝒎𝒊𝒏𝒂𝒕𝒐𝒓 𝟏
𝑫𝒆𝒏𝒐𝒎𝒊𝒏𝒂𝒕𝒐𝒓 𝟔 𝟏 𝒂𝒏𝒅 𝟕 𝒂𝒓𝒆 𝒓𝒆𝒑𝒆𝒂𝒕𝒆𝒅
Try the roots starting from the simplest.
𝑥 = ±1 does not work
𝑥 = 1 ⇒ 𝑃(𝑥) = 6 + 29 + 26 − 21 = 40 ≠ 0 ⇒ 𝑁𝑜𝑡 𝑎 𝑓𝑎𝑐𝑡𝑜𝑟
𝑥 = −1 ⇒ 𝑃(𝑥) = −6 + 29 − 26 − 21 = −24 ≠ 0 ⇒ 𝑁𝑜𝑡 𝑎 𝑓𝑎𝑐𝑡𝑜𝑟
𝑥 = 3 does not work
𝑥 = 3 ⇒ 𝑃(𝑥) = 6(27) + 29(9) + 26(3) − 21 ≠ 0 ⇒ 𝑁𝑜𝑡 𝑎 𝑓𝑎𝑐𝑡𝑜𝑟
𝑥 = −3 does work
6(−27) + 29(9) + 26(−3) − 21 = −162 + 261 − 78 − 21 = 0 ⇒ 𝑥⏟+ 3
𝑰𝒔 𝒂 𝒇𝒂𝒄𝒕𝒐𝒓

−3 6 29 26 −21
−18 −33 21
6 11 −7 0

6𝑥 3 + 29𝑥 2 + 26𝑥 − 21 = (6𝑥 2 + 11𝑥 − 7)(𝑥 + 3) = (2𝑥 − 1)(3𝑥 + 7)(𝑥 + 3)

We want to factor 6𝑥 2 + 11𝑥 − 7


𝑃𝑟𝑜𝑑𝑢𝑐𝑡 = −7 × 6 = −42 = (−1)(42) = (−2)(21) = (−3)(14)
𝑆𝑢𝑚 = 11
2
6𝑥 + 14𝑥 − 3𝑥 − 7 = 2𝑥(3𝑥 + 7) − 1(3𝑥 + 7) = (2𝑥 − 1)(3𝑥 + 7)

Second Polynomial
𝐶𝑜𝑛𝑠𝑡𝑎𝑛𝑡 𝑇𝑒𝑟𝑚 = 21 ⇒ 𝐹𝑎𝑐𝑡𝑜𝑟𝑠 𝑜𝑓 21 = ±{1,3,7,21}
𝐿𝑒𝑎𝑑𝑖𝑛𝑔 𝐶𝑜𝑒𝑓𝑓𝑖𝑐𝑖𝑒𝑛𝑡 = 3 ⇒ 𝐹𝑎𝑐𝑡𝑜𝑟𝑠 𝑜𝑓 3 = ± {1,3}

Combinations from {1,3,7,21}


⏟ and {1,3}
⏟ :
𝑵𝒖𝒎𝒆𝒓𝒂𝒕𝒐𝒓 𝑫𝒆𝒏𝒐𝒎𝒊𝒏𝒂𝒕𝒐𝒓
1 3 7 21 1 7
±{1,3,7,21}
⏟ , ±{ , , , } = ± { , 𝟏, , 𝟕}
⏟ 3 3 3 3 ⏟ 3 3
𝑫𝒆𝒏𝒐𝒎𝒊𝒏𝒂𝒕𝒐𝒓 𝟏
𝑫𝒆𝒏𝒐𝒎𝒊𝒏𝒂𝒕𝒐𝒓 𝟑 𝟏 𝒂𝒏𝒅 𝟕 𝒂𝒓𝒆 𝒓𝒆𝒑𝒆𝒂𝒕𝒆𝒅
1 7
𝐹𝑖𝑛𝑎𝑙 𝑆𝑒𝑡 𝑜𝑓 𝑃𝑜𝑡𝑒𝑛𝑡𝑖𝑎𝑙 𝑅𝑜𝑜𝑡𝑠 𝑡𝑜 𝐶ℎ𝑒𝑐𝑘 = ± {1,3,7,21, , }
3 3

P a g e 58 | 88
Aziz Manva ([email protected])

7.4 Misc: Equations in Two Variables


Example 7.29
Find the value(s) of 𝑥 such that 8𝑥𝑦 − 12𝑦 + 2𝑥 − 3 = 0 is true for all values of 𝑦. (AMC 10B 2002/13)

Factor the equation:


(4𝑦 + 1)(2𝑥 − 3) = 0
If the second term is zero, then the equality holds for all values of 𝑦.
3
2𝑥 − 3 = 0 ⇒ 𝑥 =
2

In the above question, find the value(s) of y such that the equation holds for all values of x.

(4𝑦 + 1)(2𝑥 − 3) = 0
If the second term is zero, then the equality holds for all values of 𝑦.
1
4𝑦 + 1 = 0 ⇒ 𝑦 = −
4

Example 7.30
(𝑦 + 2)𝑥 2 + (7𝑦 + 14)𝑥 + 12𝑦
A. Find the value(s) of 𝑥 such that the above expression is −24 for all values of 𝑦.
B. Find the value(s) of 𝑦 such that the above expression is −24 for all values of 𝑥.

(𝑦 + 2)𝑥 2 + (7𝑦 + 14)𝑥 + 12𝑦 = −24


𝑥 2 𝑦 + 7𝑥𝑦 + 12𝑦 + 2𝑥 2 + 14𝑥 + 24 = 0
𝑦(𝑥 2 + 7𝑥 + 12) + 2((𝑥 2 + 7𝑥 + 12)) = 0
(𝑦 + 2)(𝑥 2 + 7𝑥 + 12) = 0

Part A
𝑥 2 + 7𝑥 + 12 = 0 ⇒ 𝑥 ∈ {−3, −4}
Part B
𝑦 + 2 = 0 ⇒ 𝑦 = −2

7.5 Misc: Solving in terms of unknown Constants


A. Basics

Example 7.31
If 𝑥 3 + 𝑥 2 + 𝑥 + 1 = 0 and 𝑥 2 + 𝑝𝑥 − 𝑞 = 0, then:
A. Find the value of x as a rational function of 𝑝 and 𝑞. (Recall that a rational function will not include
square roots).
B. Show that 𝑞 ≠ 𝑝2 + 𝑝 − 1

Part A
𝑥 2 = 𝑞 − 𝑝𝑥
𝑥(𝑞 − 𝑝𝑥) + (𝑞 − 𝑝𝑥) + 𝑥 + 1 = 0
𝑥𝑞 − 𝑝(𝑞 − 𝑝𝑥) + (𝑞 − 𝑝𝑥) + 𝑥 + 1 = 0
𝑥𝑞 − 𝑝𝑞 − 𝑝2 𝑥 + 𝑞 − 𝑝𝑥 + 𝑥 + 1 = 0

P a g e 59 | 88
Aziz Manva ([email protected])

𝑥𝑞 − 𝑝2 𝑥 − 𝑝𝑥 + 𝑥 = 𝑝𝑞 − 𝑞 − 1
𝑥(𝑞 − 𝑝2 − 𝑝 + 1) = 𝑝𝑞 − 𝑞 − 1
𝑝𝑞 − 𝑞 − 1
𝑥=
𝑞 − 𝑝2 − 𝑝 + 1

Part B

𝑞 − 𝑝2 − 𝑝 + 1 = 0 ⇒ 𝑞 = 𝑝2 + 𝑝 − 1

7.6 Graphs
A. y-intercept of the graph

7.32: y-intercept
The y-intercept of a cubic equation 𝑎𝑥 3 + 𝑏𝑥 2 + 𝑐𝑥 + 𝑑 = 0 is its constant term
𝑑

At the y-axis, 𝑥 = 0, which we substitute in the given equation:


𝑥 = 0 ⇒ 𝑎(03 ) + 𝑏(02 ) + 𝑐(0) + 𝑑 = 𝑑

B. Roots

MCQ 7.33
The adjoining graph is that of a cubic polynomial function. We can see two roots. The third root:
A. Is real
B. Is complex
C. Can be either complex or real

The graph intersects the x-axis twice, which means there are at least two real roots.
Complex roots only occur in pairs, so the third root must be real as well.

C. Repeated Roots

MCQ 7.34
The adjoining graph is that of a cubic polynomial function. We can see two roots. The third root:
A. Is real
B. Is complex
C. Can be either complex or real

The graph bounces at the x-axis. That means the root has multiplicity two.
Complex roots only occur in pairs, so the third root must be real as well.

D. End Behavior
E. Number of Turning Points

7.35: Number of Turning Points

P a g e 60 | 88
Aziz Manva ([email protected])

An equation of degree n has exactly (𝑛 − 1) turning points.

Example 7.36
How many turning points will a quartic (𝑑𝑒𝑔𝑟𝑒𝑒 4) equation have?

4 − 1 = 3 𝑇𝑢𝑟𝑛𝑖𝑛𝑔 𝑃𝑜𝑖𝑛𝑡𝑠

F. Review

P a g e 61 | 88
Aziz Manva ([email protected])

PART IV: ROOTS


8. POLYNOMIAL ROOTS
8.1 Vieta’s Formulas
A. Revision: Quadratics

8.1: Relation between Roots


For a quadratic equation, 𝑎𝑥 2 + 𝑏𝑥 + 𝑐 = 0, the roots 𝛼 and 𝛽, satisfy the relations:
𝑏 𝑐
𝛼+𝛽 =− , 𝛼𝛽 =
𝑎 𝑎

The Note on Quadratics covers the relation between roots for a quadratic in some degree of detail. You should
revise it now, if needed.
B. Relations between Roots

8.2: Relation between Roots


For a cubic equation, 𝑎𝑥 3 + 𝑏𝑥 2 + 𝑐𝑥 + 𝑑 = 0, the roots 𝛼, 𝛽, 𝛾, satisfy the relations:
𝑏
𝑆𝑢𝑚 𝑜𝑓 𝑅𝑜𝑜𝑡𝑠 = 𝛼 + 𝛽 + 𝛾 = −
𝑎
𝑐
𝑆𝑢𝑚 𝑜𝑓𝑃𝑟𝑜𝑑𝑢𝑐𝑡 𝑜𝑓 𝑅𝑜𝑜𝑡𝑠 (2 𝑎𝑡 𝑎 𝑡𝑖𝑚𝑒): 𝛼𝛽 + 𝛽𝛾 + 𝛾𝛼 =
𝑎
𝑑
𝑃𝑟𝑜𝑑𝑢𝑐𝑡 𝑜𝑓 𝑅𝑜𝑜𝑡𝑠 = 𝛼𝛽𝛾 = −
𝑎

Example 8.3
1 3 3 2 5
𝑥 + 𝑥 − 𝑥+2=0
2 4 3
For the above equation find:
A. Sum of the roots
B. Product of the roots
C. Sum of the products of two roots at a time

𝑏 3 1 3 3
𝛼+𝛽+𝛾 =− =− ÷ =− ×2=−
𝑎 4 2 4 2
𝑑 1
𝛼𝛽𝛾 = − = −2 ÷ = −4
𝑎 2
𝑐 5 1 5 10
𝛼𝛽 + 𝛽𝛾 + 𝛾𝛼 = = − ÷ = − × 2 = −
𝑎 3 2 3 3

Example 8.4
Determine the product of the roots of each equation below:
𝐸𝑞𝑢𝑎𝑡𝑖𝑜𝑛 1: 2𝑥 3 + 𝑏𝑥 2 + 𝑐𝑥 + 𝑑 = 0
𝐸𝑞𝑢𝑎𝑡𝑖𝑜𝑛 2: 8𝑥 3 − 2𝑏𝑥 2 + 𝑐 ′ 𝑥 + 𝑑

𝑑 𝑑
− 𝑎𝑛𝑑 −
2 8

P a g e 62 | 88
Aziz Manva ([email protected])

Example 8.5
Find the mean of all solutions for 𝑥 when 𝑥 3 + 3𝑥 2 − 10𝑥 = 0. (MathCounts 1995 Warm-Up 10)
𝑏 3
𝑆𝑢𝑚 𝑜𝑓 𝑅𝑜𝑜𝑡𝑠 − 𝑎 − 1
𝑀𝑒𝑎𝑛 𝑜𝑓 𝑅𝑜𝑜𝑡𝑠 = = = = −1
𝑁𝑜. 𝑜𝑓 𝑅𝑜𝑜𝑡𝑠 3 3

C. Zero Root

8.6: Zero Root


If one of the roots of a cubic equation 𝑎𝑥 3 + 𝑏𝑥 2 + 𝑐𝑥 + 𝑑 = 0 is zero, then the constant term is zero.
That is the equation reduces to
𝑥(𝑎𝑥 2 + 𝑏𝑥 + 𝑐) = 0

Substitute 𝑥 = 0:
𝑎𝑥 3 + 𝑏𝑥 2 + 𝑐𝑥 + 𝑑 = 0 ⇒ 𝑑 = 0
Substitute 𝑑 = 0:
𝑎𝑥 3 + 𝑏𝑥 2 + 𝑐𝑥 = 0 ⇒ 𝑥(𝑎𝑥 2 + 𝑏𝑥 + 𝑐) = 0

Example 8.7
One of the roots of 𝑎𝑥 3 + 𝑏𝑥 2 + 𝑐𝑥 + 𝑑 = 0 is zero. Find the product of the other two roots.

Method I
Substitute 𝑥 = 0:
𝑎𝑥 3 + 𝑏𝑥 2 + 𝑐𝑥 + 𝑑 = 0 ⇒ 𝑑 = 0
Substitute 𝑑 = 0:
𝑐
𝑎𝑥 3 + 𝑏𝑥 2 + 𝑐𝑥 = 0 ⇒ 𝑥(𝑎𝑥 2 + 𝑏𝑥 + 𝑐) = 0 ⇒ 𝑥 = 0, 𝛼𝛽 ⇒ 𝛼𝛽 =
𝑎
Method II
Without loss of generality, let 𝛾 = 0. Then:
𝑐 𝑐 𝑐
𝛼𝛽 + 𝛽𝛾 + 𝛾𝛼 = ⇒ 𝛼𝛽 + 𝛽 × 0 + 0 × 𝛼 = ⇒ 𝛼𝛽 =
𝑎 𝑎 𝑎

Example 8.8
The polynomial 𝑃(𝑥) = 𝑥 3 + 𝑎𝑥 2 + 𝑏𝑥 + 𝑐 has the property that the mean of its zeros, the product of its zeros,
and the sum of its coefficients are all equal. If the 𝑦-intercept of the graph of 𝑦 = 𝑃(𝑥) is 2, what is 𝑏? (AMC 12
2001/19)

𝑦-intercept gives us the constant term


𝑐=2

𝑃𝑟𝑜𝑑𝑢𝑐𝑡 𝑜𝑓 𝑅𝑜𝑜𝑡𝑠 = −𝑐 = −2
𝑆𝑢𝑚 𝑜𝑓 𝑅𝑜𝑜𝑡𝑠 𝑎
𝑀𝑒𝑎𝑛 𝑜𝑓 𝑟𝑜𝑜𝑡𝑠 = = − = −2 ⇒ 𝑎 = 6
3 3

𝑆𝑢𝑚 𝑜𝑓 𝐶𝑜𝑒𝑓𝑓𝑖𝑐𝑖𝑒𝑛𝑡𝑠 = −2
1 + 𝑎 + 𝑏 + 𝑐 = −2
1 + 6 + 𝑏 + 2 = −2
𝑏 = −11

P a g e 63 | 88
Aziz Manva ([email protected])

D. Manipulating Expressions

Example 8.9
Consider the equation
𝑥 3 + 3𝑥 2 − 6𝑥 − 8 = 0, 𝑥 ∈ {𝛼, 𝛽, 𝛾}
A. Find the roots of the equation.
B. Find (𝛼 + 𝛽 − 𝛾)(𝛼 + 𝛾 − 𝛽)(𝛼 + 𝛽 − 𝛾)

Roots are
𝑥 ∈ {−1,2,4}

(𝛼 + 𝛽 − 𝛾)(𝛼 + 𝛾 − 𝛽)(𝛼 + 𝛽 − 𝛾) = 35

8.10: Identity
For a cubic equation with roots {𝛼, 𝛽, 𝛾}:
𝑠 𝑠 𝑠
(𝛼 + 𝛽 − 𝛾)(𝛼 + 𝛾 − 𝛽)(𝛽 + 𝛾 − 𝛼) = 8 ( − 𝛾) ( − 𝛽) ( − 𝛼)
2 2 2
Where:
𝑠 = 𝑠𝑢𝑚 𝑜𝑓 𝑟𝑜𝑜𝑡𝑠 = 𝛼 + 𝛽 + 𝛾

In the first term, add and subtract 𝛾:


𝛼+𝛽⏟+𝛾 − 𝛾 − 𝛾 = 𝛼 + 𝛽 + 𝛾 − 2𝛾 = 𝑠 − 2𝛾
𝐴𝑑𝑑 𝑎𝑛𝑑
𝑆𝑢𝑏𝑡𝑟𝑎𝑐𝑡
𝛼 + 𝛽 + 𝛾 − 𝛽 − 𝛽 = 𝛼 + 𝛽 + 𝛾 − 2𝛽 = 𝑠 − 2𝛽
𝛽 + 𝛾 + 𝛼 − 𝛼 − 𝛼 = 𝛽 + 𝛾 + 𝛼 − 2𝛼 = 𝑠 − 2𝛼

Hence:
(𝛼 + 𝛽 − 𝛾)(𝛼 + 𝛾 − 𝛽)(𝛽 + 𝛾 − 𝛼) = (𝑠 − 2𝛾)(𝑠 − 2𝛽)(𝑠 − 2𝛼)
Factor out 2 from each bracket:
𝑠 𝑠 𝑠
8 ( − 𝛾) ( − 𝛽) ( − 𝛼)
2 2 2

The method in the previous example worked only because the equation was “nice”, and we got integer roots. If
the roots are irrational, or complex, then this method would be much, much worse to work with.

This identity lets us find the value of the expression without finding the roots.

(Continuation) Example 8.11


Find the value of (𝛼 + 𝛽 − 𝛾)(𝛼 + 𝛾 − 𝛽)(𝛼 + 𝛽 − 𝛾) using the property above for the equation
𝑥 3 + 3𝑥 2 − 6𝑥 − 8 = 0, 𝑥 ∈ {𝛼, 𝛽, 𝛾}

𝑥 3 + 3𝑥 2 − 6𝑥 − 8 = 0 ⇔ (𝑥 − 𝛼)(𝑥 − 𝛽)(𝑥 − 𝛾)
E. Sum of Squares of Roots

8.12: Identity
𝛼 2 + 𝛽 2 + 𝛾 2 = (𝛼 + 𝛽 + 𝛾)2 − 2(𝛼𝛽 + 𝛽𝛾 + 𝛾𝛼)

P a g e 64 | 88
Aziz Manva ([email protected])

Example 8.13
Find the sum of the squares of the roots of the equation:
2𝑥 3 + 3𝑥 2 − 5𝑥 + 4 = 0

𝑏 2 𝑐 3 2 5 9 29
𝛼 2 + 𝛽 2 + 𝛾 2 = (𝛼 + 𝛽 + 𝛾)2 − 2(𝛼𝛽 + 𝛽𝛾 + 𝛾𝛼) = (− ) − 2 ( ) = (− ) − 2 (− ) = + 5 =
𝑎 𝑎 2 2 4 4

Example 8.14
Find the sum of the squares of the roots of the equation:
𝑥3 + 𝑥2 + 𝑥 + 1 = 0

𝑏 2 𝑐 1 2 1
𝛼 2 + 𝛽 2 + 𝛾 2 = (𝛼 + 𝛽 + 𝛾)2 − 2(𝛼𝛽 + 𝛽𝛾 + 𝛾𝛼) = (− ) − 2 ( ) = (− ) − 2 ( ) = 1 − 2 = −1
𝑎 𝑎 1 1

(Continuation) Example 8.15


𝐹𝑖𝑛𝑑 𝑡ℎ𝑒 𝑓𝑙𝑎𝑤 𝑖𝑛 𝑡ℎ𝑖𝑠 𝑠𝑜𝑙𝑢𝑡𝑖𝑜𝑛 𝑡𝑜 𝑡ℎ𝑒 𝑎𝑏𝑜𝑣𝑒 𝑞𝑢𝑒𝑠𝑡𝑖𝑜𝑛
Since 𝛼 is a root of the equation, it must satisfy the equation, and hence substitute 𝛼 in the original equation:
𝛼3 + 𝛼2 + 𝛼 + 1 = 0
2
Factor out 𝛼 from the first two terms:
𝛼 2 (𝛼 + 1) + 𝛼 + 1 = 0
Begin to isolate 𝛼 2 :
𝛼 2 (𝛼 + 1) = −(𝛼 + 1)
Isolate 𝛼 2 by dividing both sides by 𝛼 + 1:
𝛼+1
𝛼2 = − = −1
𝛼+1
Similarly,
𝛽 2 = −1, 𝛾 2 = −1

Hence,
𝛼 2 + 𝛽 2 + 𝛾 2 = −1 − 1 − 1 = −3

When you arrive at a solution using algebraic manipulation, rather than standard methods you need to be extra
careful. Each step of the solution should be valid.

When dividing by 𝛼 + 1, we need to add the condition that 𝛼 + 1 ≠ 0 ⇒ 𝛼 ≠ −1.

When check, we see that 𝑎 = −1 does satisfy the equation.


Also, if 𝛼 ≠ −1, then
𝛼 2 = −1 ⇒ α = √−1

Hence, the equation has one real root and two complex roots:
𝛼 = −1, 𝛽 2 = 𝛾 2 = −1

And hence
𝛼 2 + 𝛽 2 + 𝛾 2 = (−1)2 − 1 − 1 = 1 − 1 − 1 = −1

Example 8.16
If 𝛼, 𝛽, 𝛾 are the roots of the equation 𝑥 3 + 𝑥 2 − 𝑥 + 1 = 0, then show that

P a g e 65 | 88
Aziz Manva ([email protected])

𝛼−1 𝛽−1 𝛾−1


𝛼2 = , 𝛽2 = , 𝛾2 =
𝛼+1 𝛽+1 𝛾+1

𝛼3 + 𝛼2 − 𝛼 + 1 = 0
𝛼 2 (𝛼 + 1) = 𝛼 − 1
For 𝛼 ≠ −1:
𝛼−1
𝛼2 =
𝛼+1
Similarly,
𝛽−1 𝛾−1
𝛽2 = , 𝛾2 =
𝛽+1 𝛾+1

(Continuation) Example 8.17


If 𝛼, 𝛽, 𝛾 are the roots of the equation 𝑥 3 + 𝑥 2 − 𝑥 + 1 = 0, then find:
𝛼−1 𝛽−1 𝛾−1
+ +
𝛼+1 𝛽+1 𝛾+1

𝑏 2 𝑐 1 2 1
𝛼 2 + 𝛽 2 + 𝛾 2 = (𝛼 + 𝛽 + 𝛾)2 − 2(𝛼𝛽 + 𝛽𝛾 + 𝛾𝛼) = (− ) − 2 ( ) = (− ) − 2 (− ) = 1 + 2 = 3
𝑎 𝑎 1 1

F. Maximum and Minimum

Example 8.18
The polynomial 𝑥 3 − 𝑎𝑥 2 + 𝑏𝑥 − 2010 has three positive integer roots. What is the smallest possible value of 𝑎?
(AMC 10A 2010/21)

The sum of the roots is


−𝑎
− =𝑎
1
And the product of the roots is
−2010
− = 2010 = 2 × 3 × 5 × 67
1
To get three roots, any two of the prime factors must be multiplied together.
To make the sum minimum, the difference between the roots should be the least.
Hence, we get the roots as
5, ⏟
6 , 67
2×3
And the value of a is:
5 + 6 + 67

G. Answers in terms of Variables

Example 8.19
𝑎𝑥 3 + 𝑏𝑥 2 + 𝑐𝑥 + 𝑑 = 0 has one root −1. Find the product of the other roots in terms of the coefficients.

Without loss of generality, let 𝛾 = −1. Then:


𝑏 𝑏 𝑏
𝛼+𝛽+𝛾 = − ⇒ 𝛼+𝛽−1= − ⇒ 𝛼+𝛽 =− +1
𝑎 𝑎 𝑎

P a g e 66 | 88
Aziz Manva ([email protected])

𝑐 𝑐 𝑐 𝑐 𝑏 𝑐−𝑏+𝑎
𝛼𝛽 + 𝛽𝛾 + 𝛾𝛼 = ⇒ 𝛼𝛽 + 𝛽(−1) + (−1)𝛼 = ⇒ 𝛼𝛽 = + (𝛼 + 𝛽) = − + 1 =
𝑎 𝑎 𝑎 𝑎 𝑎 𝑎

H. Equations with Related Roots

8.2 Roots meeting conditions


A. Integers Roots

Example 8.20
Let 𝑎, 𝑏 be integers such that all the roots of the equation (𝑥 2 + 𝑎𝑥 + 20)(𝑥 2 + 17𝑥 + 𝑏) = 0 are negative
integers. What is the smallest possible value of 𝑎 + 𝑏? (PRMO 2017/4)

First Quadratic: 𝒙𝟐 + 𝒂𝒙 + 𝟐𝟎
𝑃𝑟𝑜𝑑𝑢𝑐𝑡 𝑜𝑓 𝑅𝑜𝑜𝑡𝑠 = 𝛼𝛽 = 20
𝑆𝑢𝑚 𝑜𝑓 𝑅𝑜𝑜𝑡𝑠 = 𝛼 + 𝛽 = −𝑎

−1 −2 −4
−20 −10 −5
−𝑎 −21 −12 −9
𝑎 21 12 9
Smallest

Second Quadratic: 𝒙𝟐 + 𝟏𝟕𝒙 + 𝒃


𝑃𝑟𝑜𝑑𝑢𝑐𝑡 𝑜𝑓 𝑅𝑜𝑜𝑡𝑠 = 𝛼𝛽 = 𝑏
𝑆𝑢𝑚 𝑜𝑓 𝑅𝑜𝑜𝑡𝑠 = 𝑎 + 𝛽 = −17

−16 −15 . . . −9
−1 −2 . . . −8
𝑏 16 30 . . . 72
Smallest Largest

B. Consecutive Integers

Example 8.21
𝑥 3 + 15𝑥 2 + 71𝑥 + 105 has roots which are consecutive odd integers. Find the roots.

Let the middle root be 𝛼. Then, the three roots are:


𝛼 − 2, 𝛼, 𝛼 + 2

Also, we have:
𝑆𝑢𝑚 𝑜𝑓 𝑅𝑜𝑜𝑡𝑠 = −15
(𝛼 − 2) + 𝛼 + (𝛼 + 2) = −15
3𝛼 = −15 ⇒ 𝛼 = −5
Roots are:
−3, −5, −7

P a g e 67 | 88
Aziz Manva ([email protected])

C. Consecutive Odd Integers


D. Consecutive Even Integers

E. Roots in Arithmetic Sequence

8.22: Arithmetic Sequence


An arithmetic sequence is defined as a sequence where the difference between two terms is constant. It is given
by:
… , 𝛼 − 2𝑑, 𝛼 − 𝑑, 𝛼, 𝛼 + 𝑑, 𝛼 + 2𝑑, …

Example 8.23
𝑎𝑥 3 − 6𝑥 2 + 𝑏𝑥 − 𝑐 has three distinct real roots in arithmetic progression with common difference 𝑑. (You can
use information from previous parts of the question in the following parts).
A. Find the middle root in terms of 𝑎. (Middle root is the root that is neither the largest nor the smallest).
B. Given that 𝑎 = 1, find the value of the middle root.
C. Given that 𝑑 = 1, find the other two roots.
D. Find the values of 𝑏 and c

Part A
Let the middle root be 𝛼. Then, the three roots are:
𝛼 − 𝑑, 𝛼, 𝛼 + 𝑑
Also, we have:
(−6)
𝑆𝑢𝑚 𝑜𝑓 𝑅𝑜𝑜𝑡𝑠 = −
𝑎
6
(𝛼 − 𝑑) + 𝛼 + (𝛼 + 𝑑) =
𝑎
6 2
3𝛼 = ⇒ 𝛼 =
𝑎 𝑎
Part B
2
𝑎=2⇒ 𝛼= =2
1
Part C
𝛼−𝑑 =2−1 = 1
𝛼+𝑑 =2+1 = 3
Part D
The roots are
1,2,3
And the equation will be
𝑠(𝑥 − 1)(𝑥 − 2)(𝑥 − 3) = 𝑠(𝑥 3 − 6𝑥 2 + 11𝑥 − 6) = 𝑠𝑥 3 − 6𝑠𝑥 2 + 11𝑠𝑥 − 6𝑠

And we know that the equation is


𝑥 3 − 6𝑥 2 + 𝑏𝑥 − 𝑐 ⇒ 𝑠 = 1

Hence, the equation is:


𝑥 3 − 6𝑥 2 + 11𝑥 − 6 ⇒ 𝑏 = 11, 𝑐 = 6

P a g e 68 | 88
Aziz Manva ([email protected])

8.24: Roots in Arithmetic Sequence


If the roots of 𝑎𝑥 3 + 𝑏𝑥 2 + 𝑐𝑥 + 𝑑 = 0 are in arithmetic sequence, then the value of the middle root:
𝑏
𝛼=−
3𝑎

Let the middle root be 𝛼, and the common difference of the arithmetic sequence be 𝑑. Then, the three roots are:
𝛼 − 𝑑, 𝛼, 𝛼 + 𝑑

Sum of Roots
𝑏 𝑏 𝑏
(𝛼 − 𝑑) + 𝛼 + (𝛼 + 𝑑) = − ⇒ 3𝛼 = − ⇒ 𝛼 = −
𝑎 𝑎 3𝑎

Example 8.25
The roots of 𝑥 4 − 14𝑥 3 + 51𝑥 2 + 𝑎𝑥 + 𝑏 = 0 form an arithmetic sequence. Find 𝑎 + 𝑏. (AOPS Alcumus,
Intermediate Algebra, Vieta's Formulas)

7
(𝑎 − 3𝑑) + (𝑎 − 𝑑) + (𝑎 + 𝑑) + (𝑎 + 3𝑑) = 14 ⇒ 𝑎 =
2

𝛼𝛽 + 𝛼𝛾 + 𝛼𝛿 + 𝛽𝛾 + 𝛽𝛿 + 𝛾𝛿 = 𝛼(𝛽 + 𝛾 + 𝛿) + 𝛽(𝛾 + 𝛿) + 𝛾𝛿

Substitute the values of the roots:


(𝑎 − 3𝑑)(𝑎 − 𝑑 + 𝑎 + 𝑑 + 𝑎 + 3𝑑) + (𝑎 − 𝑑)(𝑎 + 𝑑 + 𝑎 + 3𝑑) + (𝑎 + 𝑑)(𝑎 + 3𝑑)
Simplify:
= (𝑎 − 3𝑑)(3)(𝑎 + 𝑑) + (𝑎 − 𝑑)(2𝑎 + 4𝑑) + (𝑎 + 𝑑)(𝑎 + 3𝑑)

Take 𝑎 + 𝑑 common from the first and the last term:


= (𝑎 + 𝑑)[(3𝑎 − 9𝑑) + 𝑎 + 3𝑑] + (𝑎 − 𝑑)(2𝑎 + 4𝑑)
= (𝑎 + 𝑑)[(4𝑎 − 6𝑑)] + (𝑎 − 𝑑)(2𝑎 + 4𝑑)

Expand:
= 4𝑎2 − 6𝑎𝑑 + 4𝑎𝑑 − 6𝑑2 + 2𝑎2 − 2𝑎𝑑 + 4𝑎𝑑 − 4𝑑 2
= 6𝑎2 − 10𝑑2

6𝑎2 − 10𝑑2 = 51
49 147 102 45
10𝑑2 = 6𝑎2 − 51 = 6 ( ) − 51 = − =
4 2 2 2
2
45 9
𝑑 = =
20 4
3
𝑑=±
2

(𝑥 + 1)(𝑥 − 2)(𝑥 − 5)(𝑥 − 8) = 𝑥 4 − 14𝑥 3 + 51𝑥 2 − 14𝑥 − 80


𝑎 + 𝑏 = −94

F. Roots in Geometric Sequence

8.26: Geometric Sequence


An geometric sequence is defined as a sequence where the ratio between two terms is constant. It is given by:

P a g e 69 | 88
Aziz Manva ([email protected])

𝛼 𝛼
…, , , 𝛼, 𝛼𝑟, 𝛼𝑟 2 , …
𝑟2 𝑟

8.27: Roots in Geometric Sequence


If the roots of 𝑎𝑥 3 + 𝑏𝑥 2 + 𝑐𝑥 + 𝑑 = 0 are in geometric sequence, then the value of the middle root:
3 𝑑
𝛼 = √−
𝑎

Let the roots be:


𝛼
, 𝛼, 𝛼𝑟
𝑟

𝑑
𝑃𝑟𝑜𝑑𝑢𝑐𝑡 𝑜𝑓 𝑅𝑜𝑜𝑡𝑠 = −
𝑎
𝛼 𝑑 𝑑 3 𝑑
× 𝛼 × 𝛼𝑟 = − ⇒ 𝛼 3 = − ⇒ 𝛼 = √−
𝑟 𝑎 𝑎 𝑎

8.3 Vieta’s Formulas: Higher Degree


A. Relations between Roots

8.28: Relation between Roots


The fourth-degree equation
𝑎𝑥 4 + 𝑏𝑥 3 + 𝑐𝑥 2 + 𝑑𝑥 + 𝑒 = 0 𝑤𝑖𝑡ℎ 𝑅𝑜𝑜𝑡𝑠 ∈ {𝛼, 𝛽, 𝛾, 𝛿}

𝑏
𝑆𝑢𝑚 𝑜𝑓 𝑅𝑜𝑜𝑡𝑠 = 𝛼 + 𝛽 + 𝛾 + 𝛿 = −
𝑎
𝑐
2 𝑅𝑜𝑜𝑡𝑠 𝑎𝑡 𝑎 𝑡𝑖𝑚𝑒 = 𝛼𝛽 + 𝛼𝛾 + 𝛼𝛿 + 𝛽𝛾 + 𝛽𝛿 + 𝛾𝛿 =
𝑎
𝑑
3 𝑅𝑜𝑜𝑡𝑠 𝑎𝑡 𝑎 𝑡𝑖𝑚𝑒 = 𝛼𝛽𝛾 + 𝛼𝛽𝛿 + 𝛼𝛾𝛿 + 𝛽𝛾𝛿 = −
𝑎
𝑒
4 𝑅𝑜𝑜𝑡𝑠 𝑎𝑡 𝑎 𝑡𝑖𝑚𝑒 = 𝛼𝛽𝛾𝛿 =
𝑎

Example 8.29
The fourth-degree polynomial equation 𝑥 4 − 7𝑥 3 + 4𝑥 2 + 7𝑥 − 4 = 0 has four real roots, 𝑎, 𝑏, 𝑐 and 𝑑. What is
1 1 1 1
the value of the sum 𝑎 + 𝑏 + 𝑐 + 𝑑? Express your answer as a common fraction. (MathCounts 2014 State Sprint)

1 1 1 1 𝑎𝑏𝑐 + 𝑎𝑏𝑑 + 𝑎𝑐𝑑 + 𝑏𝑐𝑑 7 7


+ + + = =− =
𝑎 𝑏 𝑐 𝑑 𝑎𝑏𝑐𝑑 −4 4

B. Using the Relations

Example 8.30
Suppose 1,2,3 are the roots of the equation 𝑥 4 + 𝑎𝑥 2 + 𝑏𝑥 = 𝑐. Find the value of 𝑐. (PRMO 2017/19)

This is a fourth-degree equation. Hence, it must have four roots. We have been given three roots, and we assume
a variable for the fourth root, making the roots

P a g e 70 | 88
Aziz Manva ([email protected])

1,2,3, 𝛼

We can rewrite the given equation as


𝑥 4 + 0𝑥 3 + 𝑎𝑥 2 + 𝑏𝑥 − 𝑐 = 0

Sum of Roots
0
𝑆𝑢𝑚 𝑜𝑓 𝑅𝑜𝑜𝑡𝑠 = − = 0
1
1 + 2 + 3 + 𝛼 = 0 ⇒ 𝛼 = −6

Product of Roots
𝑐
𝑃𝑟𝑜𝑑𝑢𝑐𝑡 𝑜𝑓 𝑅𝑜𝑜𝑡𝑠 = − = −𝑐
1
𝑃𝑟𝑜𝑑𝑢𝑐𝑡 𝑜𝑓 𝑅𝑜𝑜𝑡𝑠 = (1)(2)(3)(−6) = −36

−𝑐 = −36 ⇒ 𝑐 = 36
C. Graphs

Example 8.31
The graph below shows a portion of the curve defined by the
quartic polynomial 𝑃(𝑥) = 𝑥 4 + 𝑎𝑥 3 + 𝑏𝑥 2 + 𝑐𝑥 + 𝑑. Which
of the following is the smallest? (AMC 12 2000/22)
A. 𝑃(−1)
B. The product of the zeroes of P
C. The product of the non-real zeroes of O
D. The sum of the coefficients of P
E. The sum of the real zeroes of P

This looks difficult, but estimating using properties is the


key skill here.
Option A
From the graph, we can make out that:
𝑃(−1) ≈ 4

Option B
𝑑
𝑃𝑟𝑜𝑑𝑢𝑐𝑡 𝑜𝑓 𝑟𝑜𝑜𝑡𝑠 = =𝑑
1
But, this is also the y-intercept
𝑃𝑟𝑜𝑑𝑢𝑐𝑡 𝑜𝑓 𝑟𝑜𝑜𝑡𝑠 ≈ 5.5
Option D
Sum of coefficients
= 𝑃(1) ≈ 3
Option E
Sum of real roots
≈ 1.5 + 3.5 = 5
Option C
We can see two real roots. We can also see all three turning points.
Hence, there are no more real roots, and the remaining roots are non-real (complex).
(𝐶𝑜𝑚𝑝𝑙𝑒𝑥 𝑅𝑜𝑜𝑡𝑠)(𝑅𝑒𝑎𝑙 𝑅𝑜𝑜𝑡𝑠) = 𝐴𝑙𝑙 𝑅𝑜𝑜𝑡𝑠

P a g e 71 | 88
Aziz Manva ([email protected])

𝐴𝑙𝑙 𝑅𝑜𝑜𝑡𝑠 5.5


𝐶𝑜𝑚𝑝𝑙𝑒𝑥 𝑅𝑜𝑜𝑡𝑠 = ≈ = 1. 𝑆𝑜𝑚𝑒𝑡ℎ𝑖𝑛𝑔
𝑅𝑒𝑎𝑙 𝑅𝑜𝑜𝑡𝑠 5
This is clearly the lowest.

D. Higher Degree Equations

8.32: Relation between Roots


𝑎1 𝑥 𝑛 + 𝑎2 𝑥 𝑛−1 + 𝑎3 𝑥 𝑛−2 + ⋯ + 𝑎𝑛+1 = 0
𝑎2
𝑆𝑢𝑚 𝑜𝑓 𝑅𝑜𝑜𝑡𝑠 = 𝑅𝑜𝑜𝑡𝑠 𝑡𝑎𝑘𝑒𝑛 1 𝑎𝑡 𝑎 𝑡𝑖𝑚𝑒 = −
𝑎1
𝑎3
𝑅𝑜𝑜𝑡𝑠 𝑡𝑎𝑘𝑒𝑛 2 𝑎𝑡 𝑎 𝑡𝑖𝑚𝑒 =
𝑎1
𝑎4
𝑅𝑜𝑜𝑡𝑠 𝑡𝑎𝑘𝑒𝑛 3 𝑎𝑡 𝑎 𝑡𝑖𝑚𝑒 = −
𝑎1
.
.
.
𝑎𝑛+1
𝑃𝑟𝑜𝑑𝑢𝑐𝑡 𝑜𝑓 𝑅𝑜𝑜𝑡𝑠 = (−1)𝑛
𝑎1

8.4 Transformation of Roots


A. Shifting

8.33: Roots increased/decreased by 𝒌


Consider the polynomial:
𝑓(𝑥) = 𝑎𝑛 𝑥 𝑛 + 𝑎𝑛−1 𝑥 𝑛−1 + ⋯ + 𝑎0 𝑤𝑖𝑡ℎ 𝑧𝑒𝑟𝑜𝑒𝑠 ∈ {𝑥1 , 𝑥2 , … , 𝑥𝑛 }

For some constant 𝑘, the polynomial with zeroes {𝑥1 + 𝑘, 𝑥2 + 𝑘, … , 𝑥𝑛 + 𝑘} is given by:
𝑓(𝑥 − 𝑘) = 𝑎𝑛 (𝑥 − 𝑘)𝑛 + 𝑎𝑛−1 (𝑥 − 𝑘)𝑛−1 + ⋯ + 𝑎0

𝛼+𝑘 = 𝑡 ⇒ 𝛼 = 𝑡−𝑘

8.34: Cube of a Binomial


(𝑎 + 𝑏)3 = 𝑎3 + 3𝑎2 𝑏 + 3𝑎𝑏 2 + 𝑏 3

Example 8.35
The roots of 2𝑥 3 + 4𝑥 2 − 7𝑥 + 9 are 𝛼, 𝛽 and 𝛾.
A. Find the equation with roots 𝛼 − 1, 𝛽 − 1, and 𝛾 − 1.
B. Use the new equation to find 𝛼 + 𝛽 + 𝛾 − 3
C. Use the new equation to find 𝛼𝛽 + 𝛽𝛾 + 𝛾𝛼 − 2(𝛼 + 𝛽 + 𝛾) + 3
D. Use the new equation to find (𝛼𝛽 − 𝛼 − 𝛽 + 1)(𝛾 − 1)

Part A
Use a change of variable. Let
𝛼−1=𝑡 ⇒𝛼 =𝑡+1

Substitute 𝑥 = 𝑡 + 1:

P a g e 72 | 88
Aziz Manva ([email protected])

2(𝑡 + 1)3 + 4(𝑡 + 1)2 − 7(𝑡 + 1) + 9 = 0


2(𝑡 3 + 3𝑡 2 + 3𝑡 + 1) + 4(𝑡 2 + 2𝑡 + 1) − 7𝑡 − 7 + 9 = 0
2𝑡 3 + 6𝑡 2 + 6𝑡 + 2 + 4𝑡 2 + 8𝑡 + 4 − 7𝑡 − 7 + 9 = 0
2𝑡 3 + 10𝑡 2 + 7𝑡 + 8 = 0
Part B
𝑏 10
𝛼 + 𝛽 + 𝛾 − 3 = (𝛼 − 1) + (𝛽 − 1) + (𝛾 − 1) = 𝑆𝑢𝑚 𝑜𝑓 𝑅𝑜𝑜𝑡𝑠 = − =− = −5
𝑎 2
Part C
𝛼𝛽 + 𝛽𝛾 + 𝛾𝛼 − 2𝛼 − 2𝛽 − 2𝛾 + 3
= 𝛼𝛽 − 𝛼 − 𝛽 + 1 + 𝛽𝛾 − 𝛽 − 𝛾 + 1 + 𝛾𝛼 − 𝛾 − 𝛼 + 1
= (𝛼 − 1)(𝛽 − 1) + (𝛽 − 1)( 𝛾 − 1) + ( 𝛾 − 1)(𝛼 − 1)
𝑐 7
= 𝑃𝑟𝑜𝑑𝑢𝑐𝑡 𝑜𝑓 𝑅𝑜𝑜𝑡𝑠 𝑡𝑎𝑘𝑒𝑛 2 𝑎𝑡 𝑎 𝑡𝑖𝑚𝑒 = =
𝑎 2

Part D
(𝛼𝛽 − 𝛼 − 𝛽 + 1)(𝛾 − 1)
𝑑 8
= (𝛼 − 1)(𝛽 − 1)(𝛾 − 1) = 𝑃𝑟𝑜𝑑𝑢𝑐𝑡 𝑜𝑓 𝑅𝑜𝑜𝑡𝑠 3 𝑎𝑡 𝑎 𝑡𝑖𝑚𝑒 = − = − = −4
𝑎 2

Example 8.36
The roots of 2𝑥 3 + 4𝑥 2 − 7𝑥 + 9 are 𝛼, 𝛽 and 𝛾.
A. Find the equation with roots 𝛼 + 2, 𝛽 + 2, and 𝛾 + 2.

Part A
Use a change of variable. Let
𝛼+2=𝑡 ⇒𝛼 =𝑡−2

Substitute 𝑥 = 𝑡 − 2:
2(𝑡 − 2)3 + 4(𝑡 − 2)2 − 7(𝑡 − 2) + 9 = 0
B. Scaling

8.37: Roots increased/decreased by 𝒌


Consider the polynomial:
𝑓(𝑥) = 𝑎𝑛 𝑥 𝑛 + 𝑎𝑛−1 𝑥 𝑛−1 + ⋯ + 𝑎0 𝑤𝑖𝑡ℎ 𝑧𝑒𝑟𝑜𝑒𝑠 ∈ {𝑥1 , 𝑥2 , … , 𝑥𝑛 }

For some constant 𝑘, the polynomial with zeroes {𝑘𝑥1 , 𝑘𝑥2 , … , 𝑘𝑥𝑛 } is given by:
𝑥 𝑛 𝑥 𝑛−1
𝑓(𝑥 − 𝑘) = 𝑎𝑛 ( ) + 𝑎𝑛−1 ( ) + ⋯ + 𝑎0
𝑘 𝑘

𝑡
𝑘𝛼 = 𝑡 ⇒ 𝛼 =
𝑘

Example 8.38
The roots of 2𝑥 3 − 3𝑥 2 + 4𝑥 + 7 = 0 are 𝛼, 𝛽 and 𝛾. Find the equation with roots 2𝛼, 2𝛽 and 2𝛾.

𝑡
2𝛼 = 𝑡 ⇒ 𝛼 =
2
𝑡 3 𝑡 2 𝑡
2( ) − 3( ) + 4( ) + 7 = 0
2 2 2

P a g e 73 | 88
Aziz Manva ([email protected])

𝑡 3 3𝑡
− + 2𝑡 + 7 = 0
4 4
𝑡 3 − 3𝑡 + 8𝑡 + 28 = 0

Example 8.39
𝑥3 𝑥2 𝑥 𝛼 𝛽 𝛾
The roots of 9
− 3
+ 27 + 4 = 0 are 𝛼, 𝛽 and 𝛾. Find the equation with roots 3 , 3 and 3.

𝛼
= 𝑡 ⇒ 𝛼 = 3𝑡
3

(3𝑡)3 (3𝑡)2 (3𝑡)


− + +4=0
9 3 27
𝑡
3𝑡 3 − 3𝑡 2 + + 4 = 0
9
27𝑡 3 − 27𝑡 2 + 𝑡 + 36 = 0
C. Reciprocals

8.40: Reciprocals
Consider the polynomial:
𝑓(𝑥) = 𝑎𝑛 𝑥 𝑛 + 𝑎𝑛−1 𝑥 𝑛−1 + ⋯ + 𝑎0 𝑤𝑖𝑡ℎ 𝑧𝑒𝑟𝑜𝑒𝑠 ∈ {𝑥1 , 𝑥2 , … , 𝑥𝑛 }

1 1 1
The polynomial with zeroes {𝑥 , 𝑥 , … , 𝑥 } is given by reversing its coefficients:
1 2 𝑛
1
𝑓 ( ) = 𝑎0 𝑥 𝑛 + 𝑎1 𝑥 𝑛−1 + ⋯ + 𝑎𝑛
𝑥

1 1 𝑛 1 𝑛−1
𝑓 ( ) = 𝑎𝑛 ( ) + 𝑎𝑛−1 ( ) + ⋯ + 𝑎0 = 0
𝑥 𝑥 𝑥
Multiply the last two parts by 𝑥 𝑛 :
𝑎0 𝑥 𝑛 + 𝑎1 𝑥 𝑛−1 + ⋯ + 𝑎𝑛 = 0

Example 8.41
1 1 1
The roots of 2𝑥 3 − 3𝑥 2 + 4𝑥 + 7 = 0 are 𝛼, 𝛽 and 𝛾. Find the equation with roots , and .
𝛼 𝛽 𝛾

1 1
=𝑡⇒𝛼=
𝛼 𝑡

1 3 1 2 1
2( ) − 3( ) + 4( ) + 7 = 0
𝑡 𝑡 𝑡
Multiply by 𝑡 3 :
2 − 3𝑡 + 4𝑡 2 + 7𝑡 3 = 0
7𝑡 3 + 4𝑡 2 − 3𝑡 + 2 = 0
Example 8.42
𝑥3 𝑥2 𝑥 1 1 1
The roots of 9
− 3
+ 27 + 4 are 𝛼, 𝛽 and 𝛾. Find the equation with roots 𝛼, 𝛽 and 𝛾.

D. Combinations

P a g e 74 | 88
Aziz Manva ([email protected])

Challenge 8.43
If 𝛼, 𝛽, 𝛾 are the roots of the equation 𝑥 3 + 𝑥 2 − 𝑥 + 1 = 0, then show that:
𝛼−1 𝑡+1
=𝑡⇒𝛼=−
𝛼+1 𝑡−1

𝛼 − 1 = 𝑡𝛼 + 𝑡
Collate all the 𝛼 terms on the RHS”
−𝑡 − 1 = 𝑡𝛼 − 𝛼
Factor 𝛼 on the RHS:
−𝑡 − 1 = 𝛼(𝑡 − 1)
Divide both sides by 𝑡 − 1:
−𝑡 − 1 𝑡+1
𝛼= =−
𝑡−1 𝑡−1

Find the equation with roots


𝛼−1 𝛽−1 𝛾−1
, ,
𝛼+1 𝛽+1 𝛾+1
Note: Pay careful attention to the algebra: signs, substitution, etc.

𝛼3 + 𝛼2 − 𝛼 + 1 = 0
𝑡+1
Substitute 𝛼 = − :
𝑡−1
𝑡+1 3 𝑡+1 2 𝑡+1
(− ) + (− ) − (− )+1=0
𝑡−1 𝑡−1 𝑡−1
Multiply both sides by (𝑡 − 1)3 :
[−(𝑡 + 1)]3 + [−(𝑡 + 1)]2 (𝑡 − 1) − [−(𝑡 + 1)](𝑡 − 1)2 + (𝑡 − 1)3 = 0

The first and the last terms can be simplified to:


(−)(𝑡 3 + 3𝑡 2 + 3𝑡 + 1) + (𝑡 3 − 3𝑡 2 + 3𝑡 − 1) = −6𝑡 2 − 2

The second and the third terms can be simplified to:


[(𝑡 + 1)]2 (𝑡 − 1) + [(𝑡 + 1)](𝑡 − 1)2
Take 𝑡 + 1 common:
(𝑡 + 1)[(𝑡 + 1)(𝑡 − 1) + (𝑡 − 1)2 ]
= (𝑡 + 1)[𝑡 2 − 1 + 𝑡 2 − 2𝑡 + 1]
= (𝑡 + 1)[2𝑡 2 − 2𝑡]
= 2𝑡 3 − 2𝑡 2 + 2𝑡 2 − 2𝑡
= 2𝑡 3 − 2𝑡

Combine all the terms together:


2𝑡 3 − 2𝑡 − 6𝑡 2 − 2 = 0
Divide by 2 and rewrite in standard form:
𝑡 3 − 3𝑡 2 − 𝑡 − 1 = 0

Find the value of


𝛼−1 𝛽−1 𝛾−1
+ +
𝛼+1 𝛽+1 𝛾+1

𝑡 3 − 3𝑡 2 − 𝑡 − 1 = 0
We want the sum of the roots, which is:

P a g e 75 | 88
Aziz Manva ([email protected])

𝑏 −3
− =− =3
𝑎 1

Challenge 8.44
If 𝛼, 𝛽, 𝛾 are the roots of the equation 𝑥 3 + 𝑥 2 − 𝑥 + 1 = 0, then find
𝛼−1 𝛽−1 𝛾−1
+ +
𝛼+1 𝛽+1 𝛾+1

𝑥 2 (𝑥 + 1) = 𝑥 − 1
𝑥−1
𝑥2 =
𝑥+1

𝛼−1 𝛽−1 𝛾−1


+ + = 𝛼 2 + 𝛽2 + 𝛾 2
𝛼+1 𝛽+1 𝛾+1

Use the identity (𝛼 + 𝛽 + 𝛾)2 − 2(𝛼𝛽 + 𝛽𝛾 + 𝛾𝛼) and we need to find:


𝑏 2 𝑐 1 2 1
(− ) − 2 ( ) = (− ) − 2 (− ) = 1 + 2 = 3
𝑎 𝑎 1 1

P a g e 76 | 88
Aziz Manva ([email protected])

9. MORE THEOREMS
9.1 Descartes’ Rule of Signs

9.2 Rational Inequalities: Wavy Curve Method


A. Basics

Example 9.1
𝑥 2 +5𝑥+6
Find the region where the expression 𝑥−1
is non-negative.

Write the condition that we need to meet as an inequality:


𝑥 2 + 5𝑥 + 6 (𝑥 + 3)(𝑥 + 2)
≥0⇒ ≥0
(𝑥 − 1) (𝑥 − 1)
We have now factored the numerator and denominator into a product of
linear expressions. We can use this to find the zeroes of the expression:
𝑥 + 3 = 0 ⇒ 𝑥 = −3
𝑥 + 2 ⇒ 𝑥 = −2
𝑥−1=0⇒𝑥 =1

The zeroes that we have found together comprise the critical points,
given by:
𝐶𝑟𝑖𝑡𝑖𝑐𝑎𝑙 𝑃𝑜𝑖𝑛𝑡𝑠 ∈ {−3, −2,1}

The critical points divide the real number line into four regions:
(−∞, −3) ∪ (−3, −2) ∪ (−2,1) ∪ (1, ∞)

𝑥 + 3 > 0 ⇒ 𝑥 > −3 (−∞, −3) (−3, −2) (−2,1) (1, ∞)


𝑥 + 2 > 0 ⇒ 𝑥 > −2 𝑥+3 −𝑣𝑒 +𝑣𝑒 +𝑣𝑒 +𝑣𝑒
𝑥−1>0⇒𝑥 >1 𝑥+2 −𝑣𝑒 −𝑣𝑒 +𝑣𝑒 +𝑣𝑒
𝑥−1 −𝑣𝑒 −𝑣𝑒 −𝑣𝑒 +𝑣𝑒
(−∞, −3) ∪ ⏟
(−𝟑, −𝟐) ∪ (−2,1) ∪ (𝟏,
⏟ ∞)
+𝒗𝒆 +𝒗𝒆 Result −𝑣𝑒 +𝑣𝑒 −𝑣𝑒 +𝑣𝑒
(−3, −2) ∪ (1,
⏟ ⏟ ∞)
+𝑣𝑒 +𝑣𝑒

𝐴𝑡 1, 𝑡ℎ𝑒 𝑑𝑒𝑛𝑜𝑚𝑖𝑛𝑎𝑡𝑜𝑟 𝑖𝑠 𝑛𝑜𝑡 𝑑𝑒𝑓𝑖𝑛𝑒𝑑


𝐴𝑡 − 3 𝑎𝑛𝑑 − 2, 𝑡ℎ𝑒 𝑒𝑥𝑝𝑟𝑒𝑠𝑠𝑖𝑜𝑛 𝑖𝑠 𝑧𝑒𝑟𝑜
[−3, −2] ∪ (1, ∞)

P a g e 77 | 88
Aziz Manva ([email protected])

PART V: IDENTITIES
10. IDENTITIES
10.1 Cube of a Sum/Difference
A. Revision

10.1: Square Formulas


(𝑎 + 𝑏)2 = 𝑎2 + 2𝑎𝑏 + 𝑏 2
(𝑎 − 𝑏)2 = 𝑎2 − 2𝑎𝑏 + 𝑏 2
(𝑎 + 𝑏)(𝑎 − 𝑏) = 𝑎2 − 𝑏 2

B. Cube of a Sum/Difference

10.2: Cube of a Sum/Difference


(𝑎 + 𝑏)3 = 𝑎3 + 3𝑎2 𝑏 + 3𝑎𝑏 2 + 𝑏 3 = 𝑎3 + 𝑏 3 + 3𝑎𝑏(𝑎 + 𝑏)
(𝑎 − 𝑏)3 = 𝑎3 − 3𝑎2 𝑏 + 3𝑎𝑏 2 − 𝑏 3 = 𝑎3 − 𝑏 3 + 3𝑎𝑏(−𝑎 + 𝑏)

Example 10.3
Expand:
A. (𝑥 + 𝑦)3
B. (𝑗 − 𝑘)3
1 1 3
C. (2 𝑝 + 3 𝑞)
2 3 3
D. (3 𝑚 + 2 𝑛)
1 3
E. (5 𝑑 + 5𝑒)

𝑥 3 + 3𝑥 2 𝑦 + 3𝑥𝑦 2 + 𝑦 3
𝑗 3 − 3𝑗 2 𝑘 + 3𝑗𝑘 2 − 𝑘 3
1 3 1 1 1 1 1 1 1 1 1
𝑝 + 3 ( 𝑝2 ) ( 𝑞) + 3 ( 𝑝) ( 𝑞 2 ) + 𝑞 3 = 𝑝3 + 𝑝2 𝑞 + 𝑝𝑞 2 + 𝑞 3
8 4 3 2 9 27 8 4 6 27
8 3 4 2 3 2 9 2 27 3 8 3 9 27
𝑚 + 3 ( 𝑚 ) ( 𝑛) + 3 ( 𝑚) ( 𝑛 ) + 𝑛 = 𝑚 + 2𝑚2 𝑛 + 𝑚𝑛2 + 𝑛3
27 9 2 3 4 8 27 2 8
1 3 1 2 1 1 3
𝑑 + 3 ( 𝑑 ) (5𝑒) + 3 ( 𝑑) (25𝑒 2 ) + 125𝑒 3 = 𝑑3 + 𝑑2 𝑒 + 15𝑑𝑒 2 + 125𝑒 3
125 25 5 125 5

Example 10.4
Expand
(0.3𝑥 − 0.2𝑒)3

3 2 9 3 3 27
( ) = = 0.09, ( ) = = 0.027
10 100 10 1000
2 2 4 2 3 8
( ) = = 0.04, ( ) = = 0.008
10 100 10 1000
0.027𝑥 3 − 3(0.09𝑥 2 )(0.2𝑒) + 3(0.3𝑥)(0.04𝑒 2 ) − 0.008𝑒 3
= 0.027𝑥 3 − 0.054𝑥 2 𝑒 + 0.036𝑥𝑒 2 − 0.008𝑒 3

P a g e 78 | 88
Aziz Manva ([email protected])

Example 10.5
Expand:
2 3 3
( 𝑥2 − )
3 4𝑥

8 6 4 3 2 9 27
𝑥 − 3 ( 𝑥4) ( ) + 3 ( 𝑥2) ( 2
)−
27 9 4𝑥 3 16𝑥 64𝑥 3
8 6 9 27
= 𝑥 − 𝑥3 + −
27 8 64𝑥 3
C. Radicals

Example 10.6
Expand:
3
(√𝑎 + √𝑏)

3 3 3 3
𝑎2 + 3𝑎√𝑏 + 3√𝑎𝑏 + 𝑏 2 = 𝑎2 + 𝑏 2 + 3√𝑎𝑏(√𝑎 + √𝑏)

Example 10.7
Given that √𝑎 + √𝑏 = 1, find the value of √𝑎𝑏 in terms of 𝑎 and 𝑏.

3
(√𝑎 + √𝑏) = 13
3 3
𝑎2 + 𝑏 2 + 3√𝑎𝑏(√𝑎 + √𝑏) = 1
3 3
𝑎2 + 𝑏 2 + 3√𝑎𝑏 = 1
3 3
3√𝑎𝑏 = 1 − 𝑎2 − 𝑏 2
3 3
1 − 𝑎2 − 𝑏 2
√𝑎𝑏 =
3
D. Factorization

Example 10.8
Evaluate:
1.331 + 0.363 + 0.0033 + 0.001
1.44

1.331 + 0.363 + 0.0033 + 0.001 (1.1 + 0.1)3 1.23


= = = 1.2
1.44 1.22 1.22

Example 10.9
Solve:
𝑥 3 + 3𝑥 2 + 3𝑥 + 1 = 0

(𝑥 + 1)3 = 0 ⇒ 𝑥 + 1 = 0 ⇒ 𝑥 = −1

Example 10.10
Find the ratio between 𝑎 and 𝑏 given that:

P a g e 79 | 88
Aziz Manva ([email protected])

8 3 2 2 1 1
𝑎 + 𝑎 𝑏 + 𝑎𝑏 + 𝑏 3 = 0
27 9 6 8

2 1 3 2 1 2 1 𝑎 3
( 𝑎 + 𝑏) = 0 ⇒ 𝑎 + 𝑏 = 0 ⇒ 𝑎 = − 𝑏 ⇒ = −
3 2 3 2 3 2 𝑏 4

10.2 Sum/Difference of Cubes


A. Sum of Cubes

10.11: Sum and Difference of Cubes


𝑎3 + 𝑏 3 = (𝑎 + 𝑏)(𝑎2 − 𝑎𝑏 + 𝑏 2 )
𝑎3 − 𝑏 3 = (𝑎 − 𝑏)(𝑎2 + 𝑎𝑏 + 𝑏 2 )

(𝑎 + 𝑏)(𝑎2 − 𝑎𝑏 + 𝑏 2 ) = 𝑎3 − 𝑎2 𝑏 + 𝑎𝑏 2 + 𝑎2 𝑏 − 𝑎𝑏 2 + 𝑏 3 = 𝑎3 + 𝑏 3
(𝑎 − 𝑏)(𝑎2 + 𝑎𝑏 + 𝑏 2 ) = 𝑎3 + 𝑎2 𝑏 + 𝑎𝑏 2 − 𝑎2 𝑏 − 𝑎𝑏 2 − 𝑏 3 = 𝑎3 − 𝑏 3

Example 10.12
Factorize
A. 𝑥 3 + 𝑦 3
B. 8𝑝3 + 27𝑞 3
C. 64𝑤 3 + 125𝑥 3
64 6 216
D. 729
𝑟 + 125 𝑠 9
E. 3𝑥 + 5𝑦 3
3

F. 27𝑎3 − 8𝑏 6
1 3 125
G. 8
𝑥 + 729 𝑦 3
H. 9𝑎 − 27𝑏 6
3

𝑥 3 + 𝑦 3 = (𝑥 + 𝑦)(𝑥 2 − 𝑥𝑦 + 𝑦 2 )

𝑎3 = 8𝑝3 ⇒ 𝑎 = 2𝑝
𝑏 3 = 27𝑞3 ⇒ 𝑏 = 3𝑞
(2𝑝)3 + (3𝑞)3 = (2𝑝 + 3𝑞)(4𝑝2 − 6𝑝𝑞 + 9𝑞 2 )

(4𝑤)3 + (5𝑥)3 = (4𝑤 + 5𝑥)(16𝑤 2 − 20𝑥𝑤 + 25𝑥 2 )

3 3
4 6 4 6 16 8 36
( 𝑟2) + ( 𝑠3) = ( 𝑟2 + 𝑠3) ( 𝑟4 − 𝑟2𝑠3 + 𝑠6)
9 5 9 5 81 15 25

3 3 3 3 3 3
2 3
2
(√3𝑥) + (√5𝑦) = (√3𝑥 + √5𝑦) (33 𝑥 2 − √15𝑥𝑦 + 53 𝑦 2 )

27𝑎3 − 8𝑏 6 = (3𝑎 − 2𝑏 2 )(9𝑎2 + 6𝑎𝑏 2 + 4𝑏 4 )


1 3 125 3 1 5 1 5 25
𝑥 + 𝑦 = ( 𝑥 + 𝑦) ( 𝑥 2 − 𝑥𝑦 + 𝑦 2 )
8 729 2 9 4 18 81
3 3
2
9𝑎3 − 27𝑏 6 = 9(𝑎3 − 3𝑏 6 ) = 9(𝑎 − √3𝑏 2 ) (𝑎2 + √3𝑎𝑏 2 + 33 𝑏 4 )

P a g e 80 | 88
Aziz Manva ([email protected])

Example 10.13
Factor:
A. 0.33 + 0.43
B. 0.43 + 0.63

0.33 + 0.43 = (0.3 + 0.4)(0.32 − 0.12 + 0.42 )


0.43 + 0.63 = (0.4 + 0.6)(0.42 − 2.4 + 0.62 ) = (1)(0.16 − 2.4 + 0.36) = 0.52 − 2.4 = −1.88

Example 10.14
0.23 + 0.33
0.5

0.23 + 0.33 (0.2 + 0.3)(0.22 − 0.06 + 0.32 ) (0.5)(0.04 − 0.06 + 0.09)


= = = 0.07
0.5 0.5 0.5

0.23 + 0.33 0.008 + 0.027 0.035


= = = 0.07
0.5 0.5 0.5

Example 10.15
0.63 + 0.73
1.3

0.63 + 0.73 (0.6 + 0.7)(0.62 − 0.42 + 0.72 ) (1.3)(0.36 − 0.42 + 0.49)


= = = 0.43
1.3 1.3 1.3

0.63 + 0.73 0.216 + 0.343 0.559


= = = 0.43
1.3 1.3 1.3
A. Back Calculations

Example 10.16
Suppose 𝑓(𝑥) = 𝑥 2 , and 𝑔(𝑥) is a polynomial such that 𝑓(𝑔(𝑥)) = 4𝑥 2 + 4𝑥 + 1. What are the possible values of
𝑔(𝑥)? (MathCounts 1993 Chapter Countdown)

[𝑔(𝑥)]2 = 4𝑥 2 + 4𝑥 + 1
Take square roots both sides:
𝑔(𝑥) = ±√4𝑥 2 + 4𝑥 + 1 = ±√(2𝑥 + 1)2 = ±(2𝑥 + 1)

10.3 Trinomial Identities


A. Perfect Square of a Trinomial

10.17: Perfect Square of a Trinomial


(𝑎 + 𝑏 + 𝑐)2 = 𝑎2 + 𝑏 2 + 𝑐 2 + 2(𝑎𝑏 + 𝑎𝑐 + 𝑐𝑏)

Group the first two terms:


((𝑎 + 𝑏) + 𝑐)((𝑎 + 𝑏) + 𝑐)

P a g e 81 | 88
Aziz Manva ([email protected])

Apply (𝑥 + 𝑦)2 = 𝑥 2 + 2𝑥𝑦 + 𝑦 2 :


= (𝑎 + 𝑏)2 + 2(𝑐)(𝑎 + 𝑏) + 𝑐 2
Expand:
= 𝑎2 + 2𝑎𝑏 + 𝑏 2 + 2𝑎𝑐 + 2𝑐𝑏 + 𝑐 2
Rearrange and Factor:
= 𝑎2 + 𝑏 2 + 𝑐 2 + 2(𝑎𝑏 + 𝑎𝑐 + 𝑐𝑏)

Example 10.18
Expand:
A. (𝑎 + 2𝑏 + 3𝑐)2
1 2 3 2
B. ( 𝑥 + 𝑦 − 𝑧)
2 3 4
C. (0.1𝑝 − 0.2𝑞 + 0.3𝑟)2

(𝑎 + 2𝑏 + 3𝑐)2 = 𝑎2 + 4𝑏 2 + 9𝑐 2 + 2(2𝑎𝑏 + 3𝑎𝑐 + 6𝑏𝑐) = 𝑎2 + 4𝑏 2 + 9𝑐 2 + 4𝑎𝑏 + 6𝑎𝑐 + 12𝑏𝑐


1 2 3 2 1 2 4 2 9 1 3 1
( 𝑥 + 𝑦 − 𝑧) = 𝑥 + 𝑦 + 𝑧 2 + 2 ( 𝑥𝑦 − 𝑥𝑧 − 𝑦𝑧)
2 3 4 4 9 16 3 8 2
(0.1𝑝 − 0.2𝑞 + 0.3𝑟)2 = 0.01𝑥 2 + 0.04𝑞2 + 0.09𝑟 2 + 2(−0.02𝑝𝑞 + 0.03𝑝𝑟 − 0.06𝑞𝑟)

Example 10.19
1 2 3
Given that 2 𝑥 + 3 𝑦 − 4 𝑧 = 0, we can determine the value of 36𝑥 2 + 64𝑦 2 + 81𝑧 2 in the form 𝑎𝑥𝑦 + 𝑏𝑦𝑧 + 𝑐𝑦𝑧.
Find 𝑎 + 𝑏 + 𝑐.

Square both sides:


1 2 4 2 9 2 1 3 1
𝑥 + 𝑦 + 𝑧 + 2 ( 𝑥𝑦 − 𝑥𝑧 − 𝑦𝑧) = 0
4 9 16 3 8 2
Simplify:
1 2 4 2 9 2 2 1
𝑥 + 𝑦 + 𝑧 + 𝑥𝑦 − 𝑥𝑧 − 𝑦𝑧 = 0
4 9 16 3 4
Make denominators common and add:
36𝑥 2 + 64𝑦 2 + 81𝑧 2 + 96𝑥𝑦 − 36𝑥𝑧 − 144𝑦𝑧
=0
144
Multiply both sides by 144:
36𝑥 2 + 64𝑦 2 + 81𝑧 2 + 96𝑥𝑦 − 36𝑥𝑧 − 144𝑦𝑧 = 0
36𝑥 2 + 64𝑦 2 + 81𝑧 2 = −96𝑥𝑦 + 36𝑥𝑧 + 144𝑦𝑧

𝑎 + 𝑏 + 𝑐 = −96 + 36 + 144 = 84

B. Rearranging the Identity

10.20: Rearranging Perfect Square of a Trinomial


𝑎2 + 𝑏 2 + 𝑐 2 = (𝑎 + 𝑏 + 𝑐)2 − 2(𝑎𝑏 + 𝑎𝑐 + 𝑐𝑏)

The identity for perfect square can be rearranged usefully to find the sum of the squares of three numbers.
For example, it is useful when doing the sum and product of roots of cubic equations.

Example 10.21
It is customary to assign the variables 𝛼, 𝛽, 𝛾 to the three roots of a cubic equation.

P a g e 82 | 88
Aziz Manva ([email protected])

Substitute 𝑎 = 𝛼, 𝑏 = 𝛽, 𝑐 = 𝛾 in the identity:


𝑎2 + 𝑏 2 + 𝑐 2 = (𝑎 + 𝑏 + 𝑐)2 − 2(𝑎𝑏 + 𝑎𝑐 + 𝑐𝑏)

𝛼 2 + 𝛽 2 + 𝛾 2 = (𝛼 + 𝛽 + 𝛾)2 − 2(𝛼𝛽 + 𝛽𝛾 + 𝛾𝛼)

(Continuation) Example 10.22


Let 𝛼, 𝛽, 𝛾 be the roots of the cubic equation
𝑎𝑥 3 + 𝑏𝑥 2 + 𝑐𝑥 + 𝑑 = 0

If the sum of the roots is 4, and the sum of the product of the roots, when taken two at a time, is 3, then find the
sum of the squares of the roots.

𝛼+𝛽+𝛾 =4
𝛼𝛽 + 𝛽𝛾 + 𝛾𝛼 = 3

𝛼 2 + 𝛽 2 + 𝛾 2 = (𝛼 + 𝛽 + 𝛾)2 − 2(𝛼𝛽 + 𝛽𝛾 + 𝛾𝛼) = 42 − 2(3) = 16 − 6 = 10

Example 10.23
Let 𝛼, 𝛽, 𝛾 be the roots of the cubic equation:
𝑎𝑥 3 + 𝑏𝑥 2 + 𝑐𝑥 + 𝑑 = 0

If the sum of the roots is 2, and the sum of the product of the roots, when taken two at a time, is 4, then find the
sum of the squares of the roots.

𝛼 2 + 𝛽 2 + 𝛾 2 = (𝛼 + 𝛽 + 𝛾)2 − 2(𝛼𝛽 + 𝛽𝛾 + 𝛾𝛼) = 22 − 2(4) = 4 − 8 = −4


𝛼 2 + 𝛽 2 + 𝛾 2 = −4

Note that 𝛼 2 + 𝛽 2 + 𝛾 2 is negative, but this is possible if you consider complex solutions as well.

C. Identity with Reciprocals

Example 10.24
1 1 1
Find 𝑥 2 + 𝑦2 + 𝑧2 if
1 1 1 1 1 1
+ + = 4, + + =3
𝑥 𝑦 𝑧 𝑥𝑦 𝑦𝑧 𝑧𝑥

1 1 1 1 1 1 2 1 1 1
2
+ 2
+ 2
= ( + + ) − 2( + + ) = 42 − 2(3) = 16 − 6 = 10
𝑥 𝑦 𝑧 𝑥 𝑦 𝑧 𝑥𝑦 𝑦𝑧 𝑧𝑥

Example 10.25
The sum of the reciprocals of the roots of a cubic equation is 7. The sum of the reciprocals of the product of the
roots (taken 2 at a time) is 4. Find the sum of the reciprocals of the squares of the roots.

Let the roots be


1 1 1 1 1 1
𝛼, 𝛽, 𝛾 ⇒ + + = 7, + + =4
𝛼 𝛽 𝛾 𝛼𝛽 𝛼𝛾 𝛽𝛾

P a g e 83 | 88
Aziz Manva ([email protected])

1 1 1 1 1 1 2 1 1 1
+ + = ( + + ) − 2 ( + + ) = 72 − 2(4) = 49 − 8 = 41
𝛼 2 𝛽2 𝛾 2 𝛼 𝛽 𝛾 𝛼𝛽 𝛼𝛾 𝛽𝛾

10.4 Applications
There are many different applications of algebraic identities. We look at a few, that are meant to be
representative, rather than comprehensive.
A. Volume

Example 10.26
If a cubical tank with a capacity of 8000 liters has its volume given by the expression 𝑥 3 + 3𝑥 2 + 3𝑥 + 1 𝑚3 , then
find the value of 𝑥 in meters.

1 𝑙𝑖𝑡𝑟𝑒 = 1000 𝑚𝑙 = 1000 𝑐𝑚3


8000 𝑙𝑖𝑡𝑒𝑟𝑠 = 8,000,000 𝑐𝑚3 = 8 𝑚3

1 𝑚3 = (1 𝑚)3 = (100 𝑐𝑚)3 = 1,000,000 𝑐𝑚3

𝑥 3 + 3𝑥 2 + 3𝑥 + 1 𝑚3 = 8 𝑚3
(𝑥 + 1)3 𝑚3 = 8 𝑚3
(𝑥 + 1) 𝑚 = 2 𝑚
𝑥 =1𝑚

Example 10.27
If a cubical tank with a capacity of 8000 liters has its volume given by the expression 𝑥 3 + 3𝑥 2 + 3𝑥 + 1 𝑐𝑚3 ,
then find the value of 𝑥 in meters.

As before:
𝑥 3 + 3𝑥 2 + 3𝑥 + 1 𝑐𝑚3 = 8,000,000 𝑐𝑚3
(𝑥 + 1)3 𝑐𝑚3 = (200 𝑐𝑚)3
(𝑥 + 1) 𝑐𝑚 = 200 𝑐𝑚
𝑥 = 199 𝑐𝑚 = 1.99 𝑚
B. Surface Area as Limiting Value of Volume

Example 10.28
What is the volume of the space between two concentric spheres?

Let:
𝑂𝑢𝑡𝑠𝑖𝑑𝑒 𝑠𝑝ℎ𝑒𝑟𝑒 ℎ𝑎𝑣𝑒 𝑟𝑎𝑑𝑖𝑢𝑠 𝑅, 𝐼𝑛𝑠𝑖𝑑𝑒 𝑠𝑝ℎ𝑒𝑟𝑒 𝑟𝑎𝑑𝑖𝑢𝑠 𝑟
4 3 4 3 4
𝑉𝑜𝑙𝑢𝑚𝑒 𝑜𝑓 𝑆𝑝𝑎𝑐𝑒 = 𝜋𝑅 − 𝜋𝑟 = 𝜋(𝑅 3 − 𝑟 3 )
3 3 3

Factor the above expression using difference of cubes.


Factoring
4
𝜋(𝑅 2 + 𝑅𝑟 + 𝑟 2 )(𝑅 − 𝑟)
3

What is the significance of the:


A. Trinomial

P a g e 84 | 88
Aziz Manva ([email protected])

B. Binomial

𝑇𝑟𝑖𝑛𝑜𝑚𝑖𝑎𝑙 → 𝑆𝑢𝑟𝑓𝑎𝑐𝑒 𝐴𝑟𝑒𝑎


𝐵𝑖𝑛𝑜𝑚𝑖𝑎𝑙 → 𝑇ℎ𝑖𝑐𝑘𝑛𝑒𝑠𝑠 𝑜𝑓 𝑡ℎ𝑒 𝑠𝑝𝑎𝑐𝑒

4
(𝑅 2 + 𝑅𝑟 + 𝑟 2 ) ⏟
𝜋⏟ (𝑅 − 𝑟)
3 𝑆𝑢𝑟𝑓𝑎𝑐𝑒 𝐴𝑟𝑒𝑎 𝑇ℎ𝑖𝑐𝑘𝑛𝑒𝑠𝑠

What happens to the above expression as the thickness becomes close to zero.

Use a change of variable. Let:


𝑅 =𝑟+𝑎

4
Substitute 𝑅 = 𝑟 + 𝑎 in 3 𝜋(𝑅 2 + 𝑅𝑟 + 𝑟 2 )(𝑅 − 𝑟):
4
𝜋((𝑟 + 𝑎)2 + (𝑟 + 𝑎)𝑟 + 𝑟 2 )(𝑟 + 𝑎 − 𝑟)
3
Expand:
4
𝜋[(𝑟 2 + 2𝑎𝑟 + 𝑎2 ) + 𝑟 2 + 𝑎𝑟 + 𝑟 2 ](𝑎)
3
Simplify:
4
[3𝑟 2 + 3𝑎𝑟 + 𝑎2 ] (𝑎)
𝜋⏟ ⏟
3 𝑆𝑢𝑟𝑓𝑎𝑐𝑒 𝐴𝑟𝑒𝑎 𝑇ℎ𝑖𝑐𝑘𝑛𝑒𝑠𝑠

As a becomes very small, the second expression becomes very close to the expression for surface area.
Further, as a becomes very small
3𝑎𝑟 → 𝑣𝑒𝑟𝑦 𝑠𝑚𝑎𝑙𝑙
𝑎2 → 𝑣𝑒𝑟𝑦 𝑠𝑚𝑎𝑙𝑙
4
𝜋 ⏟[3𝑟 2 ] (𝑎)
⏟ ⏟ 2]
= [4𝜋𝑟 (𝑎)

3 𝑆𝑢𝑟𝑓𝑎𝑐𝑒 𝐴𝑟𝑒𝑎 𝑇ℎ𝑖𝑐𝑘𝑛𝑒𝑠𝑠 𝑆𝑢𝑟𝑓𝑎𝑐𝑒 𝐴𝑟𝑒𝑎 𝑇ℎ𝑖𝑐𝑘𝑛𝑒𝑠𝑠

C. Remainders / Sequences

Example 10.29
Consider the sequence:
7,63,215,511,999
A. Find the 𝑛𝑡ℎ term.
B. Find the 218th term
C. Find the remainder when the 218th term is divided by 435.

𝑇1 = 7 = 8 − 1 = 23 − 1
𝑇2 = 63 = 64 − 1 = 43 − 1
𝑇3 = 215 = 216 − 1 = 63 − 1
𝑇4 = 511 = 512 − 1 = 83 − 1
𝑇𝑛 = (2𝑛)3 − 1

𝑇218 = (2 × 218)3 − 1 = 4363 − 1

𝑇218 4363 − 1 (435)(4362 + 436 + 1)


= = = 4362 + 436 + 1 ⇒ 𝑅𝑒𝑚𝑎𝑖𝑛𝑑𝑒𝑟 = 0
435 435 435

P a g e 85 | 88
Aziz Manva ([email protected])

D. Radical Equations

Example 10.30
Solve
3 3
√2𝑥 − 1 + √𝑥 − 1 = 1

𝑎3 + 𝑏 3 + 3𝑎𝑏(𝑎 + 𝑏)
Cube the given equation:
3 3 3
2𝑥 − 1 + 𝑥 − 1 + 3√(2𝑥 − 1)(𝑥 − 1)(√2𝑥 − 1 + √𝑥 − 1) = 1
3
3𝑥 − 2 + 3√(2𝑥 − 1)(𝑥 − 1) = 1
3
3√(2𝑥 − 1)(𝑥 − 1) = 3 − 3𝑥
3
√(2𝑥 − 1)(𝑥 − 1) = 1 − 𝑥

Cube the above equation again:


(2𝑥 − 1)(𝑥 − 1) = (1 − 𝑥)3
(2𝑥 − 1)(−1)(1 − 𝑥) = (1 − 𝑥)3

Case I
1−𝑥 =0⇒𝑥 =1
Try 𝑥 = 1 in the original equation:
3 3
√2(1) − 1 + √1 − 1 = 1 − 0 = 1

Case II
If 1 − 𝑥 ≠ 0, then divide both sides by 1 − 𝑥:
(1 − 2𝑥) = (1 − 𝑥)2
1 − 2𝑥 = 1 − 2𝑥 + 𝑥 2
0 = 𝑥2
𝑥=0

Try 𝑥 = 0 in the original equation:


3 3
√2(0) − 1 + √0 − 1 = 𝐶𝑜𝑚𝑝𝑙𝑒𝑥 𝑁𝑢𝑚𝑏𝑒𝑟𝑠 ⇒ 𝑁𝑜𝑡 𝑉𝑎𝑙𝑖𝑑

10.5 Sophie Germain Identity


10.31: Sophie Germain Identity
𝑎4 + 4𝑏 4 = [(𝑎 + 𝑏)2 + 𝑏 2 ][(𝑎 − 𝑏)2 + 𝑏 2 ]

Add and subtract 4𝑎2 𝑏 2:


(𝑎2 )2 + (2𝑏 2 )2 + 4𝑎2 𝑏 2 − 4𝑎2 𝑏2
= [ ][ ](𝑎2 )2 + (2𝑏 2 )2 + 4𝑎2 𝑏2 − 4𝑎2 𝑏 2

10.6 AMC Questions


Example 10.32
For certain real numbers 𝑎, 𝑏, and 𝑐, the polynomial 𝑔(𝑥) = 𝑥 3 + 𝑎𝑥 2 + 𝑥 + 10 has three distinct roots, and each
root of 𝑔(𝑥) is also a root of the polynomial 𝑓(𝑥) = 𝑥 4 + 𝑥 3 + 𝑏𝑥 2 + 100𝑥 + 𝑐. What is 𝑓(1)? (AMC 10A
2017/24), (AMC 12A 2017/23)

P a g e 86 | 88
Aziz Manva ([email protected])

Method I: Polynomial Division

Method II: Multiplication followed by method of undetermined coefficients

Method III: Root Relation

Example 10.33
The nonzero coefficients of a plynomial 𝑃 with real coefficients are all replaced by their mean to form a
polynomial 𝑄. Which of the following could be a graph of 𝑦 = 𝑃(𝑥) and 𝑦 = 𝑄(𝑥) over the interval −4 ≤ 𝑥 ≤ 4?
(AMC 12A 2002/25)

Example 10.34
The graph of the polynomial 𝑃(𝑥) = 𝑥 5 + 𝑎𝑥 4 + 𝑏𝑥 3 + 𝑐𝑥 2 + 𝑑𝑥 + 𝑒 has five distinct 𝑥-intercepts, one of which
is at (0,0). Which of the following coefficients cannot be zero? (AMC 12A 2003/21)

Example 10.35
Let 𝑃(𝑥) = (𝑥 − 1)(𝑥 − 2)(𝑥 − 3). For how many polynomials 𝑄(𝑥) does there exist a polynomial 𝑅(𝑥) of degree
3 such that 𝑃(𝑄(𝑥)) = 𝑃(𝑥) ⋅ 𝑅(𝑥)? (AMC 12A 2005/24)

Example 10.36
There is a smallest positive real number a such that there exists a positive real number b such that all the roots
of the polynomial 𝑥 3 − 𝑎𝑥 2 + 𝑏𝑥 − 𝑎 are real. In fact, for this value of 𝑎, the value of b is unique. What is the
value of 𝑏? (AMC 12A 2016/24)

Example 10.37
A set S is constructed as follows. To begin, 𝑆 = {0,10}. Repeatedly, as long as possible, if x is an integer root of
some polynomial 𝑎𝑛 𝑥 𝑛 + 𝑎𝑛−1 𝑥 𝑛−1 + ⋯ + 𝑎1 𝑥 + 𝑎0 for some 𝑛 ≥ 1, all of whose coefficients 𝑎𝑖 are elements of
S, then x is put into S. When no more elements can be added to S, how many elements does S have?
(AMC 12A 2017/21)

P a g e 87 | 88
Aziz Manva ([email protected])

Example 10.38
Which of the following polynomials has the greatest real root? (AMC 12A 2018/21)
(𝐴)𝑥 19 + 2018𝑥 11 + 1
(𝐵)𝑥 17 + 2018𝑥11 + 1
(𝐶)𝑥 19 + 2018𝑥 13 + 1
(𝐷)𝑥 17 + 2018𝑥 13 + 1
(𝐸)2019𝑥 + 2018

Example 10.39
Let 𝑠𝑘 denote the sum of the 𝑘th powers of the roots of the polynomial 𝑥 3 − 5𝑥 2 + 8𝑥 − 13. In particular, 𝑠0 = 3,
𝑠1 = 5, and 𝑠2 = 9. Let 𝑎, 𝑏, and 𝑐 be real numbers such that 𝑠𝑘+1 = 𝑎𝑠𝑘 + 𝑏𝑠𝑘−1 + 𝑐𝑠𝑘−2 for 𝑘 = 2, 3, .... What is
𝑎 + 𝑏 + 𝑐? (AMC 12A 2019/17)

Example 10.40
The polynomial 𝑥 3 − 2004𝑥 2 + 𝑚𝑥 + 𝑛 has integer coefficients and three distinct positive zeros. Exactly one of
these is an integer, and it is the sum of the other two. How many values of 𝑛 are possible? (AMC 12B 2004/23)

Example 10.41
Let 𝑎 > 0, and let 𝑃(𝑥) be a polynomial with integer coefficients such that
𝑃(1) = 𝑃(3) = 𝑃(5) = 𝑃(7) = 𝑎, and
𝑃(2) = 𝑃(4) = 𝑃(6) = 𝑃(8) = −𝑎.
What is the smallest possible value of 𝑎? (AMC 8 2010/21)

Example 10.42
Let 𝑃 be a cubic polynomial with 𝑃(0) = 𝑘, 𝑃(1) = 2𝑘, and 𝑃(−1) = 3𝑘. What is 𝑃(2) + 𝑃(−2) ? (AMC 8
2014/16)

Example 10.43
The graph of 𝑦 = 𝑓(𝑥), where 𝑓(𝑥) is a polynomial of degree 3, contains points 𝐴(2,4), 𝐵(3,9), and 𝐶(4,16).
Lines 𝐴𝐵, 𝐴𝐶, and 𝐵𝐶 intersect the graph again at points 𝐷, 𝐸, and 𝐹, respectively, and the sum of the 𝑥-
coordinates of 𝐷, 𝐸, and 𝐹 is 24. What is 𝑓(0)? (AMC 8 2017/23)

Example 10.44
Consider polynomials 𝑃(𝑥) of degree at most 3, each of whose coefficients is an element of
{0, 1, 2, 3, 4, 5, 6, 7, 8, 9}. How many such polynomials satisfy 𝑃(−1) = −9? (AMC 8 2018/22)

45 Examples

P a g e 88 | 88

You might also like